Sunteți pe pagina 1din 2606

Chapter 2 : Capital Structure and Cost of capital

Chapter 18: Cost of Capital Dividend paid is not the cost of equity capital as is normally but erroneously understood. Cost of equity represents the expected return that a firm has to pay to its present and prospective shareholders. There are two methods to determine the cost of equity capital viz. the dividend model and CAPM Capital Asset Pricing Model. As per Dividend Model, Cost of equity capital = (dividend/ market price) + growth rate Under CAPM : Cost of Equity = Risk free return + Beta (Risk Premium) Cost of Equity = Risk free return + Beta ( Market Risk Premium -- Risk free premium) Cost of debentures = Interest rate on debentures x ( 100 -- tax rate in %) Alternatively; cost of debenture = ( Interest -- Tax on interest ) / Net proceeds of debentures Cost of capital = return at zero risk + premium for business risk + premium for financial risk. Cost of debt capital is the interest rate charged by the lender. It can also be calculated by dividing total interest paid by average debt. The effective cost of debt will be ( 1--T) x Rate where T is tax rate applicable to the company.

2001 Dec 5 (b) Question 15 (cwa final dec. 01) In considering the most desirable capital structure of a company, the following estimates of the cost of debt and equity capital (after tax) have been made at various level of Debt-Equity Mix : Debt as % of capital 0 10 20 30 40 50 Cost of debt % 5.0 5.0 5.0 5.5 6.0 6.5 Cost of equity % 12.0 12.0 12.5 13.0 14.0 16.0 Calculate optimal Debt-Equity Mix for the company by calculating composite cost of capital. Solution : Composite cost of capital can be easily calculated as follows: Debt as % of capital 0 10 20 30 40 50 Cost of debt % 5.0 5.0 5.0 5.5 6.0 6.5 Cost of equity % 12.0 12.0 12.5 13.0 14.0 16.0 Composite cost of capital 12.00 11.30 11.00 10.75 10.80 11.25 Composite cost of capital is lowest at debt-equity mix of 30% i.e. 30% debt and 70% equity. Composite cost of capital = 5.5x30% + 13.0x70% = 10.75. similar for other calculations. Similar problem for practice: By varying levels of debt-equity mix, the estimates of the cost of debt and equity capital (after tax) are given below : total capital Debt as % of 0 10 20 30 40 50 Cost of debt in % 7 7 7 8 9 10 Cost of equity in % 15 15 16 17 18 21 You are required to decide on the optimal debt-equity mix for the company by calculating the composite cost of capital. Solution: Composite cost of capital is lowest (14.2%) at Debt % of 20 and 30. Problem : 2002 Dec 14 b The present capital structure of a company is as follows : (in Rs. million) Equity shares (face value Rs. 10) 240 Reserves 360

11% Preference Shares (face value Rs. 10) 120 12% Debentures 120 14% Term loans 360 Additionally the following information are available : Company's equity beta 1.06 Stock market risk premium 6% Yield on long term treasury bonds 10% Corporate tax rate 40% Current ex- dividend equity share price Rs. 15 Current ex- dividend preference share price Rs. 12 Current ex - interest debenture market price Rs. 102.50 per Rs. 100 The debentures are redeemable after 3 years and interest is paid annually. Ignoring the floatation costs, calculate the company's weighted average cost of capital (WACC). Solution : Tutorial notes : 1. As the market price of the share is given, you are to use market value method for computation of cost of equity shares. Reserves will not be taken as component of capital structure. 2. How will you calculate the cost of equity shares? Can you calculate it to be 16.36% orally? Use CAPM and find the cost of equity shares. 3. Cost of preference share is Dividend / Market price. Note that it is not equal to 11% because market price is given. In other cases it can be taken as 11%. 4. You must revise how to calculate the cost of redeemable debentures. 5. When market price of debenture is given and schedule for redemption is also given, the rate of interest on debenture will not be its cost. Hence cost of debentures will not be 12%. What will it be and how it can be calculated? The solution goes as follows : Cost of equity shares is given by CAPM = Risk free return + beta x risk premium Cost of equity = 10% + 1.06 x 6% = 16.36%. Cost of preference shares = Dividend / Market price = 1.10/ 12 = 9.2% Cost of debentures is calculated as follows : 102.50 = 12 / (1 + k) + 12 / ( 1 + k) 2 + 12 / (1 + k) 3 + 100 /(1 + k) 3 By trial and error, k comes to be 11%. You must do this calculation very fast, time factor should be kept in mind while doing such calculations. Computation of weighted average cost WACC at market value weights Sources Amount Pretax cost Cost Post tax Equity shares at market price 360 16.4% 59.0 16.4% Preference shares at market price 144 9.2% 13.2 9.2% Debentures at market price 123 11.0% 13.5 6.6% Term loan 360 14.0% 50.4 8.4% 987 136.2 Pretax WACC = 136.20 / 987 = 13.80% Post tax WACC = 110.60 / 987 = 11.2% Cost 59.0 13.2 8.1 30.2 110.6

Problem for practice : Sachin Enterprises Ltd. provides the following extracts from its accounts as at 31.3.2002 Capital and Reserves 15000000 Debt ICBI Loan (12%) 10000000 Capital Employed 50000000 ACD Loan (13.5%) 25000000 Profit before tax 18000000 Provision for tax 4500000

Profit after tax 13500000 The risk free rate of return is 10% and the premium expected from business in general is 5%. The beta of Sachin ltd. is currently 1.28. (i) Work out the WACC in percentage (accurately to two decimal places). (ii) If the beta is reduced to 1.18 in future, what will be the impact on WACC. Solution ; Tutorial notes : 1. For computing WACC, you should know the cost of capital of each component of capital structure. How would you know it? 2. What is cost of equity ? How will you determine it? 3. What is the use of profit before tax, provision for tax and profit after tax? 4. Will 'capital employed' be part of computing WACC? 5. The cost of capital for equity can be determined with CAPM. Can you calculate it? 6. Tax rate is indirectly given in the question ? Can you identify where it is given and how much is it? 7. WACC will be determined on after tax basis. It can also be on before tax basis. 8. You must use figures in lacs to save time and labour. Working notes : Using CAPM, cost of equity can be determined as Expected rate (cost of equity) = Risk free return + beta x market premium = 10% + 1.28 x 5% = 16.4%. Tax rate is given by : Provision for tax / Profit before tax = 45 / 180 = 25%. Part 1: Determination of Cost of capital with beta as 1.28. Component of Capital Amount in lacs Cost before tax Cost after tax Capital and Reserves 150 16.40% 16.40% ICBI Loan 100 12.00% 9.00% ACD Loan 250 13.50% 10.13% 500 WACC : Total cost / Total amount = 58.91 / 500 = 11.78% Note : Capital employed is not part of capital structure. Part 2: If beta is reduced to 1.18. This will affect the cost of equity. Cost of equity would be 10% + 1.18 x 5% = 15.9%. Component of Capital Amount in lacs Cost before tax Cost after tax Capital and Reserves 150 15.90% 15.90% ICBI Loan 100 12.00% 9.00% ACD Loan 250 13.50% 10.13% 500 WACC : Total cost / Total amount = 58.16/ 500 = 11.62% 2003 June [11] : M/s Smart India ltd has been expanding rapidly during the last two year. The overdraft facility has been fully utilised. The trend of expansion calls for an additional working capital of Rs. 80 lacs. The bank refused to accommodate the company with increased working capital limit. The following information is available for the latest accounts Total assets (Rs. 100 lacs) financed by; (i) Equity with retained earnings (ii) Overdraft 30 Sales 70 Cost of sales Interest Taxes (average) Rs. lacs 200 180 3 50%

No. of shares 20000 of Rs. 100 each Market price per share Rs. 800 The Board of Directors together with their associates are holding 68% of the shares while the remaining has been issued as fully paid up stock. The Board wants to retain absolute control over the company. But it is not in a position to take up new shares. Two alternatives have been given to the Board: (i) Issue just enough shares at the present market price to maintain absolute control. (ii) Restructure the capital with gearing ratio of 1:3. Half the debts will be financed by the bank through term loans and the other half being issued as convertible debentures. You are required to : (a) Comment on alternative proposals and (b) Work out a conversion formula for convertible debentures. Solution : Tutorial Notes 1. This is typical question. It requires some thinking before any attempt to solve. Read the question again and do some thinking. Don't see the solution immediately. 2. Directors do not want to loose control, it means they must have at least 51% total number of shares. If their share goes below 51% they will lose grip over company. 3. The directors do not want to take up new shares. 4. How many shares do the directors hold at present? 5. If the present requirement of Rs. 80 lacs is raised through issue of shares at market price of Rs. 800, 10,000 new shares will have to be issue to general public because the directors do not want to take up any shares. What will be the director's holding after this issue? 6. A solution to above problem may be by issue of bonus shares. Retained earnings may be used for issue of bonus shares. How much is the retained earnings? 7. What is gearing ratio? How it is different from Debt/equity ratio?

bldss ckn tSlk ltsLVSM esa fn;k gS oslk gh gy ys yhft;sA


2004 June 5(a) 9 years Government of India security is quoted at 10.85%. The 364 T Bill is quoted at 11.30. Last year Indian National Bank had issued a fixed rate bond under statutory requirement at 16% coupon for a period of 10 years. Now when remaining 9 years are yet to expire, the bank wants to convert their fixed rate obligation to floating rate due to anticipation of decrease in interest rates. Market quotation for fixed rate to floating rate swap is T bill rate vs. 75 bp over 9 year Government of India security i.e.T Bill rate of 11.60%. If the T Bill declines 20 bp over the current year and rises by 5 bp every year thereafter, what is the effective cost of funds to Indian National Bank for 10 year period. Solution : Treasury Bills : Short term borrowing instruments for GOI. Period of maturity is less than one year. A bill of say Rs. 975 (discounted value) will be paid as Rs. 1,000 (face value) after the expiry of say 364 days is an example of Treasury Bill. T bill rate is quoted for a period less than a year. It is quoted currently at 11.30. 11.30 means 11.30%. This rate changes with time. The question says this rate will decline by 20 basis point (means 0.20%) over the current year (11.30%) and then rises by 5 basis point (0.05%) every year. Market quotation for fixed to floatation rate swap is 75 bp (basis point) over current rate of 10.85. Thus the quotation is 10.85 + 0.75 = 11.60. Quotation means if any one wants to change from fixed rate to fluctuation rate, he will pay 11.60% for the swap every year.

Year 2 3 4 5 6 7 T bill rate 11.30 11.10 11.15 11.20 11.25 11.30 fixed rate 16.00 16.00 16.00 16.00 16.00 16.00 Quotation 11.60 11.60 11.60 11.60 11.60 11.60 Effective 15.70 15.50 15.55 15.60 15.65 15.70 Effective rate of interest = Fixed rate + Quoted rate for swap T bill rate. Effective cost of funds is NOT the average of above effective rate. Effective cost of funds is calculated as follows : Year annual cost Amount (principal + interest) Principal 1 0.160 1.160 1 2 0.157 1.157 1 3 0.155 1.155 1 4 0.156 1.156 1 5 0.156 1.156 1 6 0.157 1.157 1 7 0.157 1.157 1 8 0.158 1.158 1 9 0.158 1.158 1 10 0.159 1.159 1 11.571 10 Effective cost of fund = 11.571 10.0 = 1.571 = 15.71%. 2004 June (8) You work in AB Ltd. You finance manager plans for acquiring YZ Ltd. following data:

8 11.35 16.00 11.60 15.75

9 11.40 16.00 11.60 15.80

You are provided with the

AB Ltd. YZ Ltd. Expected earning per share Rs. 12 Rs. 5 Expected dividend per share Rs. 8 Rs. 2 No. of shares 20 lakhs 12 lakhs Current market price Rs. 180 Rs. 50 Your estimates about YZ indicate expected steady growth of earnings and dividend to the tune of 6% per annum. However, under the new management the growth rate is likely to go up to 8% per annum without additional investment. (a) You are asked to calculate the cost of acquisition by AB Ltd. if Rs. 60 is paid for each share of YZ Ltd. (b) If the agreed exchange ratio is one share of AB Ltd. for every three shares of YZ Ltd. in lieu of the cash acquisition as per (a) above, what will be the net cost? (c) Compute the gain from acquisition. (d) If the expected growth rate continues to be 6% per annum, how will the new share price as well as cost be different? Solution : Tutorial Notes : (a) It is the easiest part. You can calculate it to be Rs. 120 lakhs orally. Solve it before you see any further. First answer is ready. Value of YZ is Rs. 600 lakhs (Rs. 50 x 60 lakhs). The cash payable is Rs. 720 lakhs (Rs. 60 x 12 lakhs) or (Rs. 180 x 12/3 lakhs). Cost of acquisition is Rs. 120 lakhs.

(b) For this point you must revise various methods for valuation of merger. Cost of merger represents the excess amount paid over the current market price of the target company's shares. Costs here would mean the price that AB ltd. would pay to the shareholders of YZ Ltd in excess of the current market value of their (YZ ltd's) own shares. What is current market value of YZ? Since growth rate is mentioned in the question, growth rate (futuristic) should be taken into account for valuation purposes. The dividend of Rs. 2 will be the base of valuation. The current rate of capitalization is given indirectly for valuation of YZ ltd after merger. Value of the firm after merger would be computed as follows : Rs. lacs Present market value of acquirer company (AB Ltd.) 20 x Rs. 180 3,600 Future value of target company (YZ Ltd.) [Dividend / (cost of capital growth)] x no. of shares Cost of equity (or cost of capital) can be calculated with dividend growth formula: Cost of equity = (Dividend / Market Price) + Growth rate. It should be noted here that in the question, two growth rates are given at 6% and 8%. The 6% growth rate will be considered to compute the cost of capital of YZ ltd while growth rate of 8% would be considered for post merger computation. Cost of capital of YZ Ltd = 2/50 + 6% i.e. 4% + 6% = 10%. Future value of target company (YZ Ltd.) = [Dividend / ( 10% 8%) ] x no. of shares = [2 / 0.02 ] x 12 lakhs = Rs. 1200 lacs. Present market value of acquirer company (AB Ltd.) 3,600 Future value of target company 1,200 Total value after merger in Rs. lacs. 4,800 Total number of shares after merger No. of existing share in lacs 20 Issue shares for merger (12 lacs/3) 4 Total number of shares after merger 24 Price per share after merger (4,800 / 24) : Rs. 200. The target company will receive 400,000 shares of Rs. 200 each. Total consideration being paid for YZ Ltd. (200 x 4 lacs shares) 800 Less : Current market price of YZ Ltd. (12 lacs shares at Rs. 50 each) 600 Cost of merger in Rs. lacs 200 (c) Gain from merger is value of synergy of merger. Gain from merger = Value after merger less aggregate of individual values of AB and YZ Gain from merger = 4,800 (3,600 + 6,00) = 600 lacs. (d) If the cash is paid for acquisition, future growth becomes irrelevant. The cost will remain Rs. 120 lakhs. However, if payment is made in terms of shares, the value will be decided with the after merger share price. The new share price will be Rs. 175 as shown below. You should compute it yourself before you see the following. Present market value of acquirer company (AB Ltd.) 3,600 Future value of target company (Rs. 50 x 12 lakhs) 600 Total value after merger in Rs. lacs. 4,200 Total number of shares after merger No. of existing share in lacs Issue shares for merger (12 lacs/3) Total number of shares after merger

20 4 24

Price per share after merger (4,200 / 24) : Rs.175 Gross cost of YZ : Rs. 175 x 4 lakhs 700 Current market price : Rs. 50 x 12 lakhs 600 Net cost of merger Rs. lakhs 100 2004 Dec 2a : Some years ago Shibaji Ltd sub-divided its capital into ordinary shares of one rupee each. Presently, the company has annual earnings before interest and tax of Rs. 1.5 crore. These earnings are expected to remain constant. The market price per share of the company is Rs. 3.44 cum-dividend and per debenture is Rs. 105.50 ex-interest. An interim dividend of 24 paise per share has been declared. Tax rate is 30% and Shibaji long term capital structure is shown below: (figures in Rs.'000) Ordinary shares of Rupee one each 12,500 Reserves 24,300 36,800 16% debentures 31.12.2004 (Rs. 100) 23,697 60,497 Required :Calculate cost of capital of Shibaji Ltd. according to the traditional theory of capital structure as on 31.12.2004. Market value of equity and debt should be applied as weights. Solution ; Tutorial Notes : 1. How many theories of capital structure are there? Revise them. 2. EBIT is given and market price of share is also given. Where will you use this information in relation to determination of cost of capital? 3. What is cost of equity ? You can determine it with CAPM or with dividend growth formula. What will you apply and why? 4. How will you determine the cost of debt? Revise its theory portion before you proceed any further. Cost of equity capital : Cost of equity capital : Dividend / Net proceeds per share Or Dividend / Market price per share. If growth is also taken into account: Cost of equity capital = ( D/ NP ) + Growth Or ( D/ MP ) + Growth In this question, this formula will not be applicable as the dividend is not given. Can you think of some other method or formula which can be used for determining the cost of equity capital? Cost of equity capital can also be found out by using Earning yield method or earning price ratio method as given below. Stress your brain and think what these methods are? Cost of equity = Earning per share / Net Proceeds Or Earning per share / Market price per share. Please use this formula and determine the cost of equity before you see below. What is market price of the share to be used ? Is it Rs. 3.44 as given in the question or would it be something different? EBIT in Rs. lacs 1,500 Interest @ 16% of Rs. 2369.7 lacs 379 EBT 1,121 Less : Taxes @ 30% 336 Earnings after tax EAT 785 Cost of equity = Earning per share / Net Proceeds Or Earning per share / Market price per share. Here the question comes what would be the market price?

The market price is Rs. 3.44 cum dividend less interim dividend 0.44 i.e. Rs. 3.20 Cost of equity = (785 / 12,500) / 3.2 = 19.6% Cost of debentures you have already determined in earlier questions, try to determine it before you see the following: 105.50 = 16 / (1 + k) + 16 / ( 1 + k) 2 + 16 / (1 + k) 3 + 100 /(1 + k) 3 By trial and error you find that k = 9% satisfies the equation. WACC can now be computed as follows: Rs. lakhs Cost % Equity share capital : 125 lakh shares @ Rs. 3.2 per share 400 19.6% Debentures : 236,970 debentures @ Rs. 105.50 per debenture 250 9.0% WACC : (400 x 19.6%) / (65) + (250 x 9%) /(65) = 15.5%. 2004 Dec 2b: Akbar ltd. is an all equity company with an equilibrium market value of Rs. 32.5 million and a cost of capital of 18% per year. The company proposes to repurchase Rs. 5 million of equity and to replace it with 13% irredeemable loan stock. Akbar's earnings before interest and tax are expected to be constant for the foreseeable future.. The company' tax rate is 30%. All profits are distributed as dividend. Required : Using the assumption of Modigiani and Miller explain and demonstrate how this change in capital structure will affect (i) the market value (ii) the cost of equity and (iii) the cost of capital of Akbar Ltd. Solution : Tutorial Notes : 1. MM theory of relevance takes taxes into account while MM theory of irrelevance assumes that there are no taxes. 2. MM theory states that the value of the firm can be increased up to a certain limit with the use of debt in the capital structure because the interest charges on debt are tax deductible expenses. Use of debt also reduces the cost of capital up to a certain limit because debt interest charges are less than what the firm is earning out of that debt. 3. Value of the levered firm (value of firm with debt) = Value of unlevered firm + tax rate x debt. 4. The leverage will increase the value of the firm to the tune of tax saving on the debt. 5. Value of the unlevered firm is given by EBIT x ( 1 tax rate) / Overall cost of capital. 6. Using above information and knowledge, please try to solve this problem before you see below: Overall value of the firm = Value of unlevered firm + Value of debt x tax rate = 32.50 + 0.30 x 5.0 = Rs. 34.0 million. The market value will go up to Rs. 34.0 million i.e. an increase of Rs. 1.50 million. 7. Cost of capital will decrease with the use of debt, up to a certain limit. It means the cost of equity will increase with the use of debt because presence of debt is an indicator of financial risk. As seen above, the value of firm is Rs. 34 million consisting of Rs. 29 million for equity and Rs. 5 million for debt. The tax rate is 30%. The WACC is easy to determine Component of Capital Equity share capital Debt Amount After tax cost Cost 29 18% 18.0% 5.22 5 13% ( 1 0.3) 9.1% 0.46 34 5.68 WACC : 5.68 / 34 = 17% The new cost of equity can now be determined when overall cost of capital is known. The cost of equity should increase because of presence of debt.

WACC = K x [ Equity / (equity + debt)] + Debt interest x (1 t) x [D / (equity + debt)] 17 = K x (29/34) + 0.13 x ( 1 0.30) x (5/34) Giving K i.e. cost of equity = 18.6%. The cost of equity has increased from 18% to 18.6% because of presence of financial risk. 2004 Dec 6a Donald Enterprises has warrants issued that allow the holder to purchase 4 shares of a stock for a total of Rs. 80 for each warrant. Currently the market price per share of Donald equity is Rs. 16. Investors held the following probabilities of benefits about the stock 6 months later. Market price per share Rs. 10 16 20 23 27 Probability 0.10 0.20 0.30 0.25 0.15 You are required to answer the following : (i) What is the present theoretical value of the warrant? (ii) What is the expected value of the stock price six months hence/ (iii) What is expected value of theoretical value of the warrant 6 months hence? (iv) Would you expect the present market price of the warrant to equal the theoretical value? If not, why not? Solution : Tutorial notes : Revise some theory related to Warrants : 1.Warrant is a right which entitles the holder of warrant to buy a fixed number of shares at a particular price during a particular time period. 2. Exercise price : It is the price at which the holder can buy the shares. 3. Exercise ratio : It shown one warrant equals to so many shares. 4. If exercise price is say Rs. 50 and the share is already available at say Rs. 48, in this case no will exercise the right attached with warrant because it would be better to buy the share at Rs. 48 against the buying the share at Rs. 50 through warrant. Thus in cases where market price is less that the exercise price of warrant, the value of warrant is zero. 5. Premium is gain in percentage. If market price is say Rs. 30 and exercise price is say Rs. 20. The premium would be Rs. 10 i.e 50% on exercise price. The solution goes as follows: Market price per share Rs. 10 16 20 23 27 Probability 0.10 0.20 0.30 0.25 0.15 Expected market price 1.0 3.2 6 5.75 4.05 20.0 (i) The exercise price on the warrant is Rs. 80 for 4 shares i.e. Rs. 20 per share. The present market price is Rs. 16 per share. The holder will get share from the market at Rs. 16 rather than buying them through warrants at Rs.20 per share. Thus the warrant is of no use to the holder or in other words the value of warrant is zero. (ii) Expected value as calculated above is Rs. 20. (iii) What is expected value of theoretical value of the warrant 6 months hence? Market price per share Rs. 10 16 20 23 27 Probability 0.10 0.20 0.30 0.25 0.15 Value of warrant 0 0 0 3 7 Expected value of warrant 0 0 0 0.75 1.05 1.8 Expected value per share of warrant from 6 months hence is Rs. 1.8 per share.

(iv) The current value of warrant is zero and expected value after six months is Rs. 1.8 per share i.e. Rs. 7.20 per warrant. The warrant has a positive value. 2005 June 6b: XYZ company is contemplating to undertake the following investment proposals; (i) expansion of existing capacity and (ii) setting up new project not related with the present business. The company's shares are regularly traded in stock exchanges and the recent estimates of share beta is 1.2. The debt equity ratio of the company at present is 2:1. However, the new project, if undertaken, can be financed with a debt equity ratio of 1:1, since the company has adequate internal accruals. To assess business financial risk of the proposed new project, the XYZ company has identified a comparable company with debt equity mix of 1.5:1 and estimated share beta of 1.5. The comparable company is subject to an effective tax rate of 20% where the effective tax rate of XYZ company is 30%. You are required to estimate the following : (i) Cost of capital to be used as cut-off rate to evaluate the expansion project, assuming that its debt-equity ratio will be 2:1. and (ii) Cost of capital to be used as cut-off rate to evaluate the new project and (iii) Based on your calculations in (i) and (ii) above, would you recommend different cut-off rates or single cut-off rate for the expansion and new projects? Give reasons. You may further like to consider the following information while answering the above question : (a) The company will be able to negotiate loan at 15%. (b) The present yield on long dated government securities is around 7%. (c) Expected spread between return on stock index and government securities (i.e. market premium) 6%. (d) Estimated liquidity premium in the government securities is one percent. (e) Effective corporate tax rate of XYZ company will continue to be 30% even after the project being undertaken. Solution : Tutorial Notes : 1. The cost of capital of expansion project will be WACC of equity and debt combination. The amount of equity and debt is not given, however, ratio is given which is enough to compute the WACC. 2. For computing WACC, you need to have cost of equity and cost of debt. Both these parameters are supplied but secretly. Use CAPM and determine the cost of equity capital. Cost of debt is already supplied as 15%. Reduce it by tax component and determine the cut off rate. Your first answer is ready.

2005 June 8b 'Just born' is a newly formed firm providing garments for babies and children. It has forecast its total fund requirements for the coming year as follows: (figures in Rs. lakhs) Month Total requirement Permanent Requirement Seasonal Requirement January 8,500 6,900 1,600 February 8,000 6,900 1,100 March 7,500 6,900 600

April 7,000 6,900 100 May 6,900 6,900 0 June 7,150 6,900 250 July 8,000 6,900 1,100 August 8,350 6,900 1,450 September 8,500 6,900 1,600 October 9,000 6,900 2,100 November 8,000 6,900 1,100 December 7,500 6,900 600 The firm's cost of short-term and long-term financing is expected to be 9% and 15% respectively. Calculate the cost of financing using : (i) hedging approach (ii) conservative approach (iii) trade-off approach. 2006 June 4a: From the following details of HPL Ltd., calculate the cost of capital; Debt Amount Nominal interest Foreign Loan 100 Million US Dollars 5% Local currency Loan Rs. 2,200 Million 12% Expected depreciation of Rupee : 3% per annum. Current exchange rate : Rs. 45 per US dollar. Bank / FI guarantee for raising foreign capital : 1% Equity Capital : Rs. 3000 Million ; Unlevered Beta : 0.60 ; Risk free rate ; 6% and Market premium : 8% Tax rate : 30% Solution: 1.Cost of equity can be obtained by CAPM. This is levered firm (it means this firm has debt in its capital structure) and the beta is given for unlevered firm (all equity firm). Do you know how to calculate beta of levered firm with beta of unlevered firm? 2. Cost of equity can only be obtained when you know beta of levered firm. Find beta for levered firm. Use it in CAPM to find out the cost of equity capital. 3. Cost of foreign loan would be 5% + 3% + 1% = 9% and cost of local loan is given as 12%. You should determine the cost of total debt before tax and after tax. 4. Try to solve it on the above guide lines before you see below: Working note 1: Composite cost of debt : Figures in Rs. million. Component Amount rate in % Interest Foreign Loan 4,500 9% 405 (100 mdollar = 4500 mRs.) Local Loan 2,200 12% 264 6,700 669 Composite cost of debt (before tax) : (669/ 6,700) = 10% Composite cost of debt (after tax) : ( 1 tax rate) (before tax cost) = ( 1 0.30) 10% = 7% Working note : Computation of Cost of equity Computation of beta of the firm : ( Beta levered) / Total fund = Beta unlevered / Equity. If after tax figure is to be computed which is generally the case, the debt is taken as debt (1 tax rate). Total debt is 6,700 mRs. and equity is given as 3,000 mRs. Unlevered beta is given as 0.60. 0.60 / 3,000 = beta / [(1 0.30) x 6,700 + 3,000] which gives : Beta = 0.60 x (7,690 / 3,000) = 1.54.

Having known the beta of the firm, you can determine the cost of equity by using CAPM: Cost of equity = Risk free return + beta x Market premium (or risk premium) Cost of equity = 6% + 1.54 x 8% = 18.32%. Statement of computation of WACC : Component Amount Cost Cost Debt 6,700 7% 469 Equity 3,000 18.30% 549 9,700 1,018 WACC = 1,018 / 9,700) = 10.50% 2006 Dec 2b: Pioneer Technology Ltd. is foreseeing a growth rate of 12% per annum in the next 2 years. The growth rate is likely to fall to 10% for the third year and fourth year. After that the growth rate is expected to stabilize at 8% per annum. If the last dividend paid was Rs.1.50 per share and the investor's required rate of return is 16%, what would be the intrinsic value of the Pioneer Ltd as of date? Years 0 1 2 3 4 5 PV at 16% 1 0.86 0.74 0.64 0.55 0.48 Solution: Valuation of shares in parts and all parts will be added to get the aggregate value of shares. Years Growth Dividend Pv factor Pv 0 0 1.5 1 12% 1.50 x 1.12 1.68 0.86 1.44 2 12% 1.68 x 1.12 1.88 0.74 1.39 3 10% 1.88 x 1.10 2.07 0.64 1.32 4 10% 2.07 x 1.10 2.28 0.55 1.25 5 8% 30.75 0.55 16.91 (see working note below) 22.33 Working note: After fourth year the dividend has a constant growth rate of 8%, the value of share can be obtained by constant growth formula as given below: Cost of capital or Required rate of return = (Dividend / Market Price) + Growth All parameters are supplied with the question except the market price which is also value after four years. 16% = (2.28 x 1.08) / MP + 8% Or 8% x MP = 2.46 which gives MP = 2.46/0.08 = Rs. 30.75 2006 Dec 4b: In 2000 AT & T acquired NCR after a hotly contested takeover for approximately 110 per share. The free cash flows of the two firms before and after merger, were as follows: Year 1 2 3 4 5 Terminal AT &T 4,684 4,918 5,164 5,422 5,693 82,756 NCR 471 509 550 594 641 8,102 Combined Post merger 5195 5558 5948 6364 6809 97672 Note : The terminal value is at the end of 5th year. Cost of equity and debt of the individual firms and the combined firm (after merger) were estimated as given under: AT &T NCR Combined Cost of equity 14.23% 15.33% 14.34% Cost of debt 5.40% 6.00% 5.42%

Debt /(Debt + Equity) 21% 9% 20% At the time of merger deal NCR had 70.6 million outstanding shares and 537 million USD worth of outstanding debt. (a) What is the minimum price per share AT &T could have offered to NCR? (b) Do you think that the price paid by AT&T was justifiable? Give reasons. Support your answer with calculations. t You may use the formula (1/(1+r) for computation of cost of capital. 2006 Dec 6b: You are running a portfolio management business and have assembled the following portfolio for Client -B.
You are running a portfolio management business and have assembled the following information for client B : Scrip Infosys Hind. Lever Reliance Tata Motors Pfzer Value Rs. Lakhs 5.0 4.0 6.0 3.0 2.0 Beta 1.21 0.97 1.40 1.32 1.25

Client B insists that the portfolio should comprise the above 5 scrips and that each scrip should be at least 10% of the total portfolio value. You project the Sensex which is currently 11400 to move to 11800 by the end of 3 months and to 12200 by the end of 6 months.

(i) What will be the value of your portfolio at the end of 3 months and 6 months ? (ii) What could you do to improve the portfolio performance, Give your view on the market ? This is a very simple question. Solution : Scrip Value Rs. Lakhs Weight Beta Portfolio Beta Infosys 5.0 0.25 1.21 0.303 Hind. Lever 4.0 0.20 0.97 0.194 Reliance 6.0 0.30 1.40 0.420 Tata Motors 3.0 0.15 1.32 0.198 Pfzer 2.0 0.10 1.25 0.125 20.0 1.00 1.240 Value of Portfolio : Period Sensex % increase sensexPortfolio increase value Current 11400 3 months 11800 3.51% 4.35% 20.86 6 months 12200 7.02% 8.70% 21.74
Question 14 (cwa final dec. 06) Multisoft ltd. is expected to grow at a higher rate of 4 years; thereafter the growth rate will fall and stabilize at a lower level. The following information has been assembled : Base year (yr. 0) information Revenue EBIT Capital expenditure Depreciation Rs. Million 3,000 500 350 250

Working capital as % of revenue Corporate tax rate (all time) Paid up Equity capital (Rs. 10 par) Market value of debt

25% 30% 400 1,200

Inputs for the high growth period Length of high growth period 4 years Growth rate in revenue,depreciation,EBIT and Capital expdt. 20% Working capital as % of revenue 25% Cost of Debt (pre-tax) 13% Debt-equity ratio 1:01 Risk-free rate 11% Market risk premium 7% Equity Beta 1.129 Inputs for the stable growth period Expected growth rate in revenues and EBIT 10% Capital expenditure are offset by depreciation 0 Working capital as % of revenue 25% Cost of Debt (pre-tax) 12.14% Debt-equity ratio 2:03 Risk-free rate 10% Market risk premium 6% Equity Beta 1.00 Year 1 2 3 PVIF at 13% 0.885 0.783 0.693 14% 0.877 0.769 0.675 15% 0.870 0.756 0.658 (i) What is the Weightage Average Cost of capital (WACC) for the high growth period and stable growth period. (ii) What is the value of Multisoft ltd ? Solution: Tutorial Notes : Learn the following points thoroughly. 1. A firm with free cash flows to the firm (FCFF) at a stable growth rate can be valued using the following model: Value of the firm = FCFF1 / (WACC growth rate in FCFF) Where : FCFF1 Expected FCFF next year ; WACC : Weighted average cost of capital Please note that present value of cash flows in perpetuity is simply cash flows / discount rate. If these cash flows are growing at a rate of 'g', then the present value of perpetuity is cash flows / (discount rate growth rate). A firm with free cash flows to the firm (FCFF) at a stable growth rate can be valued using the following model:

2. The value of the firm, in the most general case, can be written as the present value of expected free cash flows to the firm: Value of the firm = FCFFt / ( 1 + WACC) From t = 1 to infinity 3. If the firm reaches steady state after n years, and starts growing at a stable growth rate gn after that the value of the firm can be written as Value of the firm = FCFFt / ( 1 + WACC) (From t = 1 to n) + [(FCFF n+1 / WACC growth rate)] / ( 1 + WACC) (i) Computation of WACC for high growth period : CAPM Basis High growth period Cost of equity (11% + 1.129 x 7%) 18.90% Cost of debt (tax rate 30%) (13% x 0.70) 9.10% Debt:Equity ratio 1:01

WACC

( 0.50 x 18.9% + 0.50 x 9.10%) Stable growth period (10% + 1.0 x 6%) (12.14% x 0.70)

14% 16.00% 8.50% 2:03 13% Rs.million 4 20% 6,221 1,037 311 726 1,555 259 207 259

Cost of equity Cost of debt (tax rate 30%) Debt:Equity ratio WACC ( 0.40 x 16.0% + 0.60 x 8.50%) (ii) Value of Multisoft Ltd. Year 0 1 Growth rate 20% Revenues 3,000 3,600 EBIT 500 600 Less Tax @ 30% 150 180 EAT 350 420 Working capital (25% of revenue) 750 900 Increase in working capital 150 Capital expenditure 100 120 Cash inflow * 150 *(EAT -- Increase in working capital -- capital expdt.) Cost of capital during the growth period is 14% as calculated above in (i) Pv factor @ 14% for growth period 0.877 Present value of cash inflow 131.6 Present value of terminal value of 642.8 = [642.8 / (0.13 -- 0.10)] x 0.592 Total Value of Multisoft Ltd. Rs. Million

2 20% 4,320 720 216 504 1,080 180 144 180

3 20% 5,184 864 259 605 1,296 216 173 216

0.769 138.4

0.675 145.8

0.592 153.4

2007 June 2a:


Zenith Ltd has a surplus cash of Rs. 100 lakhs and wants to distribute 35% of it to the shareholders. The company decides to buyback shares. The General Manager of the company estimates that its share price after repurchase is likely to be 15% above the buyback price, if the buy back route is taken, The number of shares outstanding at present is 10 lakhs and the current EPS is Rs. 4 You are required to determine (i) the price at which the shares can be repurchased if the market capitalization of the company should be Rs. 230 lakhs after buyback;

(ii) the number of shares that can be repurchased and (ii) the impact of share repurchase on the EPS assuming the net income is same. Solution : If the buy back price is say Z, the after buy back price would be 1.15Z (15% above buy back price).
The company wants to pay Rs. 35 lacs (35% of surplus) to shareholder at a price Z. No. of shares it can buy is (35 lakhs / Z). Total no. of shares is 10 lakhs out of which (35 lakh/z) shares are bought back. No. of share after buyback would be (10 35 lakhs/ Z).

Amount after buy back : (10 35/z) Lakhs @ Rs. 1.15 Z per share. The value after buy back is given as Rs. 230 lakhs, we have the following equation: (10 35/z) 1.15 z = 230 which gives z = Rs. 23.50 per share.
The price at which the shares should be bought back is Rs. 23.50, the amount of buy back is Rs. 35 lakhs, no. of shares to be bought back would be 35,00,000 / 23.50 = 148,944 shares.

Total number of outstanding shares ; Shares bought back Outstanding shares after the buy back

10.00 lakhs 1.49 lakhs 9.51

How can you compute the EPS after the buyback? The EPS is ratio of total earnings and total number of shares. What is total earning? Read the question again and calculate the total earning. How many outstanding shares are there? Divide the total earnings with number of outstanding shares and you have the EPS. Do it yourself before you see below.

Total earnings : Earning per share x no. of outstanding shares = 4 x 10 = Rs. 40 lakhs Outstanding shares after the buy back : 10 1.49 = 8.51 lakhs. EPS = 40 / 8.51 = Rs. 4.70. 2007 June 3b:
Neel Ltd. is in Hospitality services. The company financials are as follows : Rs. in lakhs Sales 100 EBDIT 60 Depreciation 10 Tax rate 40% Net Fixed assets 80 Sales Growth estimated in next 5 years 10% p.a. Operating Expenses grow by 8% p.a. Depreciation 10% Post-tax cost of debt 8% Market value of debt 40 Cost of equity 15% Market value of equity 110 Estimated Growth of free cash flow from 6th year 10%p.a. Net Current assets (will remain at 10% of Net fixed assets in next 5 years) 10 Year wise Investment in Fixed assets : Year 1 2 3 4 5 Investment in fixed assets Rs. lacs 20 0 10 15 0 Post-tax non-operating cash flows 10 0 5 20 0 You are required to find out the value of Neel Enterprises Ltd. Note : You may use the formula : 1/(1+r)n for determining PVIF at r (rate for cost of capital.) Solution : Tutorial notes : 1. Operating expenses grow by 8% p.a. How much are the operating expenses, has not been mentioned in the question. It is hidden information. Think where it is hidden. 2. Cost of capital is to be determined. Can you determine it right now ? 3. Depreciation is given as 10%. Fixed assets have to be computed year wise. Attempt to compute. 4. The current assets are 10% of fixed assets. The investment in current assets has to be calculated yaws. This is cash outflow. 5. Most students are likely to forget about the post-tax non-operating cash flows. These are to be added to the operating cash flows to determine free cash flow. Working notes: Rs. Lacs 1.Determination of cost of capital : Market value of debt 40 Post tax cost of debt 8% Market value of equity 110 Cost of equity 15% Cost of capital ( 0.08 x 40 +0.15 x 110) / ( 40 + 110 ) 13.31% 2. Value of Fixed assets year wise : Rs. in lacs Year 1 2 3 4 5 Opening balance 80.0 90.0 81.0 81.9 87.2 Investment in fixed assets 20.0 0.0 10.0 15.0 0.0 Total 100.0 90.0 91.0 96.9 87.2 Less Depreciation @ 10% 10.0 9.0 9.1 9.7 8.7 Net fixed assets at the end of the yr. 90.0 81.0 81.9 87.2 78.5

3. Net Current assets (@ 10% of fixed assets at the end of the year) Opening balance 10.0 9.0 8.1 Current assets at the end of the yr. 9.0 8.1 8.2 Investment in current assets (1.00) (0.90) 0.1 4. Net Investment in assets 19.0 (0.90) 10.1 5. Operating Expenses (Sales - EBDIT) = 100 60 = Rs. 40 lakhs 6. Operating expen. Growth 8% 43.2 46.7 50.4 Computation of expected Free cash flow for the years fig. in Rs. lacs Sales ( Growth 10%) Less : EBIT Earning after tax (60% of EBIT) add : Depreciation Cash inflow Less : Investment Net Cash inflow from operations Add:non-operating cash flows Free Cash Flow Pv factor at 13.13% Operating expenses Depreciation 110.0 43.2 10.0 56.8 34.1 10.0 44.1 19.0 25.1 10.0 35.1 0.88 121.0 46.7 9.0 65.3 39.2 9.0 48.2 (0.90) 49.1 0.0 49.1 0.78 133.1 50.4 9.1 73.6 44.2 9.1 53.3 10.1 43.2 5.0 48.2 0.69

8.2 8.7 0.5 15.5 54.4 146.4 54.4 9.7 82.3 49.4 9.7 59.1 15.5 43.5 20.0 63.5 0.61

8.7 7.8 (0.87) (0.87) 58.8 161.1 58.8 8.7 93.6 56.1 8.7 64.9 (0.87) 65.7 0.0 65.7 0.54 35.5

Pv of Cash inflow 31.0 38.4 33.3 38.8 Value of Neel Enter.ltd is = 176.9 + 1.10 x 35.50 / (0.1313 0.10) = Rs.1424.50 lacs.

2007 Dec 2a:


Novelty Ltd., a consumer durable manufacturer, reported earnings per share of Rs. 3.20 in 2007 and paid dividend per share of Rs. 1.70 per share in that year. The firm reported depreciation of Rs. 350 lakhs in 2007 and capital expenditure of Rs. 475 lakh. There were 160 lakh outstanding shares traded at Rs. 51 per share. The ratio of capital expenditure to depreciation is expected to be maintained in the long term. The working capital needs are negligible. Novelty had a debt outstanding of Rs. 1600 lakh and intends to maintain its current financing mix of debt and equity to finance future investment needs. The firm is in the steady state, and earnings are expected to grow at 7% per year. The stock had a beta of 1.05, the treasury bill rate is 6.25% and the market premium is 5.5%.

Requirements :
(i) Estimate the value per share using (i) the dividend discount model, (ii) the FCFE model (Free cash flow to equity). (ii) How would explain the difference between the two models, and which one would you use as a bench mark to compare with the market price?

Solution : Tutorial notes : 1. You must revise the theory portion related to dividend discount model and FCFE model.
2. In which model can use the treasury bill rate, market premium and beta? What you can calculate with this model?

3. The dividend discount model is given by Cost of equity = (Dividend / Market price) + Growth 4. The FCFE model is given by :
FCFE = Net Income (Capital expenditure Depreciation)(1 Debt financing ratio) Change in Working capital (1 Debt financing ratio)

Depreciation per share is 350/160 and capital expenditure per share is 475/160. Debt financing ratio = Debt / (Debt + Equity). The problem can be solved on following lines :

1. Find out cost of equity capital with CAPM.


2. Use dividend growth model to know market price of the share which is also its value. Note that dividend declared is 1.7 per share which is expected to grow at 7%.

3. Use FCFE model to know the value, compare the values and offer your comment.
Based on above knowledge, you should solve the above problem and compare your solution with the one given below :

Determination of cost of equity with CAPM: Cost of equity = Risk free return + beta x Market premium = 6.25% + 1.05 x 5.5% = 12%. Using Dividend growth model : 12% = 1.7 x 1.07 / MP + 7% Which gives MP = Value of equity = 1.7 x 1.07 / 0.05 = Rs. 36.4, Thus the value of share as per dividend growth model is Rs. 36.4. FCFE model also gives you the value of equity as follows: Net income is given as Rs. 3.2 per share. Depreciation per share is 350 / 160 = Rs. 2.2 per share. Capital expenditure per share is 475/160 = 3.0 per share Debt financing ratio = Debt / (Debt + Equity) = 1,600 / [1,600 + (160 x 51)] = 0.16 FCFE = 3.2 ( 3.0 2.2)(1 0.16) 0 = 2.6. Value of share : Capitalization rate will be cost of capital less growth i.e. 12% 7% = 5%. Value of equity = FCFE x 1.07 / 5% = 2.6 x 1.07 / 5% = Rs. 55.6 Comments :
The FCFE takes into account, not only the dividend paid by the firm but also the cash accumulated in the firm as a whole. This explains the higher value of the firm when FCFE is used for computation of firm's value. FCFE is recommended as it is more realistic because it considers the whole cash flow to the equity for determination of firm's value. The dividend discount model takes only the dividend paid into consideration.

2007 Dec 3a:


An investor is holding 1000 shares of Rishabh company. Presently, the rate of dividend being paid by the company is Rs. 2 per share and the share is being sold at Rs. 25 per share.

However, several factors are likely to change during the course of the year as indicated below: Existing Revised Risk free return 12% 10% In view of the above factors whether the investor Market risk premium 6% 4% should buy, hold or sell the shares? And why? Beta value 1.4 1.25 Expected growth rate 5% 9% Solution : Tutorial notes:
1. If the market price is less than the theoretical price, the share should be bought. The theory is that the market price moves towards theoretical price. The share is under priced when it sells at a lower value than the theoretical price, such shares should be bought. 2. How will you calculate the theoretical price? What formula or model you will use to determine the theoretical price? Think.

3. Dividend growth formula is as follows:


Cost of equity = Dividend/Market price + Growth. You have growth rate and dividend in the question itself. You need to know the cost of equity to determine the market price (theoretical). You can compute the market prices for existing and revised conditions both if cost of equity is known. The problem is how to determine the cost of equity? Think.

Use the following steps to solve this question :


1. Use CAPM to determine the cost of capital. Find out existing rate of return and revised rate of return using CAPM. All data is supplied with the question.

2. Check you find existing rate of return as 20.4% and revised rate as 15%.
3. Use dividend growth model to determine the existing and revised market price. Existing price :13.64 and revised : 36.3, Check it.

Existing rate of return = 12% + 1.4 x 6% = 20.4% Revised rate of return = 10% + 1.25 x 4% = 15%. Using Dividend growth model : 20.4%% = 2 x 1.05 / MP + 5% giving MP = Rs. 13.6 Revised market price : 15% = 2 x 1.09 / MP + 9% giving MP = Rs. 36.3. Comments :
1. The current market price is Rs. 25 while its theoretical price comes to be Rs. 13.6. The market price moves towards the theoretical price, hence the price of share will fall from Rs. 25 to Rs. 13.6. Falling shares should be sold. Worded differently, the share is overpriced to the tune of Rs. 11.4 ( 25 - 13.6), and should be sold. One question should also arise here, the share is over priced and should be sold at Rs. 25, who will purchase it, if it is known that its price will fall? The answer is, the share market runs on two opposite views. If one side thinks that the price will go down, the other side thinks that the price will go up. 2. The revised theoretical market price is Rs. 36.3. It can be interpreted that the price will go up from Rs. 25 to Rs. 36.3, hence such shares are advised to be held.

2008 June 3a

(CA Final Nov 07)

XY Ltd. which is specialized in manufacturing garments is planning for expansion to handle a new contract which it expects to obtain. An investment bank has approached the company and asked whether the company had considered venture Capital financing. In 2001, the Co. borrowed Rs.100 lacs on which interest is paid at 10% per annum. The Company shares are unquoted and it has decided to take your advice in regard to calculation of value of the Company that could be used in negotiations using the following available information and forecast. Company's forecast turnover for the year ended 31st March, 2005 is Rs. 2,000 lacs which is mainly dependent on the ability of the Company to obtain the new contract, the chance of which is 60%, turnover for the following year is dependent to some extent on the outcome of the year to 31st March 2005. Following are the estimated turnovers and probabilities Year 2005 Year 2006 Turnover Rs. In lacs Probabilities Turnover Rs. In lacs Probabilities 2,000 0.6 2,500 0.7 3,000 0.3 1,500 0.3 2,000 0.5 1,800 0.5 1,200 0.1 1,500 0.6 1,200 0.4 Operating costs inclusive of depreciation are expected to be 40% and 35% of turnover respectively for the year 31st March 2005 and 2006. Tax is to be paid at 30%. It is assumed that profits after interest and taxes are free cash flows. Growth in earnings is expected to be 405 for the year 2007, 2008 and 2009 which will fall to 105 each that. Industry average cost of equity net of tax is 15% Solution : This is a typical question. You should see the solution carefully. Tutorial notes : 1. The results of 2006 are dependent on the results of 2005. This implies that the probabilities of year 2006 should be joined with those of year 2005. 2. Expected values should be calculated with joint probability for the year 2006. 3. The expected values so found, should be used for other computations and forecasts. 4. The question does not ask any answer directly. While reading you must notice that it asks you to furnish the value of the company for negotiation purposes. The values may be from shareholder's view point. If industry cost of equity is taken, the horizon value can be computed as earnings / cost of equity.

5. You have been deliberately confused in the question. The probability of 60% is towards the sales of Rs. 2000 lacs in year 2005, and not towards receiving the contract as can be understood from the question. This is also evident from the probability figures. 6. This question is a very simple one and you should try to solve it before you see the solution which goes below: Expected values for year 2005 are as follows : Year 2005 Turnover Rs. In lacs Probabilities Expected value Rs. In lacs 2,000 0.6 1,200 1,500 0.3 450 1,200 0.1 120 The expected value of turnover in year 2005 1,770 Expected values for year 2006 are as follows : Year 2006 Turnover Probabilities Joint probabilities Expected values Rs. Lacs 2,500 0.7 (0.6x0.7) 0.42 1,050 3,000 0.3 (0.6x0.3) 0.18 540 2,000 0.5 (0.3x0.5) 0.15 300 1,800 0.5 (0.3x0.5) 0.15 270 1,500 0.6 (0.1x0.6) 0.06 90 1,200 0.4 (0.1x0.4) 0.04 48 The expected value of turnover in year 2006 2,298 These expected values will be used for other forecasts The cash flows for the horizon 2005 to 2010 can now be computed with these expected values. Note that from 2006 the annual growth in earnings is given as Rs. 405 lacs upto the year 2010. Beyond 2010, the annual growth is constant at Rs. 105 lacs. Statement of future cash flows : fig. in Rs. Lacs Details 2005 2006 2007 2008 2009 Expected turnover 1,770 2,296 Operating costs 40% 35% Operating costs 708 804 Interest 10 10 Total costs 718 814 PBT 1,052 1,482 Tax @ 30% 316 445 PAT 736 1,038 Add depreciation Not available Add increase in earnings 405 405 405 Cash flow after tax 736 1,039 1,444 1,849 2,254 Discount factor at 15% 0.87 0.76 0.66 0.57 0.50 Present value of CFAT 640 786 949 1,057 1,121 Computation for the value of the firm : It is assumed that the cash flow after tax beyond 2010 would remain stable at Rs. 2,359 lacs. Horizon value of the firm = CFAT / Cost of equity = 2,359 /0.15 = Rs. 15,727 lacs. Value of the firm from the shareholder's view point : Rs. Lacs Present value of CFAT upto year 2009 4,553 Present value of CFAT beyond 2009 ( 15,727 x 0.432) 6,794 Value of the firm from the shareholder's view point : 11,347 2010

105 2,359

2008 June 4b: Solution :

(also in CA final May 03, similar in Nov. 05) It is an easy one. Try yourself before you see the solution.

Your client is holding the following securities : Particulars Equity shares Company X Company Y Company Z PSU Bonds 8,000 10,000 16,000 34,000 800 800 800 3,400 8,200 10,500 22,000 32,300 0.8 0.7 0.5 1.0 Cost Rs. Dividend Rs. Market price Rs. Beta

Assuming a Risk Free rate of 15%, Calculate : Expected rate of return in each, using Capital Asset Pricing Model. Average return of the portfolio. Solution : Tutorial Notes : 1. To use CAPM, you must have market return as the risk free return and beta are already supplied. Can you think of the way to determine the market return? You can refer problem 9 solved above for assistance. Particulars Equity shares Company X Company Y Company Z PSU Bonds 8,000 10,000 16,000 34,000 68,000 8,200 10,500 22,000 32,300 200 500 6,000 (1700.0) 5,000 800 800 800 3,400 5,800 1,000 1,300 6,800 1,700 10,800 Cost Rs. Market price Rs. Capital gain Dividend Total gain Rs.

Expected return from the market : 10,800 / 68,000 = 15.9%. The expected return computed previously would be used in CAPM for determination of expected return on individual security : Particulars Equity shares Company X Company Y Company Z PSU Bonds 15 + 0.8 x (15.9 15.0) 15 + 0.7 x (15.9 15.0) 15 + 0.5 x (15.9 15.0) 15 + 1.0 x (15.9 15.0) 15.7% 15.6% 15.4% 15.9% 62.6% Average return on portfolio : 62.60 / 4 15.7% Computation

2008 Dec 6b:


Calculate the value of the share of a company, if its beta is 1.5, the previous dividend was Rs. 2 per share, and the growth rate is expected to be 8%. The risk free return is 10% and the market portfolio earns a return of 15%.

Solution : Tutorial Notes 1. Find out cost of equity capital with CAPM.
2. Use dividend growth model to know market price of the share which is also its value. Note that dividend declared is Rs. 2 per share which is expected to grow at 7%.

Required rate of return = 10% + beta x (15% 10%) = 7.5% Using Dividend growth model :

7.5% = 2 x 1.08 / (Market price) 2008 Dec 6c:

+ Growth = 2.16/MP + 8% Giving MP = Rs. 22.7

The risk free return is 6% and the return on market portfolio is 10%. If the required rate of return on stock is 13%, calculate beta.

Solution :Beta is given by : 13% = 6% + beta x ( 10% 6%) giving beta = 1.75.

2009 June 2b:


The capital structure of Hilson Ltd. as on March 31, 2009 is given under: Rs.lacs Equity shares (face value : Rs. 10 per share) 540 Reserves and Surplus 360 8% Preference share ( Rs. 100 per share) 180 10% Debentures (Rs. 100 per debenture) 180 11% Term Loans 540 1,800 All these securities are traded in the market. Recent prices are : Ex-Dividend equity share price : Rs. 15; Ex-Dividend preference share price : Rs. 120; and Ex-interest 10% debentures market value : Rs. 103. The following additional information is available: Company's equity beta : 1.06 ; Yield on Long term treasury bonds : 8%, stock market risk premium : 6% The debentures are redeemable after 3 years and interest is payable annually. The income tax rate applicable to the company is 35%. Required : Using the information in the case, determine the Weighted Average Cost of capital of Hilson Ltd. on market value weights. Extracted from the table of PV: Interest rate 6% 9% 10% 11% PVIFA (3 years) 2.577 2.531 2.487 2.444 PVIF(3 years) 0.794 0.772 0.751 0.731 Solution; Tutorial notes: 1. You have already solved this type of problem earlier. You should find no difficulty in solving it. Think how you are going to solve it. 2. Cost of equity can be easily calculated with CAPM. Find it as 14.36%. 3. Cost of preference shares is dividend/ market price. Calculate it. 4. Computation of cost of debentures is time consuming. Calculate it keeping time factor in mind.

5. As the market price of the share is given, you are to use market value method for computation of cost of equity shares. Reserves will not be taken as component of capital structure.
Cost of equity capital = 8% + beta x 6% = 8% + 1.06 x 6% = 14.3%. Cost of preference share = Dividend / Market value = 8 / 120 = 6.7% Cost of Debs. = Pre-tax cost of 10% deb. = 10 x (1 / 1 + k) + 10 x (1 / 1 + k)2 + 110 x (1 / 1 + k)3 The cost of debentures is given as Rs. 103. K can be found out by trial and errors. It comes as 9%. The pre-tax cost of term loan = 11% as given. Weighted average cost of capital will be determined with respect to market values of the various components. Market values of the component sources of capital : Face Market Rs. Lacs Cost Interest value value % Equity shares (face value : Rs.10) 540 (540/10) x 15 810 14.3% 115.8 Reserves and Surplus (not considered if market value of share is taken) 8% Pref. share ( Rs. 100) 180 (180/100) x 120 216 6.7% 14.5 10% Debentures (Rs. 100) 180 (180/100) x 103 185 9.0% 16.7 11% Term Loans 540 540 540 11.0% 59.4 1,440 1,751 206.4

After tax WACC (179.7 / 1751)

10.3%

2009 Dec 4 The key information pertaining to the proposed new financing plans of Adventure Ltd. is given below: Sources of funds Financial Plan I Financial Plan II Equity 15,000 shares of Rs. 100 each 30,000 shares of Rs. 100 each Preference shares 12% 25,000 shares of Rs. 100 each Debentures Rs. 500,000 at a coupon rate of 10% Rs. 1500,000 coupon rate 11%. The corporate tax rate is 35%. Required : (a) Determine the two EBIT EPS coordinates for each financial plan; (b) Determine the (i) Indifference point (ii) Financial break even point for each financing plan (c) Which plan has more financial risk and why? (d) Indicate over what EBIT range, if any, one plan is better than the other. (e) If the firm is fairly certain that its EBIT will be Rs. 1250,000, which plan would you recommend and why? Solution: Tutorial notes:
1. Financial break even point is the level of EBIT which is just able to pay the total financial charges i.e. interests and preference dividend. At financial break even point the eps is zero. It is easy to calculate the fbep if capital structure does not contain preference shares. It such case fbep is equal to fixed interest charges. However, if preference shares appear in the capital structure, in addition to fixed interest charges, pref. dividend is also to be paid. Pretax pref. dividend should be added to fixed interest charges to arrive at fbep.

1,250 1,250 50 165 1,200 1,085 420 380 780 705 300 0 480 705 15 30 32 24 Indifference point is the point where the eps remains same irrespective of capital structure. At this level of EBIT, the rate of return on capital employed is equal to the cost of debt and this also known as break even level of EBIT for alternative financial plans.

What is financial risk? How it is measured? Particulars EBIT Less Interest on debentures EBT Less : Taxes @ 35% EAT Less : Preference dividend Earning for shareholders Number of equity shares in '000 EPS

Plan 1

Plan 2

Particulars Plan 1 Plan 2 EBIT E E Less Interest on debentures 50 165 EBT E 50 E 165 Less : Taxes @ 35% 0.35 (E 50) 0.35 ( E 165) EAT 0.65 (E 50) 0.65 (E 165) Less : Preference dividend 300 0 Earning for shareholders 0.65 ( E 50) 300 0.65 (E 165) Number of equity shares in '000 15 30 EPS [ 0.65 ( E 50) 300] / 15 [0.65 (E 165)] / 30 At financial indifference point the eps is same for both the plans. Hence we have: [ 0.65 ( E 50) 300] / 15 = [0.65 ( E 165)] 30 Which on solution gives E = 858

At E equal to Rs. 858,000 the eps is same under both the plans. Financial break even point is the point when ebit is just equal to financial changes. Fixed interest charges : Plan 1 Debenture interest 50 Preference dividend (Pref.div / (1 tax rate) (300/0.65) 462 Financial break even point 512

Plan 2 165 0 165

Financial risk is measured by financial leverage. It is measured by EBIT / EBT. It is also given by % change in EPS / % change in EBIT.

Plan 1 Plan 2 1,250 1,250 EBIT 50 165 Interest 1,200 1,035 EBT 1.04 1.21 Financial leverage (EBIT/EBT) More the fixed interest charges, more is the financial risk. In that criteria, financial risk is more in plan 2.
EPS is same in both the plans at EBIT of Rs. 858,000. Beyond this plan 1 will give more eps, hence should be considered better.

(e) As shown in (a) above, plan 1 gives more eps for ebit of Rs. 1250000. 2010 June 4b:
Atul Company Ltd. earned an EBIT of Rs. 3 crores for the year just ended. The company has 10 lakhs equity shares outstanding with a face value of Rs. 10 each and an outstanding debt of Rs. 2 crores at 12%. The company also has free reserves to the extent of Rs. 5 crores, which can be capitalised. The company is planning a bonus issue to utilize the entire free reserves and later a stock split to make the face value of the shares Rs. 2. After the bonus issue and stock split are complete, the company plans to raise funds to the extent of Rs. 5 crore to finance additional investments. The required funds may be raised either as debt at an interest of 14% or equity, which can be issued at the par value of Rs.2. The company expects the standard deviation of total EBIT after the investments are made is Rs. 1.8 crore. Assume the income tax rate to the company is 30%. You are required to calculate the minimum level of EBIT at which the debt issue is better than the equity from the point of view of EPS by at least 95%,.

Solution :

Tutorial notes:

1. You must revise the reading of normal distribution curve. You must also know how to compute the value of parameter, if standard deviation is given and also the degree is also given, here the degree is 95%.

Total no. of shares of Rs. 10 No. of shares if converted to Rs. 2 10 x 5 No. of bonus shares (500 / 2) Total no. of shares after bonus and alteration Comparison of two plans:

10 50 250 300

lakhs lakhs lakhs lakhs

Plan 1: Raising through equity issue of Rs. 5 cores of Rs. 2. In all 250 lakhs shares will be issued making total number of outstanding shares as 550 lakhs.

Plan 2: Raising through debt of Rs. 5 crores at 14%. Plan 1 EBIT E Interest on existing debt 24 Interest on additional debt 0 EBT (E 24) Less taxes at 30% 0.30 (E 24)

Plan 2 E 24 70 (E 94) 0.30 (E 94)

EAT 0.70 (E 24) 0.70 (E 94) No. of shares in lakhs 550 550 EPS 0.70 (E 24) / 550 0.70 (E 94) / 300 For financial indifference point, both plans should give same eps. 0.70 (E 24) / 550 = 0.70 (E 94) / 300 which gives E = Rs. 178 lakhs.
Part 2: This is statistically solved with the help of normal distribution curve. You should revise this knowledge. The required EBIT (expected) is 2.97 + 1.78 = Rs. 4.75 crore.

pective shareholders.

CAPM Capital Asset

k.

dividing total interest

ny.

timates of the cost of Mix : 60 7.0 20.0

60 7.0 20.0 12.20

capital (after tax) are 60 11 24 y by calculating the

gnoring the floatation

od for computation of e.

e 16.36% orally? Use

11% because market

so given, the rate of %. What will it be and

time factor should be

n general is 5%. The

nt of capital structure.

and how much is it?

10% + 1.28 x 5% =

Cost 24.60 9.00 25.31 58.91

uity would be 10% + Cost 23.85 9.00 25.31 58.16

draft facility has been s. 80 lacs. The bank ollowing information is

Rs. 100 each

he shares while the olute control over the

d by the bank through

e. Read the question

tal number of shares.

at market price of Rs. ectors do not want to

ings may be used for

ed at 11.30. Last year at 16% coupon for a wants to convert their tes. Market quotation of India security i.e.T

hereafter, what is the

ess than one year. A the expiry of say 364

11.30 means 11.30%. point (means 0.20%) .

nt rate of 10.85. Thus nge from fixed rate to

10 11.45 16.00 11.60 15.85

are provided with the

end to the tune of 6% up to 8% per annum for each share of YZ YZ Ltd. in lieu of the

share price as well as

t before you see any

0 lakhs (Rs. 60 x 12

of merger represents s shares. Costs here excess of the current

taken into account for The current rate of

d 8%. The 6% growth rate of 8% would be

st will remain Rs. 120

he after merger share

urself before you see

upee each. Presently, These earnings are

benture is Rs. 105.50 te is 30% and Shibaji

ry of capital structure

nformation in relation

wth formula. What will

roceed any further.

ice per share.

an you think of some ital? or earning price ratio

et price per share. What is market price mething different?

et price per share.

termine it before you

million and a cost of quity and to replace it

eseeable future.. The

te how this change in ost of capital of Akbar

e assumes that there

it with the use of debt xpenses. Use of debt es are less than what

ax rate x debt. debt.

e you see below:

the cost of equity will risk. for equity and Rs. 5

s known. The cost of

ares of a stock for a ald equity is Rs. 16. r.

oretical value? If not,

shares at a particular

8, in this case no will are at Rs. 48 against

e value of warrant is

ce is say Rs. 20. The

. The present market her than buying them or in other words the

Rs. 1.8 per share i.e.

als; (i) expansion of ess. timates of share beta project, if undertaken, nternal accruals.

pany has identified a 1.5. The comparable e of XYZ company is

ff rate to evaluate the capital to be used as and (ii) above, would d new projects? Give

ove question :

i.e. market premium)

fter the project being

bination. The amount e the WACC.

these parameters are ost of debt is already Your first answer is

has forecast its total

l Requirement

nd 15% respectively.

s firm has debt in its Do you know how to beta for levered firm.

s given as 12%. You

ollar = 4500 mRs.)

30) 10% = 7%

ken as debt (1 tax

he next 2 years. The at the growth rate is 0 per share and the he Pioneer Ltd as of

king note below)

he value of share can

o value after four years. .08 = Rs. 30.75

y 110 per share. The

er) were estimated as

million USD worth of

following portfolio for

ch scrip should be at ly 11400 to move to

w on the market ?

all and stabilize at a

4 0.613 0.592 0.572 stable growth period.

g the following model:

rate. If these cash flows te growth rate). A firm owing model:

xpected free cash flows

gn after that the value

rate)] / ( 1 + WACC)

Terminal 10% 6,843 1,140 342 798 1,711 156 0 643

569.2 12,684.6 13,253.8 the shareholders. The

likely to be 15% above sent is 10 lakhs and the market capitalization of

e buy back price).

. of shares it can buy is

are after buyback would

k is Rs. 35 lakhs, no. of

total number of shares. outstanding shares are S. Do it yourself before

51 = Rs. 4.70.

een mentioned in the

calculated yaws. This is

e to be added to the

176.9

3.20 in 2007 and paid . 350 lakhs in 2007 and

capital expenditure to negligible.

ancing mix of debt and are expected to grow at remium is 5.5%.

odel (Free cash flow to

use as a bench mark to

can calculate with this

ice) + Growth

ange in Working capital

lue. Note that dividend

ution with the one given

x 5.5% = 12%.

cumulated in the firm as irm's value.

flow to the equity for o consideration.

end being paid by the

icated below:

hether the investor hares? And why?

heory is that the market alue than the theoretical

etermine the theoretical

the question itself. You mpute the market prices o determine the cost of

sed rate of return using

isting price :13.64 and

he market price moves alling shares should be hould be sold. 5, who will purchase it, if price will go down, the

go up from Rs. 25 to Rs.

le a new contract which ether the company had erest is paid at 10% per regard to calculation of mation and forecast.

is mainly dependent on r for the following year

Probabilities

pectively for the year st and taxes are free ich will fall to 105 each

ties of year 2006 should

asks you to furnish the ew point. If industry cost

s the sales of Rs. 2000 e question. This is also

ion which goes below:

lues. Note that from 010, the annual growth

4,553

premium : 6%

n Ltd. on market value

ng it. Think how you

od for computation of e.

arious components. After tax 115.8 14.5 10.8 38.6 179.7

Ltd. is given below:

s. 100 each

on rate 11%.

you recommend and why?

al charges i.e. interests

ch case fbep is equal to

harges, pref. dividend is fbep.

e. At this level of EBIT, reak even level of EBIT

5 ( E 165) 5 (E 165)

5 (E 165)

165)] / 30

o given by % change in

risk is more in plan 2.

e eps, hence should be

any has 10 lakhs equity es at 12%. alised. The company is e the face value of the

he extent of Rs. 5 crore nterest of 14% or equity,

made is Rs. 1.8 crore.

than the equity from the

o compute the value of e is 95%.

be issued making total

ise this knowledge. The

Chapter 3 : Capital Budgeting


You must know how to compute discount factor with different interest rates for different years. Computation of Initial Investment : Cost of new machine xxx Add: Installation xxx Total cost to be depreciated over life of machine xxx Less: Proceeds from sale of old machine xxx Investment tax credit xxx Adjust for Taxes on sale of old machine xxx Changes in net working capital xxx Initial Investment xxx Computation of Operating cash flows : Revenues xxx Less: Expenses excluding depreciation xxx Profit before depreciation and taxes xxx Less: Depreciation xxx Profit before tax xxx Less: tax xxx Profit after tax xxx Add : Depreciation xxx Operating Cash flows xxx Three types of projects are considered in capital budgeting: Independent Project is one which is independent of other projects. The acceptance or rejection of it does not influence the consideration of other projects. Mutually exclusive projects are the ones in which acceptance of one will lead to rejection of the other. Either this or that alternative is to be selected. Contingent project is the one the acceptance of which is dependent on acceptance of other project/projects. For example, the installation of a new machine would be required if a new project is to be taken up. All projects which are dependent on one another should be considered as one project for the purpose of evaluation. Taxation impact on capital projects: Following points should be kept in view: The sale of asset at its book value will not attract any tax. If the asset is sold for value more than its initial price: Suppose an asset was purchased for Rs. 200,000 having a book value of Rs. 140,000 and is sold at Rs. 210,000. In this situation there is a capital gain of Rs. 10,000 for which rates of capital gain will be applied. The excess of sale proceeds over the book value less capital gain will be treated as short term capital gain or ordinary income. In the present case, the excess sale proceeds of Rs. 70,000 will be divided into two parts for taxation purposes. The capital gain will Rs.10,000 (210,000 -- 200,000) and ordinary income would be Rs. 60,000 (200,000 -- 140,000).

If the asset is sold for value less than its initial price but more than its book value, the excess over book value will be treated as short term capital gain or ordinary income and the ordinary rates of income tax shall apply. For example; if the above mentioned asset is sold for Rs. 160,000, the excess of Rs. 20,000 will be treated and taxed as ordinary income. In other words the recaptured depreciation is taxed as ordinary income. If the asset is sold at a value less than its book value, there will be loss which will be used to set off ordinary operating income i.e. this loss will save taxes for the firm. It is assumed here that the firm is otherwise earning sufficient income to attract taxes. Working capital Requirements :

Sometimes acceptance of project will call for additional arrangement for working capital which should be treated as cash outflow at the time of requirement. At the end of the project, the additional working capital would be released which should be treated as cash inflow at the end of the project along with scrap value of project if any.

2002 June [6](b) A product is currently manufactured in a machine that is not fully depreciated for tax purpose and has a book value of Rs. 60,000 ( it was purchased for Rs. 1,20,000 six years ago). The product cost is as under : Direct cost : Rs. 24. Indirect labour : Rs. 8. Variable overheads : Rs. 16. Fixed overheads : Rs. 16. 10,000 units of the product are normally produced. It is expected that the old machine can be used indefinitely into the future, after suitable repairing estimated to cost Rs. 40,000 annually is carried out. There is an offer for a new machine with latest improved technology at Rs. 300,000 after trading off the old machinery for Rs. 30,000. The projected cost of the product will then be as under : Direct cost : Rs. 14. Indirect labour : Rs. 12. Variable overheads : Rs. 12. Fixed overheads : Rs.20. The fixed overheads are allocations from other departments plus the Depreciation of plant and machinery. The old machine can be sold in the open market for Rs. 40,000. The new machine will last for 10 years at the end of which it will have a salvage value of Rs. 20,000. Assume the rate of corporate tax at 50%. For the tax purpose the cost of new machine and that of the old one may be depreciated in 10 years. The minimum rate of return expected is 10%. It is also expected that the future demand of the product will remain steady at 10,000 units. Advise whether the new machine should be purchased. Ignore capital gains. Present value of Rs. 1 at 10% for 10 years are : year 1 2 3 4 5 6 7 8 9 10 0.909 0.826 0.751 0.683 0.621 0.564 0.513 0.467 0.424 0.386 Solution : Tutorial notes: 1. What is the depreciation of new machine per year ? What is the cost to be depreciated ? 2. Fixed overheads other than depreciation are not relevant for decision making. 3. The special item in this question is absence of selling price and sales revenue anywhere. If sales revenue is not there what would be cash inflow which is to be discounted ? 4. Cost of the machine is not Rs. 300,000. The cost is Rs. 300,000 + Rs. 30,000. The salvage value is Rs. 20,000. Thus the cost to be depreciated in 10 years is Rs. 330,000 -- Rs. 20,000 i.e. Rs. 310,000. The depreciation per year would be Rs. 31,000. Solution : Evaluation of existing and new machine on basis of cost and cash outflow Particulars Existing New No. of units 10,000 10,000 Rs. Rs. Variable cost per unit Rs. 48 38 Total variable cost 480,000 380,000 Add: annual repairs 40,000 Add: depreciation 6,000 31,000 Total cost 526,000 411,000 Tax saving @ 50% on cost 263,000 205,500 Less Depreciation (non cash item) 6,000 31,000 Cash outflow 257,000 174,500 Net saving in cash outflow 82,500

Thus new machine would result in net saving in cash outflow of Rs. 82,500 per year for 10 years. Rs. Present value of saving of Rs. 82,500 per year for 10 years*= 82,500 x 6.144 506,880 * 82,500 ( 0.909 +0.826 +0.751 +0.683 +0.564 +0.513 +0.467 +0.424 +0.386) Present value of cash inflow of Rs. 20,000 in tenth year (20,000 x 0.386) 7,720 Present value of cash outflow on purchase of new machine 300,000 Net present value ( 506,880 + 7,720 -- 300,000) 214,600 Net present value being positive, the new machine is recommended to be purchased. Note : You are advised to use discounting figures upto two figures after decimal.
Similar problem for practice (ca final nov.96)

Nine Gems Ltd. has just installed Machine - R at a cost of Rs. 2,00,000. The machine has a life of five years with no residual value. The annual volume of production is estimated at 1,50,000 units, which can be sold at Rs. 6/unit. Annual operating costs are estimated at Rs. 2,00,000 (excluding depreciation) at this output level. Fixed Costs are estimated at Rs.3 per unit for the same level of production. Nine Gems has just come across another model called machine-S capable of giving the same output at an annual operating cost of Rs. 1,80,000 (exclusive of depreciation). There will be no change in fixed cost. Capital cost of this machine is Rs. 2,50,000 and the estimated life is for five years with nil residual value. The company has an offer for sale of Machine--R at Rs. 1,00,000. But the cost of dismantling and removal will amount to Rs. 30,000. As the company has not yet commenced operations, it wants to sell machine-R and purchase machine-- S.
Nine gems will be zero-tax company for seven years in view of the several incentives and allowances available. The cost of capital may be assumed at 14%. PV factors for five years are as follows: Year 1 2 3 4 5 PV factors 0.877 0.769 0.675 0.592 0.519 (i) Advise whether the company should opt for the replacement. (ii) Will there be any change in your view, if Machine-R has not been installed but the company is in the process of Support your view with necessary workings. Solution : Tutorial Notes : 1. The company will be a zero-tax company. Depreciation and the tax saving due to depreciation is of no relevance. 2. The purchase of machine R is a sunk cost therefore not relevant for decision making.

3. In option 2, when selection between the two machines is to be made, cost of the machines will naturally be relevant.
Option 1: Machine R is installed and considered for replacement with Machine S: Incremental Cash inflow due to replacement: The operating cost of machines R and S Machine S Machine R Incremental Cash inflow Present Value of Cash inflow =20,000 (0.877+0.769+0.675+0.592+0.519) Present Value of Cash outflow = (250,000 -- 100,000 -- 30,000) Difference Rs. 270,000 250,000 20,000 Rs.

68,640 180,000 111,360

Cash outflow will be more by Rs. 111,360 due to replacement. It is advised to continue with the existing machine--R. Replacement is not advised.
Option 2: Selection Between Machine R and Machine S: Production in units Sales value @ Rs. 6/unit Operating expenses (excluding depreciation) Contribution Less : Fixed cost Annual Cash inflow for five years Machine R 150,000 900,000 200,000 700,000 450,000 250,000 in Rs. Machine S 150,000 900,000 180,000 720,000 450,000 270,000

Present value factor ((0.877+0.769+0.675+0.592+0.519) Present value of cash inflows Present value of cash outflow due to cost of machine Net present value

3.432 858,000 200,000 658,000 Difference 18,640 Net present value of Machine S is more by Rs. 18,640. The choice should fall on machine S. Alternatively :

3.432 926,640 250,000 676,640

Since Production, selling price, Sales revenue and fixed cost are same in both the machines, these are not relevant of selection process. Only the difference in operational expenses and cost of machines are relevant. Cash inflow is in the form of saving in operation expenses Rs.
Present value of Incremental cash inflow of Rs. 20,000 p.a. due to machine S: (Present value factor 3.432 x Rs. 20,000) Present value of Incremental cash outflow due to machine S: (250,000 -- 200,000) Net present value Machine S is recommended to be had in view of positive net present value over machine R. 68,640 50,000 18,640

2002 Dec [14] (a) : A company is faced with the problem of choosing between two mutually exclusive projects. Project A requires a cash outlay of Rs. 100,000 and cash running expenses of Rs. 35,000 per year. Project B requires Rs. 150,000 as cost and Rs. 20,000 as running expenses. Both the projects have equal life of 8 years. The salvage value of A is Rs. 4,000 and of B is Rs. 14,000. The company's tax rate is 50% and it has a 10% required rate of return. Assuming depreciation on straight line basis, ascertain which project should be selected. Solution : It is very easy one. Try to solve it on your own before you see below. Project A Project B Cash expenses 35,000 20,000 Add : Non cash depreciation 12,000 17,000 Total 47,000 37,000 Tax saving at 50% 23,500 18,500 Net Cash outflow due to operating expenses 11,500 1,500 PV of net cash outflows due to operating expense (factor 5.335) 61,353 8,003 Add : Pv of cash outflows due to cost 100,000 150,000 Total Cash outflow 161,353 158,003 Less cash inflow present value of salvage at the end of life 1,868 6,538 NPV of outflows 159,485 151,465 Net savings in outflows in project B 8,020 Present value factor for salvage is 0.467. Salvage of A : 4,000 x 0.4678 and for B : 14,000 x 0.467 Alternatively : It can also be solved using incremental analysis approach as follows: A B Incremental Cash running expenses for 8 years 35,000 20,000 15,000 Depreciation for 8 years 12,000 17,000 (5,000) Total expenses eligible for tax saving 47,000 37,000 10,000 Tax saving at 50% 5,000 Saving after tax 5,000 Add : Depreciation difference of Rs. 5,000 5,000 Net saving per annum 10,000 Pv of net saving for 8 years (factor 5.335) 53,350 Difference in salvage value Rs. 10,000 Pv of difference in salvage value (factor 0.467) 4,670 Total savings in project B 58,020 Excess cash outflow in project B (150,000 100,000) 50,000

Net saving in project B June 2004 (7) (repeat dec.06)

8,020

Consider the following details of a proposed capital investment: (a) Cost of the project is estimated to be Rs. 435 crores which includes : (i) Contingencies of Rs. 30 crores ; (ii) Margin money for working capital of Rs. 10.5 crores (iii) Interest during construction of Rs. 31 crores (iv) Capital issue expenses of Rs. 13.5 crores (b) Incremental investment on the working capital is estimated to be (Rs. Crores) Year 0 1 2 3 Inc.WC 10.5 32.3 7.0 5.6 (c) Salvage value has been estimated to be Rs. 80 crores. (d) The operation cash flows are projected as follows (Rs. Crores) Year PBIDT Taxation Interest 1 42 8 25 2 111 6 46 3 125 9 36 4 125 14 37 5 125 33 16 6 125 39 11 7 125 44 5 8 125 47 1 9 125 48 0 10 125 48 0

(e) The project will be financed with a debt of Rs. 268 crores and the remaining through equity capital. The overall cost of financing the project would be 13%.
Calculate the Net present value of the cash flows. Is this project financially viable ? Solution : Tutorial notes :

1. The interest during construction period and capital issue expenses do not form the cost of the project. These should be excluded from the cost of the project. 2. The question is silent about depreciation and tax rate. The taxation is given and PBIDT is also given. Thus cash inflows due to the project is directly given. The interest figures are superfluous data and are not relevant because the tax and the cash inflows can be computed.
Statement showing Net present value of cash inflows and outflows Year PBIDT Taxation Inflow Outflow Net inflow 1 42 8 34 32.3 1.7 2 111 6 105 7.0 98.0 3 125 9 116 5.6 110.4 4 125 14 111 111.0 5 125 33 92 92.0 6 125 39 86 86.0 7 125 44 81 81.0 8 125 47 78 78.0 9 125 48 77 77.0 10 125 48 77 77.0 10 Salvage 80 80.0 Net present value of cash inflows Net present value of initial cash outflows (380.0 +10.5) Differential net present value As the net present value is positive, the project is viable. Df at13% 0.88 0.78 0.69 0.61 0.54 0.48 0.43 0.38 0.33 0.29 0.29 Rs. Crores Pv 1.50 76.75 76.51 68.08 49.93 41.31 34.43 29.34 25.63 22.68 23.20 449.37 390.50 58.87

2005 June 5b: HPL Ltd. is a growing company. Its free cash flows for equity holders have been growing at a rate of 25% in recent years. This abnormal growth rate is expected to continue for another 5 years and then these FCFE are likely to grow at the normal rate of 8%. The required rate of return on these shares is 15%, the firms weighted average cost of capital is 12%. The amount of FCFE per share at the beginning of current year is Rs. 30.

Determine the maximum price an investor should be willing to pay now, and on free cash flow approach. The issue price of share is Rs. 500. Factors at 15% discount rate are: Year 1 2 3 4 5 Factor 0.87 0.756 0.658 0.572 0.497 Solution : Tutorial notes 1. FCFE per share is Rs. 30 now which will grow at a rate of 25% for next five years after that it will continue at a growth rate of 8%. 2. You should first find the value of cash flows for next five years at 25% growth. 3. The cost of capital is 12% and required rate of return is 15%. What discounting rate 12% or 15% will you use and why? 4. You are required to determine the price that the investor should be willing to pay now. Presently the investor would be ready to pay the present value of all the cash flows associated with the share. 5. The first cash flow is Rs. 30 + 25% growth = Rs. 37.5 which should be discounted with required rate of return of 15% and not with the cost of capital of 12%. This cash flow will grow at 25% for next four years. Present values of cash flows for 1 5 years (discount rate 15%) Year FCFE per share Rs. Discount factor PV 1 30.0 x 1.25 37.5 0.87 32.6 2 37.5 x 1.25 46.9 0.75 35.2 3 46.9 x 1.25 58.6 0.66 38.7 4 58.6 x 1.25 73.2 0.57 41.7 5 73.2 x 1.25 91.6 0.50 45.8 Total pv of fcfe for next five years 194.0 From fifth year onwards the fcfe becomes steady at a growth rate of 8%. The perpetual cash flows will be evaluated at capitalization principle. The cash flow at the end of five years is Rs. 91.6 which should be capitalised. What will be rate of capitalization ? Value of share at the end of five year 91.6 x 1.08) / (15% 8%) = Rs. 1,412 This is the value at the end of five year. It should be discounted to determine present value today. Present value of Rs. 1,412 at a discounting rate of 15% = 1,412 x 0.50 = Rs. 706. The investor should be willing to pay Rs. 194 + Rs. 712 = Rs. 906 today. 2005 Dec [3] Excel Industries Ltd. designs and manufactures toys. Past experience indicates that the product life of a toy is 3 years. Promotional advertisement produces an increase in sales in early years, but there is a substantial sales decline in the final year of toy's life. Consumer demand for new toys placed on the market trends to fall into three classes. About 30% of the new toys sell well above expectation, 60% sell as anticipated, and 10% have poor consumer acceptance. A new toy has been developed. The following demand is anticipated: Demand for new toy Estimated sales in Rs. lakhs 1 2 3 Above average 0.3 12 25 6 Average 0.6 7 17 4 Below average 0.1 2 9 1.5 Variable costs are estimated at 30% of the selling price. Special machinery must be purchased at a cost of Rs. 860,000 which will be installed in an unused portion of the factory. The company has been unsuccessfully trying for several years to rent out the vacant portion at Rs. 50,000 per year. Fixed expenses (excluding depreciation) are estimated at Rs. 50,000 per year. Probability

The new machinery will be depreciated by the written down method at the rate of 25% with an estimated value of Rs. 110,000 at the end of third year. Assume this is the only asset in the block. Advertisement expenses will be incurred uniformly, and will total Rs. 100,000 in the first year, Rs.150,000 in the second year and Rs.50,000 in the third year. The company is subject to a corporate tax of 35%. Its cost of capital is 10%. It is assumed that no depreciation will be charged in the terminal year. Requirements: 1. Prepare a schedule computing the probable sales of this new toy each of the three years. Also determine Npv of the proposal. 2. Assuming that cash flows occur uniformly throughout each year, determine Npv of the proposal. 3. Give your recommendation in both the situations. Note : PV at 10% : Re.1 received at the end of year Re.1 received uniformly during the year. Year 1 0.909 0.952 Year 2 0.826 0.864 Year 3 0.751 0.783 Solution : Tutorial notes 1. This is a lengthy question, You must keep time factor in mind. 2. When probability and sales are given, you should determine the expected sales for each of the three years. This is your working note no. 1. 3. Find depreciation of the machine for the three years by wdv method. What will be depreciation in the last year? The salvage value is not considered in wdv method. 4. Prepare the cost statement. You can now find the cash flows. 5. Two factors are given for present values. Take both the factors one by one and find the npv of each one. 6. There is no opportunity cost for using the space. The figure of notional or expected rent of Rs. 50,000 is not relevant. Whenever salvage value is given you should remember to compute the capital gain or loss (short term) at the end of life of asset. 6. This is an easy one. You should solve it on your own before you see below. Watch the time factor. Demand for new toy ProbabilityEstimated sales in Rs. lakhs 1 2 3 Above average 0.3 12 25 6 Average 0.6 7 17 4 Below average 0.1 2 9 1.5 Computation of capital gain or loss at the end of life : Cost Less: Depreciation for first year at 25% Less : Depreciation for second year Less: Depreciation for third year Salvage at the end of life Capital loss Tax saving (35%) due to capital loss Statement of cash flows from the project Sales Less Variable cost at 30% Expected sales (estimated x proba.) 1 2 3 3.6 7.5 1.8 4.2 10.2 2.4 0.2 0.9 0.15 8.0 18.6 4.4

8.60 2.15 6.45 1.61 4.84 0.00 4.84 1.10 3.74 1.31 1 8.00 2.40 2 18.60 5.58 3 4.40 1.32

Contribution (sales less variable cost) 5.60 13.02 3.08 Less Depreciation (nil for third year) 2.15 1.61 0.00 Fixed cost (cash) 0.50 0.50 0.50 Advertisement 1.00 1.50 0.50 Total Fixed cost 3.65 3.61 1.00 EBIT (Contribution less fixed cost) 1.95 9.41 2.08 Less : Taxes at 35% 0.68 3.29 0.73 EAT 1.27 6.12 1.35 Cash flow after tax (EAT + Depreciation) 3.42 7.73 1.35 Add : Salvage value 1.10 Add : Tax saving due to capital loss 1.31 Total cash flows 3.42 7.73 3.76 PV factor at 10% 0.91 0.83 0.75 Pv of cash flows 3.11 6.38 2.83 12.31 Less : Initial outlay 8.60 Npv 3.71 Npv when the cash flows occur uniformly : Total cash flows 3.42 7.73 3.76 PV factor at 10% 0.95 0.86 0.78 Pv of cash flows 3.25 6.65 2.93 12.83 Less : Initial outlay 8.60 Npv 4.23 Recommendation : Npv being positive, the proposal should be accepted in both the situations. 2005 Dec 7b: A company is considering to set up a cogeneration power plant. Cost of generation is estimated as given below: Year 1 2 3 4 5 6 7 8 9 Rs../kwh 3.87 3.99 4.11 4.23 4.36 4.49 4.62 4.76 4.90 What is the levelized cost of generation over the 9 years? Assume cost of capital of 11% for your calculation. The present value factors Year 1 2 3 4 5 6 7 8 9 Pv factor 0.9 0.8116 0.7312 0.6587 0.5934 0.5346 0.4817 0.4339 0.3909 The annuity factor for 9 years at 11% is 5.537. Solution : Tutorial notes; 1. Use only two places after decimal. 2. This is very easy one. You should try on your own before you see the solution. 3. Levelized cost means same cost over 9 years having the same pv as the pv of given cash flows Year 1 2 3 4 5 6 7 8 9 Rs../kwh 3.87 3.99 4.11 4.23 4.36 4.49 4.62 4.76 4.90 Pv factor 0.90 0.81 0.73 0.66 0.59 0.53 0.48 0.43 0.39 Pv 3.48 3.24 3.01 2.79 2.59 2.40 2.23 2.07 1.92 Total Pv = 23.71 Total annuity factor for 9 years = 5.537 Equivalent cash flows : 23.71 / 5.537 = Rs. 4.28

2006 June 7
The Loving Candy Company has been studying an investment project calling for the manufacture and introduction of a new candy bar called Yuppie Nought, targeted (you guessed it) for the yuppie market. As a consequence, Loving Candy expects to use the finest foreign chocolate and to price the candy very high relative to its cost; otherwise no self-respecting yuppie would even think of buying it. Part of the expense will consist of a vast marketing program, complete with endorsements by yuppie heroes.

The project is expected to last eight years, after which time yuppies will be more interested in dentures than candy bars. The introduction of the candy bar requires 400 new machines costing $ 10,000 each. Installing each machine costs $ 100. The machines will be depreciated on a straight-line basis over five years. The production facility will be located at a site the company already owns. The company could rent the space that the candy facility will occupy for $ 500,000 per year.

Loving Candy expects to sell 2 million bars per year for the entire life of the project. The price will be $ 2.50 per candy bar, with a production cost of $ 0.50. The plan schedules the marketing expense per candy bar at $ 1.00. Outlets for the candy bar have been chosen with yuppies in mind. The firm expects to maintain an average inventory of about 500,000 bars, and expects no other increase in working capital. The appropriate after-tax discount rate for Yuppie Nougat is 18%. Calculate NPV of the projected cash flows. You may consider the following while doing your calculations: (a) For tax purposes, depreciate the machine, including installation cost, by straight line method. (b) Estimate working capital in terms of production costs, and consider the amount as investment in the zero year. (c) Assume salvage value equal to recovery of working capital. (d) Assume tax rate of 34%. Should Loving Candy help sweeten the world with Yuppie Nougat ? Note :P.V. Factors at 18% discount rate for 0--8 years are : 1 0.8475 0.7182 0.6086 0.5158 0.4371 0.3704 0.3139 0.266 Solution : Tutorial Notes : 1. It is a full marks question and deserves second reading before beginning the solution. First para is not relevant and therefore deserves to be left out of second reading. 2. The machines are to be depreciated in five years while the life of the project is 8 years. The cash flows have to be computed in two time segments; one from first to five years and second from 6 to 8 years. 3. It is seen that students generally forget to take notice of salvage value of the project. Salvage value in this case is equal to recovery of working capital, this is cash inflow at the end of eighth year. 4. Working capital is to be treated as investment in zero year. Note that working capital is not to be depreciated. Only capital investment is to be depreciated. 5. There is opportunity cost involved in the project as well. How you are going to accommodate it ? 6. Please attempt to solve it before you see the solution. You are advised to use figures in lacs to facilitate the computations. The solution goes as follows: Capital Investment : Cost of 400 machines @ $ 10,000/machine Installation cost @ $ 100/machine Working capital : Production cost of 500,000 bars @ $ 0.50/bar Total investment at year zero Computation of Cash flows of the project : Loving Candy Company Particulars Year 1 to 5 Year 6 to 8 Sales 2 million bars @ $ 2.50/bar 50 50 Less: Production cost @ $ 0.50/bar 10 10 Expenses @ $ 1.0/bar 20 20 Opportunity cost 5 5 Depreciation (40.4/5) 8.08 0 Earnings before tax 6.92 15 Tax @ 34% 2.35 5.10 Earnings after tax 4.57 9.90 Add : Depreciation 8.08 0 $ in lacs. 40.0 0.4 2.5 42.9

Net Cash inflow 12.65 9.90 Computation of Net present value in $ in lacs. Year 1 2 3 4 5 6 7 8 Inflow 12.65 12.65 12.65 12.65 12.65 9.9 9.9 9.9 Pv factor 0.8475 0.7182 0.6086 0.5158 0.4371 0.3704 0.3139 0.266 Pv 10.72 9.08 7.70 6.52 5.53 3.67 3.11 2.63 Add recovery of working capital in eighth year (2.50 x 0.266) Total present value of cash inflow Less Present value of Investment Net present value of the project: Decision : Loving Candy should help sweeten the world with Yuppie Nougat. The net present value of the project comes to be positive.

48.96 0.665 49.62 42.90 6.72

2006 Dec 4a
GFC Ltd. is considering investing in a new equipment costing $ 30 lakh. The equipment is likely to provide a cash flow after taxes of $ 10 lakh per year for 6 years. The unrelated cost of equity capital of the company is 16%. The company intends to finance the project with 60% debt, which will bear interest rate of 12%. The loan will be repaid in equal annual principal payments at the end of each of the 6 years. Floatation costs of financing will be $ 1 lakh and the company is in a 30% tax bracket. What is the adjusted present value of the project ? Is the project acceptable ? (i) PVIF at 12% for 0 to 6 yrs. are 1.000 0.8928 0.7972 0.7118 0.6355 0.5674 0.5066 (ii) PVIFA for 6 years at 16% = 3.6847. Solution : Tutorial Notes: 1.The investment is $ 30 lakhs. 60% of it is financed by debt and 40% by equity capital. The debt of $ 18 lakhs will carry interest @ 12%; the interest will qualify for tax deduction. The loan is assumed to be taken at the beginning of the year while the interest is assumed to be paid at the beginning of the following year. 2. Floatation costs are not capital expenditure. This qualifies for tax deduction @ 30%. The effective floatation cost would be 70% of the actual cost. 3. Cash flows after taxes are directly given. Do not bother about depreciation. 4. The cash flows after tax are to be discounted at the cost of capital for arriving at the present value. fig. in $ lacs Tax saving is treated as cash inflow. The present value of tax shield is computed as below : Year Debt Interest @ 12% Tax shield @ 30% Pv factor Pv of tax saving 0 18.00 $ lacs 1 15.00 2.16 0.65 0.89 0.58 2 12.00 1.80 0.54 0.80 0.43 3 9.00 1.44 0.43 0.71 0.31 4 6.00 1.08 0.32 0.64 0.21 5 3.00 0.72 0.22 0.57 0.12 6 0 0.36 0.11 0.51 0.05 Present value of total tax shield : (cash inflow) 1.70 Present value of cash inflow @ $ 10.0 lacs for 6 years at 16% ( 10 x 3.6847) 36.85 Total present value of cash inflow 38.55 Effective present value of floatation costs (cash outflow) (1.0--0.3) 0.70 Present value of cash outflow (cost of the machine) 30.00 30.70 Net present value of the investment : 7.85 In view of the positive net present value to the tune of $ 7.85 lacs, the project appears to be acceptable. 2006 Dec 8 This is repeat of 2004 June.

2007 June 8

Trytonic Ltd. is considering a new project for manufacture of pocket video games involving a capital expenditure of Rs. 600 lakh and working capital of Rs. 150 lakh. The capacity of the plant is for an annual production of 12 lakh units and capacity utilisation during the 6-year working life of the project is expected to be as indicated below: Year 1 2 3 4 -- 6 Capacity Utilisation in % 33.33 66.67 90 100 The average price per unit of the product is expected to be Rs. 200 netting a contribution of 40%. The annual fixed cost, excluding depreciation, are estimated to be Rs. 480 lakh per annum from the third year, for the first and second year, it would be Rs. 240 lakh and Rs. 360 lakh respectively. The average rate of depreciation for tax purpose is 33.33% on the capital assets. The rate of income tax may be taken as 35%. The cost of capital is 15%. At the end of third year, an additional investment of Rs. 100 lakh would be required for working capital. Terminal value for the fixed assets may taken at 10% and for the current assets at 100%. For the purpose of your calculations, the recent amendments to tax laws with regard to balancing charge may be ignored. As a financial consultant, what recommendation on the financial viability of the project would you make to the Trytonic Ltd ? Note : Extracted from table Year Pv at15% 1 0.87 2 0.756 3 0.658 4 0.571 5 0.497 6 0.432

Solution : Tutorial notes : 1. Capital assets are to be depreciated at 33.33% p.a. The life of the project is 6 years. The terminal value is given as 10%. There will be either loss or profit on the basis of difference between the depreciated value and terminal value of capital assets. The tax implication of the profit/loss needs to be considered. In any case you should not depreciate the asset in 3 years by using straight line method. 2. The concern is ongoing concern. It is assumed that it is otherwise earning taxable income. Any loss or gain from this project would be included in that income. 3. The loss of Rs. 120 lakhs in first year of the project would result in tax saving of Rs. 42 lakhs. This is cash inflow in that year due to the project. The solution goes as follows: Working Notes: Statement of Depreciation @ 33.33% on Written down value Year 1 2 3 4 5 6 Rs. in lakhs Written down value 600.00 400.00 266.67 177.78 118.52 79.01 Depreciation 200.00 133.33 88.89 59.26 39.51 26.34 Written down value at the end of 6 years 52.67 Terminal value at the end of 6 years 60.00 Profit on sale of asset 7.33 Tax on profit on sale of asset @ 35% 2.56 Effective cash inflow due to sale of assets ( Terminal value -- tax) 57.44 Determination of Cash outflow : Rs. Lakhs Initial outlay for capital expenditure 600.0 Add : Working capital at the commencement 150.0 Working capital at the end of year 3 (100 x 0.658) 65.8 Present value of total cash outflow 815.8 Determination of Cash inflow : Rs. Lakhs Year 1 2 3 4 5 6 Capacity Utilisation % 33.33 66.67 90 100 100 100 Units sold in lacs 4 8 10.8 12 12 12 Sales revenue@ Rs. 200/unit 800.0 1600.0 2160.0 2400.0 2400.0 2400.0 Contribution @ 40% 320.0 640.0 864.0 960.0 960.0 960.0 Less : Fixed costs 240.0 360.0 480.0 480.0 480.0 480.0 Less : Depreciation 200 133.3 88.9 59.3 39.5 26.3 Earnings before tax (120) 146.7 295.1 420.7 440.5 453.7 Less : taxes @ 35% (42) 51.3 103.3 147.3 154.2 158.8

Earnings after tax (78) 95.3 191.8 273.5 286.3 294.9 Add: Recovery of Working capital 250.0 Add: Sale proceeds of fixed assets 57.4 Cash inflow after tax 122 228.7 280.7 332.7 325.8 628.6 Pv factor at 15% 0.87 0.76 0.66 0.57 0.50 0.43 Present value of cash inflow 106.1 172.9 184.7 190.0 161.9 271.6 Computation of Net present value : Present value of cash inflow 1087.2 Present value of cash outflow 815.8 Net present value 271.4 Recommendation : The net present value is positive to the extent of Rs. 271.4 lakhs. Trytonic Ltd. is advised to take up the project. Note : The loss of Rs. 120 lakhs in first year of the project would result in tax saving of Rs. 42 lakhs. This is cash inflow in that year due to the project.

1087.2

2007 Dec 5a: An analyst intends to value Infotech Limited in terms of the future cash generating capacity: Year 1 2 3 4 5 Cash flow Rs. million 176 48 64 86 117
It is further estimated that beyond 5th year, cash flows will perpetuate at a constant growth rate of 7% per annum, mainly on account of inflation. This perpetual cash flow is estimated to be Rs. 1026 million at the end of the 5th year. (i) What is the value of the company in terms of expected future cash flows? You may assume a cost of capital of 20% for your calculations. (ii) The company has outstanding debt of Rs. 362 million and bank balance of Rs. 271 million. Calculate the shareholder value, if the number of outstanding shares is 15.15 million.

(iii) The company has received a take over bid of Rs. 201 per share, is it a good offer. Solution : Tutorial notes: 1. This is very easy question. 2. Find the present values of all the cash flows for first five years at 20% cost of capital.
3. Find the present value of perpetual cash flows beyond fifth year. What formula will you use for this?

4. The value of the company will be sum of all the above present values. 5. Shareholders value will be : Value as per cash flows + Bank balance Debt.
6. No. of shares is given as 15.15 million and shareholder's value is determined in step 6 above, value per share is known. 7. Compare the bid offer with the value per share as determined above. If offer is more, it can be accepted.

Solution goes as follows: Year 1 2 3 4 5 Cash flow Rs. million 176 48 64 86 117 Pv factor at 20% 0.833 0.694 0.579 0.482 0.402 Pv of cash flows 146.6 33.3 37.1 41.5 47.0 305.5 Present value of perpetual flow of Rs. 1026 million beyond 5th year with growth rate 7%. = 1026 x [ ( 1.0 + 0.07) / (20% 7%) ] x 0.402 = Rs. 3395 million. Total value of the company = 305.5 + 3,395 = 3,700.5 million. Shareholder's value = 3,700.5 + 271 (bank) 362 (debt) = Rs. 3,609.5 million. No. of shares : 15.15 million hence value per share : 3,609.5 / 15.15 = Rs. 238.2
The offer price is Rs. 201 per share but the value is Rs. 238.2 per share. This is not a good offer and should therefore be declined.

A similar question was repeated in 2009 Dec 6b which goes as follows:


A financial analyst has been asked to appraise Networks Ltd., an IT company in terms of the future cash generating capacity. He has projected the following after-tax cash flows:

Year Cash flows Rs. lakhs

1 352

2 96

3 128

4 172

5 234

It is further estimated that beyond 5th year, cash flows will perpetuate a constant growth rate of 7% per annum, mainly on account of inflation. The perpetuate cash flow is estimated to be Rs. 2,052 lakh at the end of 5th year.

Additionally the following information are available : 1. The cost of capital is 20%. 2. The company has outstanding debt of Rs. 724 lakhs and cash/bank balance of Rs. 542 lakhs. 3. The number of outstanding shares of the company is 30.3 lakhs. Requirements: (i) What is the value of Networks Ltd. in terms of forecasted free cash flows? (ii) Calculate the value of shareholders. (iii) The company has received a take over bid of Rs. 402 per share, is it a good offer. 2007 Dec 7:
Milton thermoplastics ltd., a US based plastic manufacturer is considering a proposal to produce a high quality plastic glasses in India. The necessary equipment to manufacture the glasses would cost Rs. 10 million in India and it would last 5 years with depreciation at 25% per annum on written down value. The expected salvage value is Rs. 1.0 million (consider short term capital gain / loss for the income tax). The glasses will be sold at Rs. 4 each. Fixed cost will be Rs.2.5 million each year and variable cost Rs.2 per glass. The company estimates, it will sell 7.5 million glasses per year; tax rate in India is 35%. Milton ltd assumes 20% cost of capital for such a project. Additional working capital requirement will be Rs. 5.0 million. The company will be allowed 100% repatriation each year with a withholding tax rate of 10%. It is forecast that the Rupee will depreciate in relation to US dollar @ 2% per annum, with an initial exchange rate of Rs. 42 per dollar. Accordingly the exchange rates for the relevant 5 year period of the project will be as follows:

Year Exchange rate Rs.

0 42

1 42.84

2 43.7

3 44.57

4 45.46

5 46.37

Assume that no depreciation is charged in the terminal year. Advise Milton Ltd. regarding the financial viability of the proposal.

Solution Tutorial Notes 1. You have already solved questions of this type. This should present no difficulty. 2. Salvage value is not considered in wdv method. Compute depreciation and show it as working note.
3. Salvage value is given as Rs. 1.0 million. You should also compute the capital gain or loss at the end of life of equipment. Consider tax impact of such gain/loss. Don't forget to take the salvage value as cash inflow.

4. Exchange rates are different for different years. Be careful about them. Solution goes as follows:
Cash outflow is Rs. 10.0 million for equipment and Rs. 5.0 million for working capital totaling Rs. 15.0 million. This is in the zero year and the effective exchange rate is Rs. 42 per dollar. The initial cash outflow would be (15/42) 0.357 million dollars.

Year Sales revenue@ Rs.4 for 7.5 million Variable cost @ Rs. 2 Fixed costs Depreciation Total cost EBIT (Sales less costs) Less taxes at 35% EAT Cash flow (EAT + Depreciation) Working capital recovered Inflow due to salvage value Tax benefit on capital loss Withholding tax @ 10% Total pv of cash inflow

1 30 15 2.5 2.5 20.0 10.0 3.5 6.50 9.00

2 30 15 2.5 1.9 19.4 10.6 3.7 6.91 8.78

3 30 15 2.5 1.4 18.9 11.1 3.9 7.22 8.62

4 30 15 2.5 1.1 18.6 11.5 4.0 7.44 8.49

(0.90) 8.10

(0.88) 7.90

(0.86) 7.76

(0.85) 7.64

5 30 15 2.5 0 17.5 12.5 4.4 8.13 8.13 5.00 1.00 0.76 (0.81) 14.08

Repatriated amount (100%) 8.10 7.90 7.76 Exchange rate Rs. 42.84 43.7 44.57 Repatriated amount in million dollars 0.19 0.18 0.17 Pv factor at 20% 0.83 0.7 0.58 Pv in million dollars 0.16 0.13 0.10 Initial outlay in million dollars Net present value Working Notes: Wdv 10.0 7.5 Less depreciation @ 25% 2.5 1.9 Wdv at the end of 5 years 3.2 Salvage value 1.0 Capital loss 2.2 Tax saving in short term capital loss (@ 25%) 35% of 2.2 = 0.76 Additional Problem:

7.64 45.46 0.17 0.48 0.08

14.08 46.37 0.30 0.4 0.12

5.6 1.4

4.2 1.0

0.59 0.36 0.23 3.2 0.8

Problem : (ca final nov.06)


ABC Ltd. is considering a project in US, which will involve an initial investment of US$ 1,10,00,000. The project will have life 5 years of life. Current spot exchange rate is Rs. 48 per US$. The risk free rate in US is 8% and the same in India is 12%. Cash inflows from the project are as follows: Year 1 2 3 4 5 Cash inflows in US$ 20,00,000 25,00,000 30,00,000 40,00,000 50,00,000 Calculate the NPV of the project using foreign currency approach. Required rate of return : 14% Solution : The risk free rate in US is given, it has to adjusted for risk premium to arrive at the discount rate to be applied to cash inflows. We have (1 + 0.12) ( 1 + risk premium) = (1+0.14) which gives risk premium as 1.0179 Therefore, risk adjusted dollar rate = 1.0179 x 1.08 -- 1.000 = 0.099 or 9.90% Computation of Net present value: Year 1 2 3 4 5 Cash inflows in US$ (lacs) 20 25 30 40 50 Pv factors at 9.9% 0.91 0.828 0.753 0.686 0.624 Pv of inflows 18.2 20.7 22.59 27.44 31.2 Less : Initial Investment Net present value in US$ lacs Net present value in Rs.in lacs @ Rs. 48 per dollar

120.13 110.00 10.13 486.24

2008 June 6
Surat Paper Mills is considering setting up a power plant to minimize production losses that occurs due to frequent interruption of power supply. The proposed plant is contemplated to meet the power requirement of the duplex board paper manufacturing continuous process plant. The capital cost of the plant is estimated to be Rs.126 million with phasing of expenditure as given below:

Year 0 : Rs. 84 million and year 1 : Rs. 42 million


The capital cost will be met through company's own capital of Rs. 38 million and borrowing of balance amount from the financial institution at the interest rate of 8.85%.

The saving in electricity cost is projected as under : Year Generation Present cost Plant generation cost million kwh Rs./kwn Rs. / kwh 1 7.55 4.88 3.87 2 24.53 5.07 3.99 3 24.53 5.26 4.11 4 24.53 5.47 4.23 5 24.53 5.68 4.36 6 24.53 5.9 4.49 7 24.53 6.14 4.62

Saving Rs./kwh 1.01 1.08 1.15 1.24 1.32 1.41 1.52

8 9 Required :

24.53 24.53

6.37 6.63

4.76 4.90

1.61 1.73

(i) Do you think setting up a plant is a viable option for the company? Support your answer with necessary calculations. (ii) Also estimate levelized cost of generation per unit using the plant. You may ignore tax effect and assume cost of equity as 16%.

PVIF at 11% for 0 to 9 years 1.000 0.9009 0.8116 0.7312 0.6587 0.5935 0.5346 0.4817 0.4334 0.3909 PVIF at 16% for 0 to 9 years 1.0000 0.8621 0.7432 0.6407 0.5523 0.4763 0.4104 0.3538 0.305 0.263 PVIFA for 9 years at 11% : 5.5370 and at 16% 4.6065 Solution ; 1. What rate of discounting will you use? 2. What do you understand by 'levelised cost of generation'? First you have to determine the discounting rate. The company will invest Rs. 38 million at 16% rate of interest and the balance of Rs. 88 million will be taken as loan at 8.85%. The composite cost of capital should be taken as discount rate. Compute the discount rate and show it as working note. Determination of composite cost of capital : Equity capital 38 16% 6.08 Loan 88 8.85% 7.79 126 13.87 Composite cost of capital : 13.87 / 126 = 11% Year Capital Generation Saving Saving Net DF at 11% Pv of million kwh Rs./kwh million Rs. Cash inflow cash flow 0 (84) (84) 1.00 (84.00) 1 (42) 7.55 1.01 7.63 (34.37) 0.90 (30.97) 2 24.53 1.08 26.49 26.49 0.81 21.50 3 24.53 1.15 28.21 28.21 0.73 20.63 4 24.53 1.24 30.42 30.42 0.66 20.04 5 24.53 1.32 32.38 32.38 0.59 19.22 6 24.53 1.41 34.59 34.59 0.53 18.49 7 24.53 1.52 37.29 37.29 0.48 17.96 8 24.53 1.61 39.49 39.49 0.43 17.12 9 24.53 1.73 42.44 42.44 0.39 16.59 Expected surplus of cash flow 36.57 Owing to the surplus fund to the tune of Rs. 36.57 million, the project is viable one and is recommended to be taken up. Levelised cost of generation : You should compute it before you see below: Cost Pv factor 3.87 3.99 4.11 4.23 4.36 0.90 0.81 0.73 0.66 0.59 3.49 3.24 3.01 2.79 2.59 Levelised cost of generation = 23.71 / 5.54 = Rs. 4.28 per kwh Check : Cost 4.28 4.28 4.28 4.28 4.28 Pv factor 0.90 0.81 0.73 0.66 0.59 3.86 3.47 3.13 2.82 2.54 2008 Dec 5: 4.49 0.53 2.40 4.62 0.48 2.23 4.76 0.43 2.06 4.90 0.39 1.92

4.28 0.53 2.29

4.28 0.48 2.06

4.28 0.43 1.85

4.28 0.39 1.67

XYZ Ltd. is considering two mutually exclusive projects. Both require an initial cash outlay of Rs. 10,000 each for machinery and having a life of 5 years. The company's required rate of return is 10% and it pays tax at 50%. The project will be depreciated on a straight line basis. The net cash flows (before taxes) expected to be generated by the projects and the present value of factor at 10% are given below: Year 1 2 3 4 5 Project 1 Rs. 4,000 4,000 4,000 4,000 4,000 Project 2 Rs. 6,000 3,000 2,000 5,000 5,000 Pv factor at 10% 0.909 0.826 0.751 0.683 0.621 You are required to calculate : (i) the pay back period of each project and (ii) the npv and profitability index of each project Solution : Year 1 2 3 4 5 Cash flows 4,000 4,000 4,000 4,000 4,000 Less : Depreciation 2,000 2,000 2,000 2,000 2,000 EBT 2,000 2,000 2,000 2,000 2,000 Less : tax at 50% 1,000 1,000 1,000 1,000 1,000 Net income 1,000 1,000 1,000 1,000 1,000 CASh flows after tax 3,000 3,000 3,000 3,000 3,000 Cumulative cash flows 3,000 6,000 9,000 12,000 15,000 Payback period would be the time when initial investment is recovered in cash. The investment is Rs. 10,000. Payback period would be between 3 and 4 years. By interpolation it would be 3.33 years. Year 1 2 3 4 5 Cash flows 6,000 3,000 2,000 5,000 5,000 Less : Depreciation 2,000 2,000 2,000 2,000 2,000 EBT 4,000 1,000 0 3,000 3,000 Less : tax at 50% 2,000 500 0 1,500 1,500 Net income 2,000 500 0 1,500 1,500 CASh flows after tax 4,000 2,500 2,000 3,500 3,500 Cumulative cash flows 4,000 6,500 8,500 12,000 15,500 Payback period would be between 3 and 4 years. By interpolation it would be 3.43 years. Additional Problems :
Problem (cwa inter II June 00) XYZ ltd. Is considering purchase of a machine in replacement of an old one. Two models viz. Molin and Skoda are offered at prices of Rs. 22.50 lakhs and Rs. 30 lakhs respectively. Further particulars regarding these models are given below: Particulars Molin Skoda Economic life in years 5 6 Scrap value at the end of economic life Rs. In lakhs 2 2.5 After tax annual cash inflows Rs. In lakhs Year 1 2 3 4 5 6 Molin 5.0 7.5 10.0 9.0 8.5 0.0 Skoda 6.0 8.0 10.0 12.0 10.5 9.5 Pv at 12% 0.893 0.797 0.712 0.636 0.567 0.507 Evaluate the two proposals under (I) Pay back period (ii) Net present value method Which model would you recommend and why? Solution : Computation of payback period Molin Cost Rs. 22.50 lakhs Skoda Cost Rs. 30.0 lakhs End of yr Cash inflow Cumulative Cash inflow Cumulative Rs. Lacs Rs. Lacs Rs.lacs Rs. Lacs 1 5.0 5.0 6.0 6.0 2 7.5 12.5 8.0 14.0

3 10.0 22.5 10.0 4 9.0 31.5 12.0 5 8.5 40.0 10.5 6 0.0 9.5 Pay back period 3 years Recommendation : Molin model having less pay back period, is recommended. Limitations. 1.Time value of money is not considered. 2. Cash inflow after the pay back period is ignored. Computation of Net present value Molin End of year discount factor Inflow Pv Inflow 1 0.893 5.0 4.47 6.0 2 0.797 7.5 5.98 8.0 3 0.712 10.0 7.12 10.0 4 0.636 9.0 5.72 12.0 5 0.567 8.5 4.82 10.5 6 0.507 0.0 0.00 9.5 scrap (2x 0.567) 1.13 2.5 Total 29.24 Less initial investment 22.50 Net present value 6.74 Skoda model having more net present value is recommended.

24.0 36.0 46.5 56.0 3.5 years

Skoda fig. in Rs. Lakhs Pv 5.36 6.38 7.12 7.63 5.95 4.82 1.27 38.52 30.00 8.52

2009 June 5b: Vedika Ltd. with a limited investment funds of Rs. 600,000 is evaluating the desirability of 5 (five) investment proposals. Their profiles are summarized below: Project Investment Rs. Annual cash flow after tax Life in years M 100,000 36,000 10 N 200,000 100,000 40 O 200,000 60,000 8 P 300,000 80,000 16 Q 400,000 60,000 25 Project N and Q are mutually exclusive. The cost of funds is 10%. Required : (i) Find out the feasible combination of projects and rank them on the basis of net present value. Year 10 4 8 16 25 PVIFA at 10% 6.145 3.170 5.335 7.824 9.077 Solution : Project Investment Cash flow Annuity Pv (cash flow x annuity) Npv (pv invest.) M 100,000 36,000 6.145 221,220 121,220 N 200,000 100,000 3.170 317,000 117,000 O 240,000 60,000 5.335 320,100 80,100 P 300,000 80,000 7.824 625,920 325,920 Q 400,000 60,000 9.077 544,620 144,620 Life of project is not relevant in determination of npv. Statement of feasible combination : Combination Investment Npv Rank M,N and P 600,000 564,140 1 M, N and O 540,000 318,320 4 O and P 540,000 406,020 3 M and Q 500,000 265,840 5 N and P 500,000 442,920 2 N and Q 600,000 261,620 6 Additional Problem:
Problem: (ca final nov.98)

S Ltd. has Rs. 10,00,000 allocated for capital budgeting purposes. The following proposals and associated indexes have been determined: Project Amount Rs. Profitability Index 1 300,000 1.22 2 150,000 0.95 3 350,000 1.20 4 450,000 1.18 5 200,000 1.20 6 400,000 1.05 Which of the above investments should be undertaken? Assume that projects are indivisible and there is no alternative use of money allocated for capital budgeting. Solution : Statement showing ranking of projects on the basis of Profitability index Project Amount Rs. Profitability Index Rank 1 300,000 1.22 1 2 150,000 0.95 5 3 350,000 1.20 2 4 450,000 1.18 3 5 200,000 1.20 2 6 400,000 1.05 4 If PI is made the basis of selection of projects for capital budgeting purposes, the S ltd. is advised to under take the project 1 (Rs. 300,000), 3 (Rs. 350,000) and project 5 (Rs. 200,000). The selection should add most to the shareholder's wealth, the Net present value method is appropriate with this objective. Statement showing NPV of the projects Cash inflows Npv of project Rs. Project Amount Rs. Profitability Index 1 300,000 1.22 366,000 66,000 2 150,000 0.95 142,500 (7500) 3 350,000 1.20 420,000 70,000 4 450,000 1.18 531,000 81,000 5 200,000 1.20 240,000 40,000 6 400,000 1.05 420,000 20,000 The allocation of funds to the projects 1,3 and 5 on the basis of PI, will give Npv of Rs. 176,000 and Rs. 150,000 will remain unspent. However, the Npv of the projects 3,4 and 5 is Rs. 191,000 which is more than the combined Npv of projects 1,3 and 5 (Rs. 176,000). Further, by undertaking the projects 3,4 and 5, no money remains unspent. 2000 June 6 A product is currently manufactured in a machine that is not fully depreciated for tax purpose and has a book value of Rs. 60,000 ( it was purchased for Rs. 1,20,000 six years ago). The product cost is as under : Direct cost : Rs. 24. Indirect labour : Rs. 8. Variable overheads : Rs. 16. Fixed overheads : Rs. 16. 10,000 units of the product are normally produced. It is expected that the old machine can be used indefinitely into the future, after suitable repairing estimated to cost Rs. 40,000 annually is carried out. There is an offer for a new machine with latest improved technology at Rs. 300,000 after trading off the old machinery for Rs. 30,000. The projected cost of the product will then be as under : Direct cost : Rs. 14. Indirect labour : Rs. 12. Variable overheads : Rs. 12. Fixed overheads : Rs.20. The fixed overheads are allocations from other departments plus the Depreciation of plant and machinery. The old machine can be sold in the open market for Rs. 40,000. The new machine will last for 10 years at the end of which it will have a salvage value of Rs. 20,000.

Assume the rate of corporate tax at 50%. For the tax purpose the cost of new machine and that of the old one may be depreciated in 10 years. The minimum rate of return expected is 10%. It is also expected that the future demand of the product will remain steady at 10,000 units. Advise whether the new machine should be purchased. Ignore capital gains. Present value of Rs. 1 at 10% for 10 years are : year 1 2 3 4 5 6 7 8 9 0.909 0.826 0.751 0.683 0.621 0.564 0.513 0.467 0.424 Solution : Tutorial notes: 1. What is the depreciation of new machine per year ? What is the cost to be depreciated ? Cost of the machine is not Rs. 300,000. The cost is Rs. 300,000 + Rs. 30,000. The salvage value is Rs. 20,000. Thus the cost to be depreciated in 10 years is Rs. 330,000 -- Rs. 20,000 i.e. Rs. 310,000. The depreciation per year would be Rs. 31,000. 2. Fixed overheads other than depreciation are not relevant for decision making. 3. The special item in this question is absence of selling price and sales revenue anywhere. If sales revenue is not there what would be cash inflow which is to be discounted ? Solution : Evaluation of existing and new machine on basis of cost and cash outflow Particulars Existing New No. of units 10,000 10,000 Rs. Rs. Variable cost per unit Rs. 48 38 Total variable cost 480,000 380,000 Add: annual repairs 40,000 Add: depreciation 6,000 31,000 Total cost 526,000 411,000 Tax saving @ 50% on cost 263,000 205,500 Less Depreciation (non cash item) 6,000 31,000 Cash outflow 257,000 174,500 Net saving in cash outflow 82,500 Thus new machine would result in net saving in cash outflow of Rs. 82,500 per year for 10 years. Present value of saving of Rs. 82,500 per year for 10 years*= 82,500 x 6.144 * 82,500 ( 0.909 +0.826 +0.751 +0.683 +0.564 +0.513 +0.467 +0.424 +0.386) Present value of cash inflow of Rs. 20,000 in tenth year (20,000 x 0.386) Present value of cash outflow on purchase of new machine Net present value ( 506,880 + 7,720 -- 300,000) Net present value being positive, the new machine is recommended to be purchased. Rs. 506,880 7,720 300,000 214,600

10 0.386

2009 Dec 3b Ankit Ltd. a manufacturing company produces 25,000 liters of special lubricants in its plant. The existing plant is not fully depreciated for tax purposes and has a book value of Rs. 3.0 lakhs (it was bought for Rs. 6.0 lakh six years ago). The cost of the product is as under : Variable cost : Rs. 60.0 per liter and Fixed overheads : Rs. 15 per liter Total : Rs. 75 per liter. It is expected that the old machine can be used for further period of 10 years by carrying out suitable repairs at a cost of Rs. 2 lakh annually. A manufacturer of machinery is offering a new machine with the latest technology at Rs. 10 lakh after trading off the old machine for Rs. 1.0 lakh. The projected cost of the product with new machine will then be : Variable cost : Rs. 45.0 per liter and Fixed overheads : Rs. 20 per liter Total : Rs. 65 per liter. The fixed overheads are allocations from other departments plus the depreciation of plant & machinery.

The old machine can be sold for Rs. 2 lakh in the open market. The new machine is expected to last for 10 years at the end of which, its salvage value will be Rs. 1 lakh. Rate of taxation is 50%. For the tax purposes, the cost of the new machine and that the old one may be depreciated in10 years. The minimum rate of return expected is 10%. It is also anticipated that in future the demand for the product will remain at 25,000 liters. Advise whether new machine should be purchased, Ignore capital gains or losses.
Present value of Rs. 1 at 10% for 10 years are : year 1 2 3 4 0.909 0.826 0.751 0.683 5 0.621 6 0.564 7 0.513 8 0.467 9 0.424 10 0.386

Solution : Solved on lines of previously problem. Salient figures are given below. Year Cash flows Rs. lacs Pv factor 0 Out flow due to cost 10.0 1 1 10 Annual saving 3.225 6.145 10 Salvage 1.0 0.386 Net present value

Pv (10) 19.82 0.38 10.20

ailable.

Machine S

project will have life 5 in India is 12%. Cash

plied to cash inflows.

5.54 23.71

5.54 23.70

ed indexes have been

no alternative use of

er take the project 1

with this objective.

150,000 will remain

cts 1,3 and 5 (Rs.

Chapter 3 : Financial services and Lease Financing


Evaluation of lease or borrow option 1. The cost of capital is generally given. Sometimes the question is silent about whether the cost of capital is aftertax or before-tax. The student should assume that the cost of capital is after-tax or before-tax, and such assumption must clearly be stated in your answer. 2. The option which is better may change if rate of discounting is changed from before-tax to after-tax. It may happen that after-tax cost of capital may favour leasing option while before-tax cost of capital may favour purchasing option. Sometimes both the rates may favour the same option. 3. The depreciation of asset may also present some confusion. AS-6 issued by the ICAI clearly states that Original cost less salvage value should be depreciated over the life of the asset if SLM is followed. Original cost should be depreciated if WDV method is followed. The salvage value should be treated as cash inflow at the end of life of the asset and be discounted accordingly. 4. The salvage value should always be compared with the book value at the end of life. The difference between the two will either be short term capital gain or loss. If it is a loss, it will lead to tax saving which should be treated as cash inflow and be discounted accordingly. It is a gain, tax will be paid on it and after tax value should be taken as cash inflow. 5. It should be assumed that the company is already making adequate profits and paying taxes at the mentioned rate. Any loss or gain from the current venture under consideration will simply be clubbed with the existing income. 6. The choice between the machines having different costs of investment should be based on profitability index.

2001 Dec 6b:


ABC Ltd. is considering a proposal to acquire an equipment costing Rs. 5,00,000. The expected effective life of the equipment is 5 years. The company has two options - either to acquire it by obtaining a loan of Rs. 5 lakhs at 12% interest per annum or by lease. The following information is available: (i) The principal amount of loan will be repaid in 5 equal installments; (ii) the full cost of equipment will be written off over a period of 5 years on straight line basis and it is to be assumed that such depreciation charge will be allowed for tax purpose; (iii) the effective tax rate for the company is 40% and the after-tax cost of capital is 10%; (iv) the interest charge, repayment of principal and the lease rental are to be paid on the last day of each year. You are required to work out the amount of lease rental to be paid annually, which will match the loan option. The discount factor at 10% are as follows: Year 1 2 3 4 5 Discount factor 0.909 0.826 0.751 0.683 0.621 Solution : The interest on loan and depreciation on asset both qualify for deductions for tax purposes. The savings on these two accounts will reduce the cash outflow. Calculation of Interest and Depreciation on Equipment : Rs. In thousands Principal amount at Repayment at the Principal at the end of Interest @ Year the beginning of the end of the year the year 12% Depre. year 1 500 100 400 60 100 2 400 100 300 48 100 3 300 100 200 36 100

4 200 100 100 5 100 100 0 Calculation of Present Value of Cash outflow Year Principal Interest Depre. Tax Benefit on Payment Interest Deprec. 1 100 60 100 24.00 40.00 2 100 48 100 19.20 40.00 3 100 36 100 14.40 40.00 4 100 24 100 9.60 40.00 5 100 12 100 4.80 40.00

100 100 Rs. In thousands Total benefit Net cash Pv factor Present benefit outflow Value 64.00 96.00 0.909 87.26 59.20 88.80 0.826 73.35 54.40 81.60 0.751 61.28 49.60 74.40 0.683 50.82 44.80 67.20 0.621 41.73 3.79 314.44 Thus the lease rental should equate to after-tax cash outflow having present value of Rs. 314,440 After tax lease rental = 314,440/ 3.79 = Rs. 82,966. Before tax lease rental for tax rate of 40% = Rs. 82,996 / 0.60 = Rs. 138,277

24 12

2003 June 14b: ABC Finance Ltd. is a hire-purchase and leasing company. It has been approached by a firm interested in acquiring a machine through leasing. The quoted price of the machine is Rs. 500,000. 10% sales tax extra. The lease will be for a primary lease period of 5 years. The finance company wants 8% post-tax return on the outlay. Its effective tax rate is 35%. The income tax rate of depreciation on the machine is 25% on WDV. Lease rents are payable in arrear at the end of each year. Calculate the annual rent to be charged by ABC. Solution : Tutorial notes 1. What should be the cash outflow at the beginning ? Rs. 500,000 ? Or Rs. 500,000 + 10% sales tax ? Or some other figure? Think before you proceed further. 2. The company wants 8% post tax return. Post tax cash flows should be taken and should be discounted at 8%. You should compute discount factors for five years at 8% and show it as working note. 3. Lease rents are tax-deductible expenses. Tax rate is given as 35%. If lease rent is X, then effective lease rent for discounting purposes would be 0.65 X. 4. As the lease rents are same for the lease period of five years, you can take annuity factor for five years in your computations. 5. The discount factors and annuity factor for 5 year at 8% are as follows: Year 1 2 3 4 5 Discount factor 0.93 0.86 0.79 0.74 0.68 3.99 If lease rent is Rs. A per annum, then you can compute A with the following equation 0.65 A x 3.99 = Pv of cash outflow at the beginning. The question is what is cash outflow? 6. The machine is to be depreciated at 25% per annum on WDV basis. What figure should be depreciated Rs. 500,000 or Rs. 550,000? 7. Depreciation would provide tax relief at 35% which will reduce the cash outflows. Compute depreciation and tax relief on it and solve the question. The solution goes as follows (you try first and then see below). Cost of machine Rs. 500,000 Add : Sales tax at 10% 50,000 Total cost to be depreciated 550,000 Computation of tax relief on depreciation :

Year Wdv Depreciation at 25% Tax saving at 35% Df at 8% Pv of saving 1 550,000 137,500 48,125 0.93 44,560 2 412,500 103,125 36,094 0.86 30,945 3 309,375 77,344 27,070 0.79 21,489 4 232,031 58,008 20,303 0.74 14,923 5 174,023 43,506 15,227 0.68 10,363 Present value of tax relief due to depreciation 122,280 If lease rent is Rs. A per annum, then you can compute A with the following equation 0.65 A x 3.99 = 550,000 122,280 Which gives A = Rs. 164,508 2003 Dec 3
As an investment manager you are given the following information : Investment in shares of Cement Ltd. Steel Ltd. Liquor Ltd. GOI Bonds You are required to calculate : (i) Expected rate of returns of portfolio in each using CAPM. (ii) Average return of portfolio. Solution : Required rate of return = Risk free return + beta x ( market return risk free return) The expected rate of return or required rate of return mean the same here. In CAPM formula shown above; you don't know the market return. Can you think of some way to determine the market returns from the data given. Determination of Market rate of return : Initial price End price Capitan gain Rs. Cement Ltd. Steel Ltd. Liquor Ltd. GOI Bonds 25 35 45 1,000 1,105 Market rate of return = 291 / 1,105 = 26.3% Determination of Required rate of return as per CAPM : Stock Cement Ltd. Steel Ltd. Liquor Ltd. GOI Bonds Computation 14+ 0.8 x (26.3 14) 14+ 0.7 x (26.3 14) 14+ 0.5 x (26.3 14) 14+ 0.99 x (26.3 14) Average rate 92.9 / 4 Alternatively : Average of Betas : ( 0.8 + 0.7 + 0.5 + 0.99) / 4 = 0.75 Required returns % 23.9 22.6 20.2 26.2 92.9 23.2 Rs. 50 60 135 1,005 25 25 90 5 Dividend Rs. 2 2 2 140 27 27 92 145 291 Total Initial price Rs. 25 35 45 1000 Dividend Market price at the end of year Rs. 2 2 2 140 Rs. 50 60 135 1005 Beta 0.8 0.7 0.5 0.99

Risk free return may be taken as 14%.

Average return = 14 + 0.75 x (26.3 14)

23.2%

2004 June 3 A portfolio manager purchased 1,000 equity shares of Reliance Industries at Rs. 510 per share. He wants to hedge the position by writing up April call with a strike price of Rs. 530 and call premium Rs. 10. Alternatively, he wants to hedge by buying put option of strike price Rs.510 and premium of Rs. 10. (a) Find out his profit or loss if the share price goes up to Rs. 540. (b) Find out his profit or loss if the share price goes up to Rs. 525 till the date of expiration of the option. (c) Find out his profit or loss if the share price comes down to Rs. 490. (d) Does the strategy of buying a stock and writing a call manage his risk effectively? (e) Under which circumstance should the portfolio manager buy a put option? (f) Assume that the share price goes down to Rs. 490, what will be the profit or loss of his portfolio if he buys put option? In case he buys one Rs. 510 put (premium Rs. 10) what will be his profit or loss for the share price of Rs. 540, Rs. 530, Rs. 520 and Rs. 490? Solution : Take the same answer.
Problem 2 (cwa final june.04) LB Ltd. has decided to acquire machine M costing Rs. 63,000. It will have an operational life of 4 years with no scrap value. Tax is payable at 30% on operating cash flows in the same year. Capital allowances are available at 25% a year on a reducing balance basis. The company has the opportunity either to purchase the machine or to lease it under a finance lease arrangement, at an annual rent of Rs. 20,000 for four years, payable at the end of each year. The company can borrow to finance the acquisition at 10%. Should the company lease or buy the machine ? Solution : Tutorial notes: No cash inflows have been given. Thus the cash inflows will be in the form of tax benefits arising out of availment of both the options. What will be the % of discounting the cash flows ? The company can borrow the funds at 10%. The interest on borrowing would attract tax benefit. The effective rate of discounting would not be at 10%. Solution goes as follows: Capital allowance will give tax benefits as per the following table :tax rate 30%, life 4 yrs. Year Allowance available Rs. Tax benefit @ 30% 1 25% of Rs. 63,000 15,750 4,725 2 75% of Rs. 15,750 11,813 3,544 3 75% of Rs. 11,813 8,859 2,658 36,422 4 Balance (Rs. 63,000 -- Rs. 36,422) 26,578 7,973 Net present value under option of buying :(marginal cost of borrowing 10%(1-0.3) = 7% Year 1 2 3 4 Rs. Cash inflows 4,725 3,544 2,658 7,973 Pv factor at 7% 0.93 0.87 0.82 0.76 Pv of cash inflows 4,416 3,095 2,170 6,083 15,764 Pv of cash outflows at yr 0 63,000 Net present value of cash outflow 47,236

Net present value of cash outflows under leasing option: The lease rentals qualify for tax benefits. Lease rent is Rs. 20,000 per year. This will save tax of Rs. 6,000 per year. Thus effective cash outflows per year will be Rs. 14,000. Year 1 2 3 4 Cash outflows 14,000 14,000 14,000 14,000 Pv factor at 7% 0.93 0.87 0.82 0.76 Pv of cash outflows 13,084 12,228 11,428 10,681 47,421 The net present value of cash outflow is marginally lower by Rs.185 in case of buying. Buying is recommended. Alternatively Year 1 2 3 4 Cash outflows Leasing 14,000 14,000 14,000 14,000 Cash inflows Buying 4,725 3,544 2,658 7,973 Differential cash flows 18,725 17,544 16,658 21,973 Pv factor at 7% 0.93 0.87 0.82 0.76 Pv of cash outflows 17,500 15,324 13,598 16,763 63,185 Cash outflow under buying 63,000 The net present value of cash outflow is marginally lower by Rs.185 in case of buying. Buying is recommended.

2005 June 4: The following data are furnished by the Sigma Leasing Ltd.: Investment cost : Rs. 99 lakhs; Primary Lease Term : 3 Years and Residual value : Nil Pretax required rate of return : 22% per annum. The lease can be renewed for an additional period of 3 years (secondary lease). The lease rental for the secondary period will be 5% of the rental charged during the primary period. Sigma Ltd. seeks your advice in determining the annual lease rental under the following rental structures: (a) Equated (b) Stepped (annual increase 12%) (c) Ballooned (annual rental of Rs. 15 lakhs for year 1 and 2) (d) Deferred (deferment period of 1 year) You are required to compute the annual rentals under four rental structure. Show your workings. Solution : 1. Pv of lease rent (say Y) for three years plus lease rent (0.05Y) for next 3 years would be equal to initial outlay of Rs. 99 lakhs. 2. You must compute the discount factors for 22% as follows Year 1 2 3 4 5 6 DF 0.82 0.67 0.55 0.45 0.37 0.30 Lease rent for equated rental would be given by : Y (0.82 + 0.67 + 0.55) + 0.05 Y (0.45 + 0.37 + 0.30) = 99 Which gives Y as 47.2 lakhs. Lease rental would be : Year 1,2 and 3 as Rs. 47.2 lakhs and for year 4,5,6 as Rs. 2.36 (5% of 47.2) 2. In this case the lease period is 6 years and lease rent increases by 12% per year. Timing of payment has not been given. It is assumed that first lease rent is paid at time t = 0 i.e. when asset is taken. The rental structure would be as follows: Year DF Rent 1 0.82 Y 1 2 0.67 1.12Y 1.12 3 0.55 2 1.12 Y 1.25 4 0.45 1.123 Y 1.40 5 0.37 1.124 Y 1.57 6 0.30 1.125 Y 1.76

0.82 0.75 0.69 0.63 0.58 0.53 4.01 Lease rent would be given by : Y (0.82 + 0.75 + 0.69 + 0.63 + 0.58 + 0.53) = 99 Which gives 4.01 Y = 99 giving Y = 24.67 lakhs. 3. Ballooned structure means Rs. 15 lakhs for first two year, Y lakhs for third year and 0.05 Y for 3 5 years (secondary lease). Year 1 2 3 4 5 6 DF 0.82 0.67 0.55 0.45 0.37 0.30 Lease 15 15 Y 0.05Y 0.05Y 0.05Y Lease equation is 15 ( 0.82 + 0.67) + Y (0.55) + 0.05Y (0.45 + 0.37 + 0.30) = 99 This equation gives Y = Rs. 126.2 lakhs. 4. Deferred payment would be like the following : Year 1 2 3 4 5 6 DF 0.82 0.67 0.55 0.45 0.37 0.30 Lease 0 Y Y 0.05Y 0.05Y 0.05Y Lease equation is Y (0.67) + Y (0.55) + 0.05Y (0.45 + 0.37 + 0.30) = 99 Which gives Y = Rs. 77.4 lakhs. 2003 Dec. 3a
Bansali textile has annual sales of Rs. 200 crores. About 80% of its sales is on credit, and the average collection is 90 days. The company's bad debts, as the past trend reveals, are around 0.9% of credit sales. The company's annual cost of administering credit sales is Rs.75 lacs. It is possible to save Rs.55 lakhs, out of the bad debts and sales administering costs, if the company avails of full-factor service from a factoring company. The company has approached a factoring company and got the following terms : Advance payment : 80%. Discount rate : 14% per annum. Commission for service : 1.0% to be paid upfront. (i) What will be the effective cost of factoring on an annual basis assume 360 day in a year) (ii) Bansali textile can borrow the advance payment offered by the factoring company from a bank at 14% p.a. Should the company avail of the factoring service ? Give reasons. Solution : Tutorial notes What is meant by discount rate ? How much would be the advance payment ? What is upfront ? (dictionary meaning of upfront is direct, frank, straight. Upfront is commission.) You are advised to revise your knowledge of factoring. Rs. Crores Annual Credit sales 80% of Rs. 200 crores 160 Average receivables for collection period of 90 days = 160 x (90/360) 40 Factoring costs are as below : Advance payment : 80% of average receivables ( 0.80 x 40) 32.00 Less: Discount (32 x 0.14 x 90/360) 1.12 Commission for service : (0.01 x 40) 0.40 Effective collection 30.48 Add: Saving in admin. Cost 0.55 Total receivable against 32 crores for a period of 90 days 31.03 Cost being paid every 90 days for collection of 31.03 crores 0.97 Cost in % for 90 days period (0.97/31.03) 3.13% Cost of acquiring factoring services in % p.a. (0.97 x 90/360) / 31.03 12.50% The company will pay 12.50% interest to factoring company for availing services whereas it will have to pay 14% to the bank. The factoring services are economical to the company hence recommended for availment.

2005 June 5a:

The turnover of Bharat Ltd. is Rs. 240 crores of which 80% is on credit. Debtors are allowed 90 days to clear off dues. The company's annual cost of administering credit sales is Rs. 90 lakhs. It is possible to save Rs. 66 lakhs out of the sales administering costs and avoid bad debts at 1% on credit sales (dues) if the co avails a full-factor service from a factor company. The company has approached a factor company and got the following terms: Advance payment : 90% Discount rate : 15% p.a. and commission : 1% to be paid upfront. A bank has come forward to make advance payment of 90% of the debts at an annual interest rate of 13%. Should the company avail of the factoring service or the offer of the bank? Give reasons. Assume 360 days in a year. Solution : Solve on the lines of previous problem. Salient figures are supplied below: Net Factoring cost : 43.20 1.62 0.48 (commission) + 0.66 + 0.48 (bad debts) = 42.24 Annual cost of factoring : 9.41% which is less than bank rate of 13%. Factoring is recommended. 2006 June 4b
MB Leasing Company has been approached by a client to write a 5-year lease on an equipment. The equipment is eligible for depreciation at 25% for Income tax purpose. In the terminal year, the client will be required to pay 1% of the equipment cost to acquire the ownership of the asset. The post tax return of the leasing company is 12%. Assuming that the lessor is subjected to a Corporate tax rate of 35%, calculate pre-tax annual lease rental payable in arrear, and express the same in terms of standard lease quotation i.e. rupees per Thousand per month. Note :Extracted from the table: (i) The present value factors at 12% discount rate for 0 to 5 years are 1.000, 0.8928, 0.7972, 0.7118, 0.6355, 0.5674. (ii) The present value factor of an annuity of Re.1 for 60 months at 12% = 44.955 Solution : Tutorial notes : 1. There is no cash inflow at a glance. The cash inflow is in the form of tax saving due to depreciation. The leasing company is owner and is entitled to claim depreciation as expense. 2. The quotation is to be presented is rupees per Thousand per month. The asset value should therefore be taken as Rs. 1000. 3. The present value factor of an annuity of Re.1 for 60 months has been given. Why months ? Think of it. Where this factor is going to be used ? Opening value depreciation @ 25% Pv@12% 1,000 250.0 0.8928 750.0 187.5 0.7972 562.5 140.6 0.7118 421.9 105.5 0.6355 316.4 79.1 0.5674 Total present value of depreciation Tax saving due to depreciation @ 35% Present value of Cash inflow at the end of five years @ 1% of asset value = 1% of Rs. 1,000 x 0.5674 Computation of post-tax rentals: Asset Value Rs. Less : Tax saving Less : Residual cash inflow Net post tax rentals Post tax lease rental in Rs. per thousand per month (789.69/44.955) Pre-tax lease rental (tax rate 35%) (17.57/0.65) Problem 4 (cwa final dec.01) Pv 223.2 149.5 100.1 67.0 44.9 584.7 204.6 5.67 1,000 204.64 5.67 789.69 17.57 27.03

ABC Ltd. is considering a proposal to acquire an equipment costing Rs. 5,00,000. The expected effective life of the equipment is 5 years. The company has two options - either to acquire it by obtaining a loan of Rs. 5 lakhs at 125 interest per annum or by lease. The following information is available: (i) The principal amount of loan will be repaid in 5 equal installments; (ii) the full cost of equipment will be written off over a period of 5 years on straight line basis and it is to be assumed that such depreciation charge will be allowed for tax purpose; (iii) the effective tax rate for the company is 40% and the after-tax cost of capital is 10%; (iv) the interest charge, repayment of principal and the lease rental are to be paid on the last day of each year. You are required to work out the amount of lease rental to be paid annually, which will match the loan option. The discount factor at 10% are as follows: Year 1 2 3 4 5 Discount factor 0.909 0.826 0.751 0.683 0.621 Solution : The interest on loan and depreciation on asset both qualify for deductions for tax purposes. The savings on these two accounts will reduce the cash outflow. Calculation of Interest and Depreciation on Equipment : Rs. In thousands Year Principal amount at Repayment at the Depre. Principal at the end of Interest @ 12% the beginning of the end of the year the year 1 500 100 400 60 100 2 400 100 300 48 100 3 300 100 200 36 100 4 200 100 100 24 100 5 100 100 0 12 100 Calculation of Present Value of Cash outflow Rs. In thousands Total benefit Net cash Pv factor Present Year Principal Interest Depre. Tax Benefit on benefit Payment Interest Deprec. outflow Value 1 100 60 100 24.00 40.00 64.00 96.00 0.909 87.26 2 100 48 100 19.20 40.00 59.20 88.80 0.826 73.35 3 100 36 100 14.40 40.00 54.40 81.60 0.751 61.28 4 100 24 100 9.60 40.00 49.60 74.40 0.683 50.82 5 100 12 100 4.80 40.00 44.80 67.20 0.621 41.73 3.79 314.44 Thus the lease rental should equate to after-tax cash outflow having present value of Rs. 314,440 After tax lease rental = 314,440/ 3.79 = Rs. 82,966. Before tax lease rental for tax rate of 40% = Rs. 82,996 / 0.60 = Rs. 138,277 Problem 6 (cwa final may 96) Welsh Ltd. is faced with decision to purchase or acquire on lease a mini car. The cost of mini car is Rs. 1,26,965. It has a life of 5 years. The mini car can be obtained on lease by paying equal lease rentals annually. The leasing company desires a return of 10% on the gross value of the asset. Welsh Limited can also obtain 100% finance from its regular banking channel. The rate of interest will be 15% per annum, and the loan will be paid in five annual installments, inclusive of interest. The effective tax rate of the company is 40%. It is to be assumed that the asset will be written off over a period of 5 years on a straight line basis. (a) Advise Welsh Limited about the method of acquiring the car. (b) What should annual lease rental to be charged by the leasing company to match the loan option? For your exercise use the following discount factors: Year 1 2 3 4 5 Discount rate 10% 0.91 0.83 0.75 0.68 0.62 15% 0.87 0.76 0.66 0.57 0.49 9% 0.92 0.84 0.77 0.71 0.65

Solution : Tutorial notes: Cost of capital after tax should be used for discounting purposes. The rate of interest 15% and the rate of tax is 40%. The effective rate of interest for calculation of discount factors would be 15% x 60% i.e. 9%. The loan installments will be calculated with 15% interest. Option 1: The asset is acquired on loan from bank @ 15% interest Computation of Installments : Suppose the loan installment is Rs. X per annum and the installment is paid at the beginning of the yr. We have the equation : 126,965 = X (1.00+0.87+0.76+0.66+0.57) which gives X = Rs. 32,892 Computation of interest in debt payments : As the installment of Rs. 32,892 per annum is inclusive of interest on loan as well and the interest is tax Year 0 1 2 3 4 Opening balance of Principal 126,965 94,073 75,292 53,694 28,856 Interest @ 15% 0 14,111 11,294 8,054 4,036 * Repayment of installment 32,892 32,892 32,892 32,892 32,892 Closing balance 94,073 75,292 53,694 28,856 0 *( 32,892--28,856 = 4,036) If the asset is obtained on loan, the interest paid and the depreciation on asset, both qualify for tax saving Total Tax annual Depreciati (dep.. + benefit @ Net Cash Pv factors Pv of cash Yr.end instalm. Interest on Inter.) 40% outflows at 9% flows 0 32,892 0 32,892 1.00 32,892 1 32,892 14,111 25,393 39,504 15,802 17,090 0.92 15,723 2 32,892 11,294 25,393 36,687 14,675 18,217 0.84 15,303 3 32,892 8,054 25,393 33,447 13,379 19,513 0.77 15,025 4 32,892 4,036 25,393 29,429 11,772 21,120 0.71 14,995 5 0 0 25,393 25,393 10,157 (10157) 0.65 (6602) Total present value 87,336 Option 2: The asset is acquired on lease: Computation of Lease rentals: Leasing interest is 10%. Lease rentals will be calculated at 10%. Suppose the lease rentals amount to Rs. X per annum. The rentals are paid at the beginning of yr. We have 126,965 = X (1.00+0.91+0.83+0.75+0.68) which gives X = Rs. 30,447. Lease rentals are tax deductible expenses. Depreciation is not considered as the asset does not belong to the assessee but to the lessor. Yr.end Lease rentals Tax shield @ 40% net cash outflows Pv factors at 9% Pv Rs. 0 30,447 0 30,447 1.00 30,447 1 30,447 12,179 18,268 0.92 16,807 2 30,447 12,179 18,268 0.84 15,345 3 30,447 12,179 18,268 0.77 14,067 4 30,447 12,179 18,268 0.71 12,970 5 0 12,179 (12179) 0.65 (7916) Total present value : Rs. 81,720 As the present value of cash outflow is lower in case of lease option, it is advised to acquire the asset on lease. (b) The lease rentals be X. The after tax lease rentals would be 0.60X. The present value will be 0.60X ( 1.00+0.91+0.83+0.75+0.68) = 87,336 (pv under loan option) This gives X = Rs. 34,906. Therefore the lease option would be Rs. 34,906 to match the loan option.

2007 Dec 4b

Titanic Instruments Ltd is in the business of manufacturing bearings. Some more product lines are being planned to be added to the existing system. The company has decided to acquire a machine costing Rs. 10.0 lacs having a useful life of 5 years with the salvage value of Rs. 2.0 lacs (consider short-term capital gain for the income tax). The full value of the machine can be financed by bank loan at the rate of 10% p.a. repayable in five equal installments falling due at the end of each year. Alternatively, the machine can be procured on a five years lease, year end lease rentals being Rs. 2.50 lacs p.a. The company follows Wdv method of depreciation at the rate of 25%. The company is in 30% tax bracket. Requirements : (i) What is the present value of cash outflow for each of these financial alternatives using the after-tax cost of debt? (ii) Which of the two alternatives is preferable? Solution : 1. If financed by bank, the loan of Rs. 10.0 lacs along with 10% interest has to be paid in five equal installments. How will you calculate the installments ? 2. Schedule of depreciation is to be prepared. Compute tax relief on depreciation. 3. Salvage value is not considered in computing the depreciation if Wdv is followed. 4. The value of machine at the end of year 5, comes to Rs. 2.37 if wdv method is followed while salvage value as given in the question is Rs. 2.0 lacs. Thus there is a capital loss of Rs. 37,000 at the end of year 5. This loss would save taxes of the value of 0.30 x 37,000 = Rs. 11,100. The present value of this loss would be 0.71 x 11,1000 = Rs.7,800. You must use discounting rate of 7% and not 10%. The question specifically mentions that after cost of debt should be used for discounting purposes. Suppose A is the annual installment paid for five years. The present value of these installments must be equal to Rs. 10.0 lacs which is cost of machine. Present value at 10% for five years would be as follows: Year 1 2 3 4 5 Df 0.91 0.83 0.75 0.68 0.62 3.79 The value of installment would be given by : 3.79 A = 10.0 giving A = 10 / 3.79 = Rs. 2.64 lacs After you have determined the installments, you have to separate the capital and revenue components of every installment. How you will do it? Schedule of depreciation on Wdv method at 25% : Year Opening balance Depreciation Closing balance Tax shield at 30% on depre. 1 10.00 2.50 7.50 0.75 2 7.50 1.88 5.63 0.56 3 5.63 1.41 4.22 0.42 4 4.22 1.05 3.16 0.32 5 3.16 0.79 2.37 0.24 Computation of interest component in installments: Year end Loan Interest at 10% Total due Installments paid 1 10.00 1.00 11.00 2.64 2 8.36 0.84 9.20 2.64 3 6.56 0.66 7.21 2.64 4 4.57 0.46 5.03 2.64

Balance 8.36 6.56 4.57 2.39

0.24 2.64 2.64 0.00 3.19 13.20 Evaluation of loan option : In this case interest as well as depreciation would qualify for tax deductible expenses. Tax relief on both the items would be 30%. What will be the discounting factor for this option ? The question specifically mentions that after tax discounting rate should be used. Year end Instalmt. Df Pv of cash outflow 1 2.64 0.93 1.49 2 2.64 0.87 1.60 3 2.64 0.82 1.65 4 2.64 0.76 1.67 5 2.64 0.71 1.66 8.06 Note : Since post tax analysis is being conducted, post tax cost of funds should be taken for discounting purposes. Discounting rate would be 10% less tax 30% i.e. 7%. Determination of Npv of the option : Pv of cash out flow 8.06 Less : Pv of salvage value (0.71 x 2.0) (1.42) Less : Pv of tax saving on capital loss (0.71 x 0.111) (0.08) Net present value of loan option 6.56 Evaluation of lease option : it is comparatively easy. The lease rent is Rs. 250,000 per annum. It is tax deductible expense. The effective lease rent after tax would be 250,000 less 30% i.e. Rs. 175,000. Lease is for five years. Pv can simply be computed. Year end 1 2 3 4 5 Pv factor 0.93 0.87 0.82 0.76 0.71 4.10 Pv of lease rental : 175,000 x 4.10 = Rs. 771,500. Npv of loan option (borrow and buy option) 6.56 Npv of lease option 7.71 Saving in loan option 1.15 Recommendation : In view of the saving of Rs. 1.15 lac, the loan option is recommended. 2008 June 2b Progressive Ltd. sells its to wholesale distributors. The management is worried over the liquidity and is exploring the methods of improving the cash flow by speeding up collection from debtors. The following table summarizes its turnover and profits for the last two years and comparable debtors level as at the end of last two years. The alternatives before the management are : (i) Offer a 2% discount to customers who settle within 10 days of invoicing. It is estimated that 50% of customers would take advantage of this offer. (ii) Seek the services of a factor, who will operate on a 'service only' basis for collecting payments from customers. Savings are expected to be Rs. 500,000 annually; also debtor days will come down to 45 days. Charges payable to the factor would be 1.5% of the turnover. Progressive Ltd. can borrow from bank at 15% per annum. Year 0 Turnover Profits
60,000 11,500

2.39

Tax relief Interest Depreciation 0.30 0.75 0.25 0.56 0.20 0.42 0.14 0.32 0.07 0.24

Total Net cash outflow 1.05 1.59 0.81 1.83 0.62 2.02 0.45 2.19 0.31 2.33

Year 1 Amounts in Rs.'000


80,000 15,000

Required : Analyze the costs and benefits of both alternatives and stat the preferred course of action.

Required : Analyze the costs and benefits of both alternatives and stat the preferred course of Debtors 8,000 13,000 action. Solution : Option 1 : Giving discount of 2% Turnover in year 1 (amount in Rs.'000) 80,000 Debtors at the end of year 1 13,000 Average collection period (13 / 80) x 365 59 days If 2% discount is given it will bring down the collection period for 50% of debtors Average collection period after discount (59/2 + 10/2) 34.5 days Revised value of debtors : 80,000 800 79,200 Average level of debtors 79,200 x 34.5/365 827 Cost of discount 2% of 50% of 80,000 800 Saving 27 Option 2: Availing factoring services Reduction in collection period : (59 45) 14 days Value of sales for reduction in days (14 x 80,000/ 365) 3,068 Saving in interest (3,068 x 15%) 460 Saving in administration cost 500 960 Service charge (1.5% of 80,00) 1,200 Net cost of factoring (1,200 960) 240 Recommendation : Factoring is not attractive. Discount policy is recommended.
2008 Dec 4b: AB Ltd. is considering to buy an equipment and it has two options. The cost of the equipment is Rs. 100,000. Option 1: to buy with borrowed funds at a cost of 18% p.a., repayable in five equal installments of Rs. 32,000. Option 2: to take the equipment on lease on an annual rental of Rs. 32,000. The salvage value of the equipment at the end of five-year period will be zero. The company uses straight line depreciation. Assume tax @ 40%. Which of the two options would you recommend? Discounting factors are : Yr.1 Yr.2 Yr.3 Yr.4 Yr.5 9% 0.917 0.842 0.772 0.708 0.65 11% 0.901 0.812 0.731 0.659 0.593 18% 0.847 0.718 0.609 0.516 0.437 3.127 Solution: What should be the discounting rate? Question is silent about. Cost of borrowed fund is 18%. Let us take after-tax rate as discounting rate. The after tax rate would be 18% ( 1 - 0.40) =10.8% say 11%. Evaluation of lease option is easier hence let us take it first. Evaluation of lease option Net present value of cash outflow : 32,000 (1 - 0.4) [0.901+0.812+0.731+0.659+0.593] = Rs. Evaluation of borrowed fund option: (interest rate :18%) fig. in Rs. Annual payment Interest Capital Dep. Total TaxNet outflow Pv 32,000 18,000 14,000 20,000 38,000 15,200 16,800 0.901 32,000 15,480 16,520 20,000 35,480 14,192 17,808 0.812 32,000 12,506 19,494 20,000 32,506 13,003 18,997 0.731 32,000 8,998 23,002 20,000 28,998 11,599 20,401 0.659 31,840 4,857 26,983 20,000 24,857 9,943 21,897 0.593 Rs. Comparison : Npv of lease option 70,963 Npv of borrowed fund option 69,913 Advantage of using lease option 1,050 Recommendation : Lease option is better (under after-tax discounting rate) by Rs. 1,050. Additional Knowledge :

70,963 Npv 15,137 14,460 13,887 13,444 12,985 69,913

Suppose discount rate is taken as before tax cost of borrowed fund of 18%; Following picture would emerge: NPv of lease option = 32,000 (1 - 0.4)[0.847+0.718+0.609+0.516+0.437) = Rs. 60,038 Evaluation of borrowed fund option: fig. in Rs. Annual payment Interest Capital Dep. Total TaxNet outflow Pv Npv 32,000 18,000 14,000 20,000 38,000 15,200 16,800 0.847 14,230 32,000 15,480 16,520 20,000 35,480 14,192 17,808 0.718 12,786 32,000 12,506 19,494 20,000 32,506 13,003 18,997 0.609 11,569 32,000 8,998 23,002 20,000 28,998 11,599 20,401 0.516 10,527 31,840 4,857 26,983 20,000 24,857 9,943 21,897 0.437 9,569 Rs. 58,681 Comparison : Npv of lease option 60,038 Npv of borrowed fund option 58,681 Advantage of using lease option 1,357 Recommendation : Lease option is better (under before-tax discounting rate) by Rs. 1,357.

s with no scrap value.

25% a year on a

arrangement, at an o finance the acquisition

of availment of both the

nterest on borrowing

6,000 per year. Thus

mmended.

mmended.

e collection is 90 days. annual cost of ales administering ched a factoring

o be paid upfront.

14% p.a. Should the

to pay 14% to the bank.

est rate of 13%.

equipment is eligible 1% of the equipment

ate tax rate of 35%, ase quotation i.e.

0.6355, 0.5674.

. The leasing company

ore be taken as Rs. of it. Where this factor

ctive life of the akhs at 125 interest

o be assumed that

ach year. option.

ngs on these two

Rs. 1,26,965. It has a life sing company desires a egular banking channel. nclusive of interest. The a period of 5 years on a

15% and the rate of tax The loan installments

ng to the assessee but

on lease.

aight line depreciation.

ake after-tax rate as

Chapter 5 : Working capital management


2001 Dec. 1b: The sales forecast for January to May, 2002 and the actual sales for November and December,2001 for Plysales Co. are given under : Nov.01 Dec.01 Janu.02 Feb.02 Mar.02 Apr.02 May.02 Actual sales in Rs. 80,000 70,000 Forecast sales in Rs. 80,000 100,000 80,000 100,000 90,000 20% of sales is in cash and rest is on credit, payment of which is realised in the third month. The following information are also available: (i) Amount of purchase is budgeted at 60% of the Sales Turnover of a month and paid in the third month of purchase. (ii) Variable expenses is 5% of turnover-time lag for payment half month. (iii) Commission of credit sales @ 5% is payable in the third month. (iv) Rent and other expenses amounting Rs. 3,000 paid every month. (v) Payment for purchase of fixed assets Rs. 50,000 in March,02. (vi) Payment for taxes in April,02 Rs. 20,000. (vii) There will be opening balance of cash of Rs. 25,000. You are required to prepare a Cash budget for the months from January to May 2002. Tutorial Notes: 1. Variable expenses payable @ 5% of turn over. The time lag is half month,. It implies that half of variable expenses is paid in the same month while next half is paid in next month. Working Notes: All figures in Rs. Solution : Computation of Cash and credit sales and commission: Nov.01 Dec.01 Janu.02 Feb.02 Mar.02 Apr.02 May.02 Sales 80,000 70,000 80,000 100,000 80,000 100,000 90,000 Cash sales 16,000 14,000 16,000 20,000 16,000 20,000 18,000 Credit sales 64,000 56,000 64,000 80,000 64,000 80,000 72,000 Commission payable* 3,200 2,800 3,200 4,000 3,200 Payment received 64,000 56,000 64,000 80,000 64,000 Purchases 48,000 42,000 48,000 60,000 48,000 60,000 54,000 Payment for purchases 48,000 42,000 48,000 60,000 48,000 Variable expenses 2.5% of current month and 2.5% of previous month sales. *5% of respective credit sales. Commission for Nov. is obtained in Jan. Plysales Ltd. Cash Budget for the period from January 02 to May 02 Janu.02 Feb.02 Mar.02 Apr.02 May.02 Receipts: Rs. Rs. Rs. Rs. Rs. Opening Balance 25,000 47,050 70,750 42,050 50,550 Cash sales 16,000 20,000 16,000 20,000 18,000 Collection from debtors 64,000 56,000 64,000 80,000 64,000 Total receipts 105,000 123,050 150,750 142,050 132,550 Payments : Payment to creditors 48,000 42,000 48,000 60,000 48,000 Variable expenses 3,750 4,500 4,500 4,500 4,750 Commission 3,200 2,800 3,200 4,000 3,200 Rent 3,000 3,000 3,000 3,000 3,000 Fixed assets 50,000 Taxes 20,000 Total payments 57,950 52,300 108,700 91,500 58,950 Closing Balance 47,050 70,750 42,050 50,550 73,600

Failure is success if we learn from it. 2001 Dec.3b X Ltd. currently has an annual turnover of Rs. 20 lakhs and an average collection period of 4 weeks. The company propose to introduce a more liberal credit policy which they hope will generate additional sales, as shown below : Proposed credit policy Increase in Increase % of default Collection period by in sales Rs. 1 2 weeks 200,000 2% 2 4 weeks 250,000 3% 3 6 weeks 350,000 5% 4 8 weeks 500,000 8% The selling price of the product is Rs. 10 and the variable cost per unit is Rs. 7. The current bad debt loss is 1% and the desired rate of return on investment is 20%. For the purpose of calculation, a year is to be taken to comprise of 52 weeks. Indicate which of the above policies you would recommend the company to adopt. Solution : Comparative statement of various credit policies Particulars Current Policy 1 Policy 2 Policy 3 Policy 4 Sales Rs. Lac 20.0 22.0 22.5 23.5 25.0 Contribution @ 30% 6.0 6.6 6.8 7.1 7.5 Bad debts % 1% 2% 3% 5% 6% Amount of bad debts Rs. Lac 0.2 0.4 0.7 1.2 1.5 Average collection period in weeks 4.0 6.0 8.0 10.0 12.0 Average debtors Rs. Lacs 1.5 2.5 3.5 4.5 5.8 Cost of debtors @ 20% 0.3 0.5 0.7 0.9 1.2 Contribution - bad debts -cost of deb. 5.5 5.7 5.4 5.0 4.8 The net benefit is the highest with policy 1 with credit period of 6 weeks. It is recommended for adoption. 2002 June 1b: Estimate the requirement of total capital of the following project with an estimated production of 250 tonne per annum of chemical X, presently imported and which can be entirely sold at the rate of its landed cost of Rs. 8,500 per tonne. You are also required to find out: (i) % of yield on investment (ii) % of profit on sales (iii) rate of cash generation per annum before tax. Details of the proposed project for expected production of 250 tonne are as under : (i) Investment (Rs.) Land 100,000 Building 800,000 Plant & machinery 1,200,000 (ii) Cost of production per annum (Rs. ) Imported raw material 650,000 Indigenous raw material 626,000 Salaries and wages 135,000 Repairs and maintenance 5% of plant cost and 2% on building Depreciation 7% on plant and 2.5% on building Admn. and other expenses 50000 Steam requirement per tonne @ Rs. 16 per tonne Power 6000 Packing drums (500 gms capacity) Rs. 30 each (iii) Working capital requirement : Imported Raw materials stock 6 months Indigenous raw materials stock 3 months Stock of finished goods 1 month Credit to customers 1 month Credit from suppliers (indigenous materials) 1 month Cash expenses 1 month

Solution :

bldk lkY;w'ku Hkstk ugha gS] oghs ls ys yhft;sA


2002June 2a CIKA Ltd. has an and of 350 days.

Solution :

bldk lkY;w'ku Hkstk ugha gS] oghs ls ys yhft;sA


2002 Dec 11 The Chief Financial Officer of Agra Oil Company has produced the following summary forecast profit statement and balance sheet of the company for the next twelve months: Rs. lacs Rs. lacs Summary of profit forecast Sales income 200,000 tins of oil at Rs. 600 per tin 1,200 Less : Variable cost 900 Less : Fixed cost 150 1,050 Profit 150 Summary of Balance sheet : Investment in fixed assets 1,500 Investment in Working capital Debtors 200 Stock 80 Cash 304 24 Less : Current liabilities (60) Total Investment (fixed assets + working capital) 1,744 Profit as return on investment : 8.6%. The directors are worried over the forecast low return on investment specially as a lot of the in in fixed assets will remain under-utilised. A detailed study shows that selling price cannot be increased nor the costs be reduced. So there is a dire need to increase the sales volume. Currently all sales are on credit and the company operates a very strict credit control procedure which has virtually eliminated bad debts. Because of this a number of potential customers have had to be refused and some new customers have taken their business elsewhere. The suggestion has been made that a relaxation of the credit policy could increase sales volume substantially specially if, the company were the company were to introduce a scheme whereby a 2% discount (at present no discount is given) were given on accounts paid within ten days and if the company is willing to accept 'riskier' customers, the sales volume would increase by 40%. Probably 65% of the customers would avail themselves cash discount and the average collection period of the remainder would be half of what it is at present. Bad debts would be the order of 2% to 6% of sales. Comment on whether the credit policy should be relaxed as suggested. Ignore taxation. State your assumptions, if any. Solution : Tutorial Notes 1. Following assumptions are made : (i) The variable cost, debtors and creditors move proportional to sales volume. (ii) Fixed cost remains same despite increase in sales volume. 2. The question says that if discount of 2% would be given, the sales would increase by 40% (over current sales of Rs. 1,200 lacs) and 65% of the customers would avail the discount facility. What should be the total discount in Rs. ? 3. The total discount would be 2% of 65% of Rs. 1,680 lacs i.e. Rs. 21.84 say Rs. 22 lacs. Show this as working note. 4. With proposed credit policy, the collection period would be half of what is present. What is the present collection period? Read the question again and find out.

5.How can you compute the investments required in debtors in proposed policy? Try to calculate before you see the solution. 6. Try to compute the total investment required in proposed policy. The investment will consist of fixed assets (given as Rs. 1,500 lacs) and current assets viz. Stock, Debtors and Cash. 7. If you take collection period in days, you may take year as 365 days. Present Proposed Sales in Rs. lacs (40% above 1,200) 1,200 1,680 Less Variable cost @ 75% of sales 900 1,260 (assumed to be proportional to sales) Contribution 300 420 Less : Fixed cost 150 150 (assumed to be constant) Profit 150 270 Less : discount 0 22 (2% of 65% of 1,680) Profit after discount 150 248 The bad debts are from 2% to 6%, the range of profit for proposed sales would be Bad debts to customers 2% 6% Profit before bad debts 248 248 Bad debts on sales (Rs. 1,680 lacs) 34 101 Profit after discount 214 147 Thus for bad debts range of 2% to 6%, the corresponding profit range will be Rs. 214 lacs to Rs. 147 lacs. Computation of Investment required : 1. The investment in debtors would be in two types of debtors (i) those who avail the options i.e. 65% of credit sales (in this question, all sales are credit sales) and (ii) those who receive cash discount of 2% and pay within 10 days. 65% avail discount and pay within 10 days, the investment required would be : 65% of Rs. 1,680 lacs x (10 / 365) = Rs. 30 lacs For remaining debtors i.e. 35%, the collection period would be half of what is at present. The present collection period is indirectly given. Total sales at present is Rs. 1,200 lacs per annum or Rs. 100 lacs per month and total debtors are Rs. 200 lacs. Thus present collection period would be two months. The collection period with proposed credit policy would be one month. 35% of debtors take one month to pay. The investment in them would be : 35% of Rs. 1,680 lacs x 30 / 365 Or 35% of Rs. 1,680 lacs x 1/12, both are correct. Investment in debtors : 35% x Rs. 1,680 lacs x 1/12 = Rs. 49 lacs Comparative statement of present and proposed policy Present Proposed (figures in Rs. lacs) Sales 1,200 1,680 Investment in fixed assets 1,500 1,500 Investment in current assets Stock Stock 80 112 (proportional to sales) Debtors 200 79 (as computed above) Cash 24 34 (proportional to sales) Less : Current liabilities (60) (84) (proportional to sales) Total Investment 1,744 1,641 Return with profit of Rs. 214 lacs (with bad debts of 2%) 13% (214 / 1,641) Return with profit of Rs. 147 lacs (with bad debts of 6%) 9% (147 / 1,641) Recommendation : Current return on investment is 8.6% (150 / 1,744). The return with 2% bad debts is 13% and that with 6% is 9%. In view of more return, the proposed credit policy is recommended to be adopted. 2004 Dec 8 (b)

Hindustan Products Ltd. is considering the revision of its credit policy with a view to increasing its sales and profits. Currently all its sales are on credit and the customers are given one month's time to settle dues. It has a contribution of 40% on sales and it can raise additional funds at a cost of 20% per annum. The marketing director of the company has given the following options with draft estimates for consideration : Particulars Current Option-1 Option-2 Option-3 Sales Rs. Lac 200 210 220 250 Credit period months 1 1.5 2 3 Bad Debts as % of sales 2 2.5 3 5 Cost of administration Rs. Lac 1.2 1.3 1.5 3 Advise the company to take the right decision. Solution : fig. in Rs. Lacs Particulars Current Option-1 Option-2 Option-3 Sales Rs. Lac 200 210 220 250 Credit period months 1.0 1.5 2.0 3.0 Debtors (sales for credit period) 16.7 26.3 36.7 62.5 Cost of debtors @ 20% 3.3 5.3 7.3 12.5 Credit administration cost 1.2 1.3 1.5 3.0 Bad Debts as % of sales 2.0 2.5 3.0 5.0 Bad Debts 4.0 5.3 6.6 12.5 Total cost (debtors + admin.+bad debts) 8.5 11.8 15.4 28.0 Contribution as 40% of sales 80.0 84.0 88.0 100.0 Net Gain 71.5 72.2 72.6 72.0 Option 2 provides highest gain. It is recommended for adoption. 2005 Dec 4a: The Marketing manager. Provision for bad debt?

bldk lkY;w'ku tks Hkstk gS] oghs ys yhft;sA


2005 Dec 4b: ABC company currently sells on terms 'net 45'. The company has sales of Rs. 3.75 million a year with 80% being the credit sales. At present, the average collection period is 60 days. The company is now considering offering term ' 2/10 net 45'. It is expected that the new credit terms will increase current credit sales by 1/3 rd. The company now expects that 60% of the credit sales will be on discount and average collection period will be reduced to 30 days. The average selling price of the company's product is Rs. 100 unit, and variable cost per unit works out to be Rs. 85. The company is subject to tax rate of 40% and its before tax borrowing for working capital is 18%, Should the company change its credit terms to '2/10 net 45 days'? Support your answer by calculating the expected change in net profit. (Assume 360 days in a year). Solution : Tutorial notes : Discounts are generally given to speed up the payment of debts. Suppose a firm has an annual sales of Rs. 300 lacs. The monthly sales would be Rs. 25 lacs and if the collection period is say 2 months, the investment in debtors would be Rs. 50 lacs. Suppose 3% discount is proposed to be given for cash discount who pay cash immediately. Suppose further that 50% of debtors decides to avail the discount offer, this means that Rs. 25.0 lacs (50% of total debtors) would be released on account of discount offer in every two months. The discount paid to debtors for this release would be 3% of Rs. 25 lacs i.e. Rs. 75,000 every two months or say Rs. 4.5 lacs per annum. If the cost of Rs.25 lacs for one year is greater than Rs. 4.5 lacs, the discount should be given.

Suppose further that ROI of the firm is 20%, it is obvious that the firm will gain Rs. 25 lacs for the whole year and the return would be 20% of Rs. 25 lacs i.e. Rs. 5 lacs. As the discount to be given is Rs. 4.5 lacs, the saving of Rs. 50,000 (5.0 lac less 4.5 lac) justifies the new policy. The credit terms may be expressed as ''2/15 net 45''. This means that a 2% discount will be granted if the customer pays within 15 days, if he does not avail the offer he must make payment within 45 days. Debtors can be calculated on sales or on cost of sales. If cost of sales is given as in the present case, it is better to calculate debtors on cost of sales rather than on sales. Illustration : A company has annual sales amounting to Rs. 10.0 lacs for which it grants a credit of 60 days. At present no discount is offered to customers. The company is considering a plan to offer a discount '3/15 net 60'. The offer of discount is expected to bring the total credit period from 60 days to 45 days and 50% of the customers (in value) are likely to avail the discount facility. The selling price of product is Rs.15 and its variable cost is Rs. 12. Please advise the company which to resort to discount facility if the rate of return is 20% and a month is equal to 30 days. Annual credit sales Rs. 10.0 lacs Present investment in receivables : Collection period is 60 days and cost of sales is 80% (12/15) of sales. The investment is considered in cost of sales and not in sales. Present investment would be cost of sales for 60 days collection period and expected investment would be cost of sales for 45 days period. Present investment in receivables : 10.0 x (60/360) x 80% Rs. 1.34 lacs Expected investment in receivables : 10.0 x (45/360) x 80% Rs. 1.00 ;acs Saving in investment Rs. 0.34 lacs Return on saved investment @ 20% per annum (0.34 x 0.2) Rs. 6,800 Cash discount to be given : 3% x 50% x Rs. 10.0 lacs Rs. 15,000 The company gives Rs. 15,000 as discount and gains Rs. 6,800 on account of saving, the policy is not recommended. In the present case there will be two gains (i) increased contribution from increase in sales and (ii) Return on savings in investment. Total gains will be compared with the discount proposed to be given.

Discount to be given 2% of 60% of Rs. 40.0 lacs Computation of return due to saving in investment : Present investment is for a period of 60 days 30 x (60/360) x 85% Proposed investment is for a period of 30 days 40 x (30/360) x 85% Saving in investment Return due to saving in investment (@18%) 18% of Rs. 125,000 Computation of increased sales Contribution from present sales (15% of Rs. 30 lacs) Contribution from increased sales (15% of Rs. 40 lacs) Extra contribution due to discount policy Evaluation of Discount policy : Extra contribution Return on saving Total gain Less : Discount to be given Overall gain 2005 Dec 5b:

48,000 Rs. 425,000 300,000 125,000 22,500 450,000 600,000 150,000 150,000 22,500 172,500 48,000 124,500

bldk lkY;w'ku tks Hkstk gS] oghs ys yhft;sA

2006 Dec 6c: The annual turnover of VIBGYOR Limited is Rs. 12 million of which 80% is on credit. Debtors are allowed one month to clear off the dues. AllBank Factors Ltd. (a factor company) is willing to advance 90% of the bill raise on credit for a fee of 2% a month plus a commission of 3% on the total amount of debts. Vibgyor ltd. as a result of this arrangement, is likely to save Rs. 43,200 annually in management costs and avoid bad debts at 1% on the credit sales. A scheduled bank has come forward to make an advance equal to 90% of the debts at an interest rate of 12% per annum. However, the processing fee will be 2% on the debt. Should the company avail of the factoring service or offer of the bank? Give reasons. Solution ; Tutorial Notes

Factoring
The factors are organizations which assist the company in credit collection and management. A factor provides three basic services to clients : (i) Maintaining the credit sales ledgers (ii) credit collection and protection against bad debts (iii) Providing financial assistance to the company by providing advance payments against receivables/debtors. There are two types of costs involved in factoring (i) commission or service fee (ii) interest on advance granted by the factors to the firm. To properly understand the concept of factoring let us take Illustration from (CWA Final June 00) A small firm has a total credit sales of Rs. 80 lakhs and its average collection period is 80 days and bad debt losses are 1%. The variable and avoidable cost of credit administration is Rs. 120,000. A factor is ready to buy the firm's receivables at 2% commission and will pay advance against the receivables to the firm at an interest rate of 18% after withholding 10% as reserve. What should the firm do ? Rs. lacs Average debtors or receivables (80 / 360) x 80 days 17.78 The advance which the factor will pay will be average debtors less factoring commission, reserves and interest on advance. Factoring commission will be 2% on the total receivables. The factor will also withhold 10% as reserve. The Advance given by factor : Total receivables 17.78 Less : Factoring commission @ 2% 0.36 Less : Reserve as 10% 1.78 2.13 Advance to paid by the factor 15.65 At the time of giving advance, the factor will deduct the interest to be paid to him at 18%. Advance to be paid 15.85 Less : Interest @ 18% for 80 days on Rs. 15.85 lacs 0.63 Net advance to be paid to the firm 15.22 Let us calculate effective cost of factoring to the firm : The firm saves the following costs : Cost of credit administrative set up 1.20 Cost of bad debt losses 0.80 Total saving per annum 2.00 The firm incurs the following costs : Factoring commission 2% of total debts 1.60 Interest charges (0.63 x 360) / 80 2.84 Total cost incurred per annum 4.44 Net cost of factoring : 4.44 2.00 = Rs. 2.44 lacs per annum. This means that the firm has to pay Rs. 2.44 lacs against a deposit of Rs. 15.22 lacs received from the factor. The effective cost of factoring is ( 2.44 / 15.22) The solution to present problem goes as follows: 16.0%

The solution goes as follows:

1. Compute the cost of factoring and compare it with the cost of bank advances. Lesser of the two should be recommended. 2. Cost of factoring per month Cost of fee at 2% of monthly sales Commission at 3% on monthly sales Saving due to factoring 1% saving of bad debts on monthly sales Net cost of factoring per month You should try to fill the above and determine the cost of factoring. 3. Question is silent about the processing fee of bank advance. In general processing fee is taken one time when the loan formalities are processed. The cost is calculated on per month basis. Hence processing will be added on monthly basis. Cost of bank advances : Interest per month on 90% of 80% of Rs. 120 lacs Add: Processing fee at 2% of monthly sales of Rs. 8.0 lacs Add: Bad debts which cannot be avoided (1% of Rs. 8.0 lacs) Note : Processing fee will be applicable on first year only. In computation for subsequent years, the processing fee will not figure. Add : Less: Less: Please try to solve this on the above lines and compare your solution with the one given below. Total sales : Rs. 120 lacs; Credit sales : 80% of 120 i.e. Rs. 96 lacs. Monthly credit sales : Rs. 8.0 lacs. 2. Cost of factoring per month Cost of fee at 2% of monthly sales 2% of 90% of Rs. 8.0 lacs 14,400 Add : Commission at 3% on monthly sales 3% of Rs. 8.0 lacs 24,000 38,400 Less: Saving due to factoring (43,200 / 12) (3,600) Less: 1% saving of bad debts on monthly sales (8,000) (11,600) Net cost of factoring per month 26,800 Cost of bank advances : Interest per month on 90% of 80% of Rs. 120 lacs 7,200 Add: Processing fee at 2% of monthly sales of Rs. 8.0 lacs 16,000 Add: Bad debts which cannot be avoided (1% of Rs. 8.0 lacs) 8,000 31,200 Comment : Cost of factoring is Rs. 26,800 per month while cost of bank advances is Rs. 31,200. The company should avail of factoring service. Note : Processing fee will be applicable on first year only. In computation for subsequent years, the processing fee will not figure. For subsequent years, cost of factoring would be Rs. 26,800 and cost of bank advances would be Rs. 15,200. If the arrangement is going to last for longer duration, the bank advances would be beneficial for the company. 2007 June [7b] VIBGYOR Ltd. has sales of Rs. 250 million a year with 80% being credit sales. The present credit terms of the company are '2/15 net 45'. At present the average collection period is 30 days. The proportion of sales on which customers currently take discount is 0.50. Vibgyor Ltd. is considering relaxing its discount terms to '3/15 net 45'. Such a relaxation is expected to increase current credit sales by Rs. 10 million and reduce the average collection period to 27 days and increase the proportion of discount sales to 0.60. The average selling price of product is Rs. 1,000 per unit and variable cost per unit is Rs. 800. The company is subjected to a tax rate of 40% and its before tax, rate of borrowing for working capital is 12%. Should the company change its credit terms to '3/15 net 45'? Support your answer by calculating the expected change in net profit. (assume 360 days in a year). Solution : Tutorial Notes

1. Compute the increase in contribution due to increased sales. This is first benefit of the new policy. 2. Due to new policy, the collection period would come down to 27 days from current period of 30 days. This will reduce the investment in debtors. The reduction will save @ 12% per annum. The saving will be taxed at 40%. This saving after tax is the second benefit. 3. Due to new policy, the discount of 2% will be increased to 3%. The increase in discount given, would be the loss of new policy Additional contribution due to new policy : 20% of Rs. 10 million i.e Rs. 2.0 million Current Proposed Sales 200.00 210.00 Collection period in days 30.00 27.00 Debtors 200/30 and 210/27) 16.67 15.75 0.917 Reduction in debtors due to change in discount rate = 16.67 15.75 = 0.92 This investment would result in saving at the rate of 12%. The saving will be subject to tax rate of 40%. Saving due to reduction in investment 12% of Rs. 0.92 million 0.1104 Rs. million Less : Tax on saving at the rate of 40% 0.0442 Rs. million Net saving after tax 0.0662 Rs. million Now let us compute the increase in discount cost and compare it with net saving due to reduction in investment. At present 50% customers (debtors) take advantage of 2% discount. Total sales is Rs. 250 million. 80% of total sales i.e. Rs. 200 millions is credit sales. Half of credit sales i.e. Rs. 100 million take advantage of discount policy. Thus discount given would be 2% of Rs.100 million i.e. Rs.2 million With the introduction of new discount policy, the credit sales would increase by Rs. 10 million. Thus the credit sales would be Rs. 200 million plus Rs. 10 million i.e Rs. 210 million. 60% of credits sales will avail the discount (3%) offer. The discount given as per new policy would be 3% of 60% of Rs.210 million i.e. Rs. 3.78 million You should present the above narration as given below Rs. million Current discount given 2% of 50% of 80% of Rs. 250 million 2.00 Discount proposed 3% of 60% of Rs. 210 million 3.78 Extra discount given due to new policy 1.78 Extra contribution due to discount policy 2.000 Add : Saving due to new policy (net) 0.066 2.066 Less : Additional discount to be given 1.780 Net benefit due to new policy 0.286 Comment : In view of the additional benefit of Rs. 0.286 million, the new policy is recommended.
2008 CWA Final Dec. The Hyundai Instrument Corporation is trying to determine the effect of its inventory turnover ratio and days sales outstanding (dso) on its cash flow cycle. The Hyundai corporation's sales last year (all on credit) were Rs. 150,000 and it earned a net profit of 6%. Its inventory turnover ratio was 5 and DSO was 36.5 days. The firm had fixed assets totaling Rs. 35,000 and its payable period is 40 days. Calculate : (i) Cash Conversion Cycle (ii) Total assets turnover and ROA, if it holds negligible amounts of cash and marketable securities and (iii) Cash conversion cycle, Total asset turnover and Return on Assets, if its Inventory Turnover can be raised to 7.3. Solution : Cash Conversion Cycle = Inventory Conversion period + Receivable collection period less Payable Deferral period. Cash Conversion Cycle = 365 / 5 + 36.5 - 40 = 69.5 days. Total Assets Turnover = Sales / Total Assets. Sales has been given as Rs. 150,000 but you have to compute the Total Assets. Think how can you calculate the Total Assets.

Total Assets will consist of Inventory, Receivables and Fixed assets. Inventory turnover is 5 which means sales are 5 times the inventory. The sales are Rs. 150,000 meaning that the inventory must be 150,000 / 5 = Rs. 30,000. Inventory can also be calculated on the basis of Cost of goods sold but the question is silent about it, hence it can be safely assumed that inventory turnover ratio is related to Sales and not to Cost of goods sold. You can write a note in this respect in your answer. Receivable are collected in every 36.5 days. The year is of 365 days and yearly sales are Rs. 150,000. The DSO is 36.5 days which means that at any point, sales of 36.5 days are outstanding which constitute the receivables. Thus receivables must be 150,000 / 365 x 36.5 = Rs. 15,000. The Fixed assets are given as Rs. 35,000. Thus total assets are Rs. 30,000 + Rs. 15,000 + Rs. 35,000 = Rs. 80,000. The sales are known. You can know the Total Assets Turnover. Total Assets = Inventory + Receivables + Fixed assets Total Assets = Rs. 150,000 /5 + (150,000/365) x 36.5 + Rs. 35,000 = Rs. 80,000 Total Assets = 150,000 / 80,000 = 1.875 ROA stands for Return on Asset. The return is net profit i.e. 6% of sales. The return works out to Rs. 9,000 and total assets as computed are Rs. 80,000. Return on assets must be 9,000 / 80,000 = 11.25%. (iii) The third part is just repetition of second part. Inventory turnover is 7.3 which means sales are 7.3 times the inventory. The inventory must be Rs. 150,000/7.3 = Rs. 20,548. Other assets viz. receivables and fixed assets are same. Total assets are Rs. 20,548 + Rs. 15,000 + Rs. 35,000 = Rs. 70,548. The ROA = 9,000 / 70,548 = 12.7%. Cash conversion cycle = 365/73 + 36.5 - 40 = 46.5 days. Total assets turnover = 150,000 / 70,548 = 2.12 and ROA = 9,000 / 70,548 = 12.7%.

doption.

sales outstanding earned a net profit 000 and its payable

sh and marketable er can be raised to

e the Total Assets.

les are 5 times the y can also be med that inventory nswer.

DSO is 36.5 days eivables must be

. 80,000. The sales

and total assets as

the inventory. The otal assets are Rs.

Chapter 6 : Dividend and Retention policies.


2002 June 5b: The following are the data will be at 6% interest. Answer : Take the same solution. 2003 June 12b (i) Young limited last paid a dividend of Rs. 2 per share. Its earnings and dividends are expected to grow @ 8% p.a. The beta of the company is 1.3. If the risk free return is 6% and the return from market portfolio is 10% p.a., what is the price per share of the equity stock? Solution : Tutorial notes
Cost of equity capital = (dividend/ market price) + growth rate

The above formula can also be written as : Market price = Dividend (expected) / (Rate of return Growth rate) The company paid Rs. 2 as dividend, the expected dividend would be 8% above the current dividend i.e. Rs. 2 x 1.08. The rate of return or cost of capital for equity share both denote the same meaning. The rate of return can be determined from CAPM. You should solve this problem on the following lines : 1. Find the cost of capital or rate of return from CAPM. 2. This rate of return is to be used in Gordon model or Gordon Growth formula. Determine the share price. 3. Frame your solution and compare with the following : Required rate of return = Risk free return + Beta x ( Market return risk free return) = 6% + 1.3 ( 10% 6%) = 11.2%. Market price = Rs. 2 x 1.08 / (11.2% 8%) which gives Market price = Rs. 67.50. Additional information : (ii) What will be the effect on the price per share of Young limited if the following changes take place together? There is a decrease in the inflation premium by 2% per annum. The expected growth rate increases by 1% per annum. Market price = 2 x 1.09 / (11.2% 9%) = Rs. 99.1. 2004 Dec 3 : The following data relate to Steady Limited: Year ending 31st March 2000 2001 2002 2003 2004 Net Earnings per share 28 27 27 26 25 Net dividend per share 16.8 17.6 18.0 19.0 20.0 There are 10 lacs equity shares issued and majority of these shares are owned by private investors. There is no debt in capital structure. The company has been experiencing difficult trading conditions over the past few years. In the current year, net earnings are likely to be Rs. 2.0 crore, which will be just sufficient to pay a maintained dividend of Rs. 20 per share. Required : Comment on Company's (a) Dividend policy between 1999 2000 and 2003 2004 and (b) on its possible consequences for earnings. Solution : Year ending 31st March 2000 2001 2002 2003 2004 Net Earnings per share 28 27 27 26 25 Net dividend per share 16.8 17.6 18.0 19.0 20.0 Dividend payout ratio 60% 65% 67% 73% 80% (dividend / earning) Total earning in Rs. lacs 560 540 540 520 500 The company is paying dividends at a growing rate even when the net earnings are decreasing year by year. Dividend payout ratio has increased from 60% in the year 2000 to 80% in year 2004.

The company does not seem to have any expansion plans as indicated by growing pay out ratios. The reducing proportion of retain earnings year by year is not a good sign with respect to growth and expansion plan of the company. 2004 Dec 5b: Relyonus Ltd has manufacturing subsidiaries in three overseas countries : Subsidiary Country Corporate tax Proposed net dividend in Rs.'000 1 Mars 40% 1,000 2 Jupiter 35% 1,600 3 Venus 20% 8,000 The Indian corporate tax is 30%. There are no taxes in the tax heaven country. Bilateral tax treaties exist between India and the countries where each of the subsidiaries is located, which allow a tax credit against Indian Corporate tax liability up to maximum of the Indian tax liability. This tax credit may be assumed to be available even when dividends are channeled via a tax heaven. Indian corporation taxation on overseas earnings may be assumed to be based upon the total dividends remitted to India (grossed up by one minus the relevant national tax rate(s) for each overseas country. Required : Evaluate whether or not Relyonus would benefit from using a tax heaven holding company through which dividends would be channeled. Solution : The language of the question is hard. You have to read it twice for proper understanding. You should revise the bilateral tax treaty concept before you solve this question. Country Mars Jupiter Venus Total Net dividend (figure in Rs.'000) 1,000 1,600 8,000 Tax rate 40% 35% 20% Gross Dividend (Net dividend / ( 1 tax rate) 1,667 2,462 10,000 14,129 Local tax (Gross dividend Net dividend) 667 862 2,000 3,529 Indian tax liability @ 30% of Gross dividend 500 738 3,000 4,238 Indian tax to be paid Nil Nil 1,000 From the above, it is clear that total grossed up dividend is Rs. 141.29 lacs on which Indian tax liability @ 30% comes to Rs. 42.38 lacs. The local taxes paid by the company are Rs. 35.29 lacs for which tax credit is available. Thus Indian tax liability is 42.38 minus 35.29 = Rs.7.09 lacs whereas Indian tax to be paid is Rs. 10.0 lacs as computed. This gives tax saving of Rs. 2.91 lacs (10.0 7.09), 2006 June 8c Multinational company has paid out 75% of its earnings as dividend in the current period. The company's present ROE is 30%, and it expects to maintain both ROE and dividend payout ratio in the long run. The company's share is currently trading at Rs. 150. Do you think that the share is overvalued? Give reasons. You may consider the following information : Current EPS : Rs. 10; Share beta : 0.75; Risk free rate : 6% and Market return : 8%. Solution : Tutorial notes : 1. Whenever beta, risk free rate and market rate are given, CAPM should come to your mind. 2. Growth rate you have to determine to compute the theoretical market price. Growth involves that portion of ROE which is retained in the business. The company pays 75% of ROE as dividend and that leaves only 25% of ROE for growth. The ROE is 30%, hence the growth would be 25% of 30% i.e. 7.5%. The required rate of return = risk free rate + beta ( market return risk free rate) = 6% + 0.75 x 8% = 12%. Note market premium = market return risk free rate. Value of share can be estimated as 12% = [ Dividend x ( 1 + Growth) / Market price ] + Growth Dividend is 75% of EPS i.e. 75% of Rs. 10 = Rs. 7.5. The equation becomes : 12% = [ 7.5 (1.075) / Market price] + 7.5% which gives price as Rs. 179.

The current share price is Rs. 150 which is less than the intrinsic or theoretical value of Rs. 176. The share is undervalued. 2008 June 7b The following information pertains to RICO Ltd. (amount in Rs. lakhs) Net Profit 60 Outstanding 12% preference shares 200 ROI 20% Number of shares outstanding 600,000 Equity capital rate 16% Required : (i) What should be dividend pay out ratio so as to keep the share price at Rs. 41.25 using Walter model? and (ii) What is the optimum dividend pay out ratio according to Walter model? Solution : (i) Walter model is given by P = [ D + (E D) ( r/ k)] / k where P = Market price per share; E = Earnings per share ; D = Dividend per share and K is cost of equity. (ii) According to Walter's model when the return on investment is more than the cost of equity capital, the price per share increases as the dividend pay-out ratio decreases. You can solve this problem on the following lines : 1. Find out the eps. 2. Use Walter model to find out the dividend pay out ratio. (ii) Answer to this part is easy. The ROI (20%) is more than cost of equity (16%). Hence all earnings should be invested in the company and dividend pay out ratio should be nil. Earnings per share is computed as follows : Net Profit 60 Rs. lacs Less Preference dividend 24 Rs. lacs Earnings for share holders 36 Rs. lacs No. of outstanding shares in lacs 6 Rs. lacs Earnings per share = 36 / 6 = Rs. 6 per share, (i) Walter model is given by P = [ D + (E D) ( r/ k)] / k where Putting the values assuming dividend payout ratio is say d. D = 6d, r = 20%, K = 16% and E =6. Dividend payout ratio is 60% to maintain the share price at Rs. 41.25.

Chapter 7: Financial analysis and planning


2001 Dec 3a The fixed assets and equities of ABC Ltd. are given below at the beginning and at the end of the year 2000-2001. As at 1-4-00 As at 31-3-01 Rs. Rs. Plant (less Depreciation) 63,500 142,500 Investment 132,000 290,000 Bonds Payable 250,000 70,000 Capital 400,000 400,000 Retained Earnings 238,000 410,500 The following additional information are available: (i) Dividend amounting to Rs. 37,500 was paid. (ii) The net income included Rs. 13,000 as profit on sale of equipment. There has been an increase of Rs. 93,000 in value of Gross Plant even though equipment worth Rs.29,000 with a net book value of Rs. 19,000 was disposed off. From the above prepare a Statement of Sources and Application of funds, Tutorial notes: 1. Complete balance sheet has not been given. You must check whether the assets and liabilities tally. If they don't, you should figure out what and why is the difference. 2. The Funds flow statement should be prepared while going through the problem itself. In the question you may find many items which can directly be transferred to funds flow statement. For example, in this question, the dividend paid can directly be written as application of funds. It is clear in the first reading of the question that Investment of Rs.158,000 is application of funds while the payment of Rs. 180,000 on the account of bonds payable is also application of funds. 3. The plant account should be prepared either taking gross figures or net figures. The increase in value of gross block by Rs. 93,000 after selling the equipment worth Rs. 29,000, states that there must have been a purchase of Rs. 122,000 ( Rs. 93,000 + Rs. 29,000) during the year. The plant account should be prepared and you should think why and what is balancing figure of the account. 4. The equipment was sold for what price ? Read the question again and find out. Additional information says that equipment of net value of Rs. 19,000 was disposed off during the year. Another information says that the net income included Rs. 13,000 as profit on sale of plant. Thus it can be concluded that the equipment was sold for Rs.32,000 ( Rs.19,000 + Rs. 13,000) during the year. This is source of funds. Solution: 1. Comparison of Assets and Liabilities on 1.4.00 and on 31.3.01. As at 1-4-00 As at 31-3-01 Rs. Rs. Plant (less Depreciation) 63,500 142,500 Investment 132,000 290,000 195,500 432,500 Bonds Payable 250,000 70,000 Capital 400,000 400,000 Retained Earnings 238,000 410,500 888,000 880,500 Difference 692,500 448,000 It is obvious that the difference is due to working capital. The working capital declines from Rs. 692,500 to Rs. 448,000. The decline of Rs. 244,500 is source of funds. 2. Plant and machinery account in Rs. Opening balance 63,500 Plant sold a/c 19,000 Purchases 122,000 Depreciation a/c 24,000 Closing balance 142,500

185,500 185,500 3. Computation of funds from operations ; Profit for the year ( retained earnings difference) (410,500 -- 238,000) 172,500 Add : Dividend paid during the year 37,500 Less: profit on sale of equipment 13,000 Funds from operations 197,000 Funds flow statement for the year ended 31.3.01 Sources Rs. Application Rs. Funds from operations 197,000 Purchase of plant 122,000 Add: Depreciation 24,000 221,000 Addition to Investment 158,000 Sale of equipment 32,000 Repayment of Bonds 180,000 Decrease in Working capital 244,500 Dividend paid 37,500 497,500 497,500 2002 Dec 13 The following are the balance sheets of Tamilnadu Industrial Corporation Ltd. Liabilities Rs. Rs. Fixed assets Rs. Rs. Paid up Capital 165,000 202,500 Property 111,375 108,187 Reserves 22,500 30,000 Machinery 84,713 94,650 Profit and loss a/c 29,768 30,915 Goodwill 7,500 Current liabilities Current assets Creditors 29,250 31,245 Stock 82,500 69,000 Bills payable 25,342 8,250 Debtors 64,620 52,073 Bank Overdraft 45,000 Cash and Bank 1,125 8,250 Provision for taxation 30,000 37,500 Pre-payments 2,527 750 346,860 340,410 346,860 340,410 During the current year ended 31st March, a dividend of Rs. 10,500 was paid and the assets of another company were purchased for Rs. 37,500 payable in fully paid up shares, Such assets purchased are : Stock : Rs. 16,230 ; Machinery : Rs. 13,770 and Goodwill : Rs. 7,500 In addition, plant at a cost of Rs. 4,237 was purchased during the year. The following depreciation was written off during the year : On property : Rs. 3,188 and On machinery : Rs. 8,070 Income tax during the year amounting to Rs. 21,578 was charged to provision for taxation. Net profit for the year before tax was Rs. 57,225. (a) Prepare statement of changes in financial position on total resources basis (ii) Prepare a schedule of changes in working capital. Solution : Tutorial Notes : 1. Prepare the schedule of changes in working capital first because it is the easier of the two. Current assets Previous Current Current liabilities Previous Current Stock 82,500 69,000 Creditors 29,250 31,245 Debtors 64,620 52,073 Bills payable 25,342 8,250 Cash and Bank 1,125 8,250 Bank Overdraft 45,000 Pre-payments 2,527 750 Provision for taxation 30,000 37,500 150,772 130,073 129,592 76,995 Working capital : = current assets current liabilities 21,180 53,078 Increase in working capital = 53,078 21,180 = Rs. 31,898. This is application of funds. 2. Determine the funds from operation. Begin from balance on profit and loss a/c of current year, make adjustments as given in the question and find the funds from operation. Determination of funds from operations: Profit and loss a/c for the current year 30,915 Add : Depreciation Property 3,188 Machinery 8,070 11,258 Dividends 19,500

7,500 69,173 Less: Profit and loss a/c for the previous year (29,768) Funds from business operations 39,405 Sources of Funds Application of Funds Funds from business operations 39,405 Purchase of Plant (for cash) Issue of shares 37,500 Purchase of Machinery (for shares) Goodwill (for shares) Payment of dividend Increase in working capital 76,905

Transfer to reserve (30,000 22,500)

4,237 13,770 7,500 19,500 31,898 76,905

2003 June 12b(ii) ;s Ldsuj isa v/kwjk Nik gS bls lq/kkfj;sA Solution : Please see chapter 6 2003 June 12b(i) 2003 June 14 a(i) Beauty Ltd. has an excess cash of Rs. 16,00,000 which it wants to invest in short-term securities. Expenses relating to investment will be Rs. 40,000. The securities invested will have an annual yield of 8%. The company seeks your advice as to the period of investment so as to earn a pre-tax income of 4%. (ii) Also find the minimum period for the company to break-even the investment expenditure. Ignore time value of money. Solution : Company wants to earn pre-tax income of 4% on Rs. 16.0 lacs i.e. it wants to earn Rs. 64,000. The issue expenses are Rs. 40,000. Thus the company should earn Rs. 64,000 + Rs. 40,000 = Rs. 104,000. What is the time required to earn Rs. 104,000 if Rs. 16.0 lacs are invested at the rate of 8%? The company can earn Rs. 128,000 (8% of Rs. 16.0 lacs) in a year, it will require 104,000 / 128,000 = 0.8125 year or 9.75 months to earn Rs. 104,000. (ii) to break even, the company should earn the issue expenses of Rs. 40,000. The company earns Rs. 128,000 in a year, it will require 40,000 / 128,000 = 0.3125 year or 3.75 months to earn Rs. 40,000. 2003 June 15(a) : Company X and Company Y belong to dividend tax. Solution : Take the same answer. 2003 June 15b Trust Ltd. is deciding whether to payout Rs. 480,000 in excess cash in the form of dividend or go for share repurchase. Current earnings are Rs. 2.40 per share and the stock sells for Rs. 24. The market value balance sheet currently is as follows Balance sheet (in Rs. 000) Equity 2,400 Assets other than cash 2,560 Debt 640 Cash 480 3,040 3,040 Evaluate the two alternatives in terms of the effect on the price per share of the stock, the EPS and the P/E ratio. Whether alternative do you recommend? Give reasons. Solution : Tutorial notes: Read the question closely and answer the following points: 1. How many shares are outstanding ? 2. If the company decides to repurchase shares, how many can it repurchase? 3. What will be the EPS after buyback ? 4. What will be price of share after the buyback?

If the company decides to pay dividend and if it is assumed that price earning ratio remains same which means that the earning per share will also be the same before and after declaration of dividend. In such case, the market price will be same before and after dividend. The solution goes as follows: Alternative 1: Rs. 480,000 is paid as dividend : No. of shares : 24,00,000 / 24 = 100,000. Dividend per share : 480,000 / 100,000 = Rs. 4.8 The dividend will not affect the EPS hence price will remain at Rs. 24 as before. Alternative 2: Going for repurchase for Rs. 480,000: No. of shares repurchased : 480,000 / 24 = 20,000. Remaining shares : 80,000. EPS of Rs. 2.4 will change to 2.4 x 100,000 / 80,000 = Rs. 3 i.e. 25% increase on existing value. EPS goes parallel with market price hence the price will also go up by 25% i.e. from Rs. 24 to Rs. 24 + Rs. 6 = Rs. 30. Alternative 2 is better in view of better EPS. 2003 Dec 6: An analysis intends to value an IT company in terms of the future cash generating capacity. He has projected the following after-tax cash flows: Year 1 2 3 4 5 Cash flows Rs. In million 166 45 60 81 110 It is further estimated that beyond the 5th year, cash flows will perpetuate at a constant growth rate of 7% p.a., mainly on account of inflation. The perpetual cash flows is estimated to be Rs. 968 million at the end of 5th year. (i) What is the value of the company based on expected future cash flows ? You may assume a cost of capital of 20% for your calculation. (ii) The company has outstanding debt of Rs. 342 million and cash/bank balance of Rs. 256 million. Calculate shareholder value, if the number of outstanding shares is 15.15 million. (iii) The company has received a take over offer bid of Rs. 190 per share. Is it a good offer ? Solution : Year 1 2 3 4 5 Pv factor of 20% 0.833 0.694 0.579 0.482 0.402 Cash flows 166 45 60 81 110 Pv of cash flows 138.33 31.25 34.72 39.06 44.21 287.57 Pv of perpetual cash flows of Rs. 968 million per year at 7% growth rate due to inflation Rs.million = 0.402 x ( 968 x 1.07) / ( 0.20 0.07 ) 3202.89 Pv of cash flows 287.57 Total value of the company 3490.46 Shareholder's value : 3490.46 + 256 (cash) -- 342 (loan) 3404.46 Value per share (3404.46/15.15) Rs. 224.72 The offer price of Rs. 190 per share is much lower than the value, bid value is not a good offer. 2007 Dec 5b Ashika Text Ltd., a manufacturing company producing textiles product, has a sales of Rs. 9.60 crore, variable cost Rs. 5.60 crore and fixed cost of Rs. 1.04 crore. The company has debt equity resources worth Rs. 11.20 crores and Rs. 16.0 crore respectively. The cost of debt is 10%. Calculate : (i) The company's ROI (ii) EBT if sales decline to Rs. 6 crores (iii) The % change in EPS if the sales increase by 5%. Ignore taxation. Solution : 1. For computing ROI, you must have EBIT at present. ROI is computed as % of EBIT per unit of Capital employed or Investment. What is EBIT and EBT and What is total capital employed? Read the question again and compute the ROI before you see any further. 2. The relationship between EPS and Sales is represented by Combined leverage. You should revise your knowledge about operating, financial and combined leverages before you solve the second part. Determination of ROI : Total investment = Debt + Equity = 11.20 + 16.0 = Rs. 27.2 crores.

EBIT = Sales variable cost fixed cost = 9.60 5.60 1.04 = Rs. 2.96 crores. ROI = EBIT / Investment = 2.96 / 27.2 = 10.9%, (ii) EBT if sales is Rs. 6.0 crores. The variable cost is proportional to sales. For sales of Rs. 9.60 crores, the variable cost is Rs. 5.6 crores i.e. 58.33%. Same % will be applied for reduced sales of Rs. 6.0 crores. Sales 6.00 Less : Variable cost at 58.33% of sales 3.50 Contribution 2.50 Less : Fixed cost 1.04 EBIT 1.46 Less : Interest 1.12 EBT 0.34 (iii) Combined leverage = Contribution / EBT = (9.60 5.60) / (2.96 1.120 = 2.17. If the sales increase by 5%, the EPS will increase by 2.17 x 5% = 10.85%. 2008 June 7a Welsh Ltd has 150,000 equity shares of Rs. 10 each and 12% long-term debt of Rs. 12,00,000 outstanding at the beginning of the year 2008 09. The finance department of the company has generated the following forecast financial statistics for the year 2008 -- 09: Return on Total Assets (ROTA) (EBIT / Total Assets) : 25% Debt ratio (External liabilities / Equity) 0.75 Effective Interest Rate (EIR) (interest expenses / Total liabilities) 10% Current assets to Fixed assets 0.60 :1 Tax rate 40% The assets, liabilities and equity figures used to compute the above financial statistics are based on forecast balances as at 31.3.2009. The company has no plans to change its equity and long term debt. Requirements: (i) Prepare the forecast balance sheet as at 31st March 2009 with as many details as possible and (ii) Forecast Earnings per share. Show necessary workings. Solution : Tutorial notes: 1. This question is a typical one. It is an advanced problem of ratio analysis. 2. Figures are given for the beginning of 2008 09. You are asked to give figures for the end of the year. 3. Total interest expense is 10% of total liabilities. Total liabilities will be debt (Rs. 12 lacs) plus other liabilities. Interest expense is 12% of Rs. 12 lacs i.e. Rs. 1.44 lacs. From this clue, you can determine the total liabilities and other liabilities. 4. You should find total liabilities as Rs. 14.4 lacs consisting of Rs. 12.0 lacs as debt and Rs. 2.4 lacs as other liabilities. 5. Equity at the beginning of the year is Rs. 15.0 lacs, how much is it at the end of the year? External liabilities are 75% of equity as the debt ratio is given as 0.75. The external liabilities are known in previous step as 14.40 lacs. The equity would be 14.40 / 0.75 = Rs. 19.20 lacs. 6. Total assets are equal to equity plus external liabilities. You have already calculated equity as Rs. 19.2 lacs and external liabilities as Rs. 14.4 lacs. Thus total assets (fixed assets + current assets) will be Rs. 33.6 lacs. Current assets are 0.6 time the fixed assets. You can determine fixed assets as Rs.21 lacs and current assets as Rs.12.60 lacs.

;gka ij tks lkY;w'ku fn;k gS oks Nkfi;sA


2008 Dec 2a
Problem 25: (CWA Final Dec. 08)

Complete the balance sheet and sales information in the table below for Godrej Industries using the following financial data :
Debt ratio : 50% ; Quick ratio : 0.80 ; Total Asset Turnover : 1.5 ; Days sales outstanding : 36.5 days Liabilities Rs. Assets Rs.

Accounts Payable Cash Long-term Debt 60,000 Accounts receivables Common Stock Inventories Retained earnings 97,500 Fixed assets Total Liabilities and Equity Total assets 300,000 Sales Cost of goods sold Solution: Tutorial notes: 1. The solution is given below. You should try before you see the computation given below the solution. Liabilities Rs. Assets Rs. Accounts Payable 90,000 Cash 27,000 Long-term Debt 60,000 Accounts receivables 45,000 Common Stock 52,500 Inventories 90,000 Retained earnings 97,500 Fixed assets 138,000 Total Liabilities and Equity 300,000 Total assets 300,000 Sales 450,000 Cost of goods sold 337,500

Take the given solution here. ;gka


2008 Dec 3a

ij lkY;w'ku Nkfi;sA

The Hyundai Instrument Corporation is trying to determine the effect of its inventory turnover ratio and days sales outstanding (dso) on its cash flow cycle. The Hyundai corporation's sales last year (all on credit) were Rs. 150,000 and it earned a net profit of 6%. Its inventory turnover ratio was 5 and DSO was 36.5 days. The firm had fixed assets totalling Rs. 35,000 and its payable period is 40 days. Calculate : (i) Cash Conversion Cycle (ii) Total assets turnover and ROA, if it holds neglibible amounts of cash and marketable securities and (iii) Cash conversion cycle, Total asset turnover and Return on Assets, if its Inventory Turnover can be raised to 7.3.
Solution :

Cash Conversion Cycle = Inventory Conversion period + Receivable collection period less Payable Deferral period.
Cash Conversion Cycle = 365 / 5 + 36.5 - 40 = 69.5 days.

Total Assets Turnover = Sales / Total Assets. Sales has been given as Rs. 150,000 but you have to compute the Total Assets. Think how can you calculate the Total Assets. Total Assets will consist of Inventory, Receivables and Fixed assets. Inventory turnover is 5 which means sales are 5 times the inventory. The sales are Rs. 150,000 meaning that the inventory must be 150,000 / 5 = Rs. 30,000. Inventory can also be calculated on the basis of Cost of goods sold but the question is silent about it, hence it can be safely assumed that inventory turnover ratio is related to Sales and not to Cost of goods sold. You can write a note in this respect in your answer. Receivable are collected in every 36.5 days. The year is of 365 days and yearly sales are Rs. 150,000. The DSO is 36.5 days which means that at any point, sales of 36.5 days are outstanding which consitute the receivables. Thus receivables must be 150,000 / 365 x 36.5 = Rs. 15,000. The Fixed assets are given as Rs. 35,000. Thus total assets are Rs. 30,000 + Rs. 15,000 + Rs. 35,000 = Rs. 80,000. The sales are known. You can know the Total Assets Turnover.
Total Assets = Inventory + Receivables + Fixed assets Total Assets = Rs. 150,000 /5 + (150,000/365) x 36.5 + Rs. 35,000 = Rs. 80,000 Total Assets = 150,000 / 80,000 = 1.875

ROA stands for Return on Asset. The return is net profit i.e. 6% of sales. The return works out to Rs. 9,000 and total assets as computed are Rs. 80,000. Return on assets must be 9,000 / 80,000 = 11.25%. (iii) The third part is just repetition of second part. Inventory turnover is 7.3 which means sales are 7.3 times the inventory. The inventory must be Rs. 150,000/7.3 = Rs. 20,548. Other assets viz. receivables and fixed assets are same. Total assets are Rs. 20,548 + Rs. 15,000 + Rs. 35,000 = Rs. 70,548. The ROA = 9,000 / 70,548 = 12.7%.
Cash conversion cycle = 365/73 + 36.5 - 40 = 46.5 days. Total assets turnover = 150,000 / 70,548 = 2.12 and ROA = 9,000 / 70,548 = 12.7%.

2009 June 4b Mintex Ltd. gives you the following information for the yea ended 31st March, 2009 : (i) Sales for the year totalled Rs. 96,00,000. The company sells goods for cash only. (ii) Cost of goods sold was 60% of sales. Closing inventory was higher than opening inventory by Rs. 20,000. (iii) Tax paid amounted to Rs. 7,00,000. Other expenses totalled Rs. 21,45,000. Outstanding expenses on 31st March, 2008 and 31st March, 2009 totalled Rs. 82,000 and Rs. 91,000 respectively. (iv) New machinery and furniture costing Rs. 10,50,000 in all were purchased. One equipment was sold for Rs. 20,000. (v) A rights issue was made of 50,000 shares of Rs. 10 each at a premium of Rs. 3 per share. The entire money was received with application. (vi) Dividends totalling Rs. 400,000 were distributed among the shareholders. (vII) Cash in hand and at Bank as at 31st March, 2008 and 31st March 2009 totalled Rs. 210,000 and Rs. 414,000 respectively. Prepare cash flow statement for the year ended 31st March, 2009 using the direct method.
Solution: Tutorial Notes : Format for Cash Flow Analysis: The format for cash flow analysis is given below : Activities Cash inflow Cash outflow Operating activities Investment activities Financing activities Cash flow from operating activities 1. Cash generated from operations Cash receipts from customers Less : Cash paid to suppliers and other operating expenses Less : Interest paid Less : Income tax paid 2. Cash flow from extraordinary items Less : Extraordinary items 3. Net Cash from / used in operating activities Cash flow from investing activities 1.Loans and advances 2. Interest and Dividend received 3. Proceeds on sale of investments 4. Proceeds on sale of fixed assets 5. Less : Purchase of fixed assets Investment is subsidiaries Investments in trade investments Loans and advances repaid Investments on current assets Cash flow from Financing Activities 1. Proceeds from issue of share capital 2. Proceeds from Long-term borrowings 3. Less : Repayment of loans Dividend paid 1. The cash flow statement can be prepared either by direct method or by indirect method. You should know both the methods. Sometimes it is simply mentioned in the question that CFS is to be prepared as per AS3. According to AS - 3,cash flow can be derived either from direct method or indirect method. 2. In direct method, gross receipts and gross payments of cash are disclosed. In indirect method, profit and loss a/c is adjusted for the effects of transactions of noncash nature. Cash Flow Statement for the year ended 31st March, 2009 (Under Direct Method) Rs. In lacs. Rs. Lacs Cash flow from Operating Activities:

Cash receipts from customers Cash paid to suppliers and employees Cash inflow from operation Tax paid Net Cash from Operating Activities Cash flow from Investing Activities Purchase of Fixed assets Proceeds from sale equipment Net cash from investing activities Cash Flow from Financing Activities: Proceeds from issue of share capital Dividend paid Net cash from financing activities

96.00 (79.16) 16.84 (7.00) 9.84 (10.50) 0.2 (10.30) (50,000 x Rs. 13) 6.50 (4.00) 2.50 2.04 2.10 4.14 Rs. Lacs 57.60 21.45 0.82 0.20 80.07 0.91 79.16

Cash and cash equivalents as at 31.3.2008 Cash and cash equivalents as at 31.3.2009 Working Notes: 1. Calculation of cash paid to suppliers and employees: Cost of sales @ 60% of Rs. 96.0 lakhs. Add : Expenses incurred Outstanding expenses on 31.3.2008 Excess of closing inventory over operning inventory Less: Outstanding expenses on 31.3.2009

Preparation of cash flow statement is explained here by an illustration here: The following are summarised Balance sheet of X Ltd. as on March 31,2005 and 2006 : Liabilities 31.3.05 31.3.06 Rs. Rs. Equity share capital 1,000,000 1250000 Capital Reserve --10,000 General Reserve 250,000 300,000 Profit and loss a/c 150,000 180,000 Long term loan from the Bank 500,000 400,000 Sundry Creditors 500,000 400,000 Provision for taxation 50,000 60,000 Proposed Dividends 100,000 125,000 2,550,000 2725000 Assets : Land and building 500,000 480,000 Machinery 750,000 920,000 Investment 100,000 50,000 Stock 300,000 280,000 Sundry Debtors 400,000 420,000 Cash in hand 200,000 165,000 Cash at bank 300,000 410,000 2,550,000 2725000 Additional Information : (i) Dividend of Rs. 100,000 was paid during the year ended March 31,2006. (ii) Machinery during the year was purchased for Rs. 125,000. (iii) Machinery for another company was purchased for a consideration of Rs. 100,000 payable in equity shares. (iv) Income-tax provided during the year Rs. 55,000. (v) Company sold some investment at a profit of Rs. 10,000, which was credited to Capital reserve. (vi) There was no sale of machinery during the year. (vii) Depreciation written off on Land and Building Rs. 20,000. From the above particulars, prepare a cash flow statement for the year ended March 31, 2006 as per AS -- 3 (Indirect method). Solution : Tutorial Notes :

1. You will have to prepare several working notes before you reach final solution. 2. Use figures in thousands for convenience. 3. You can begin with the first item in balance sheet i.e Share capital. The share capital goes up from Rs. 10 lac to Rs. 12.5 lacs. Shares worth Rs. One lac were issued for the purchase of machinery, thus Shares worth Rs. 1.50 lacs must have been received in cash. This is cash inflow due to financing activities. 4. The second item is general reserve. It goes up from Rs. 250,000 to Rs. 300,000. The increase of Rs. 50,000 will be added back to net profit to arrive at cash from operating activities. Similar treatment is applicable to profit and loss a/c increase of Rs. 30,000. 5. The loan goes down from Rs. 5 lacs to Rs. 4 lacs. There is payment of Rs. One lac. This is cash outflow due to financing activities. 6. Sundry creditors, provision for taxation, proposed dividend are items of working capital. These will be adjusted in cash flows from operating activities. Alternatively you can compute the working capital for both periods and adjust the final increase or decrease in working capital in cashflows from operating activities. 7. In land and building, the opening balance is Rs. 500,000, depreciation is Rs. 20,000 and closing balance is Rs. 480,000. This shows that there is neither any purchase or sale of land during the year. However, the depreciation will be added back to net profit to calculate the cashflows from operating activities. 8. Next item is Machinery. Opening balance is Rs. 750,000. The purchase of machinery during the year is Rs. 225,000 (125,000 + 100,000). Thus the closing balance must be Rs. 975,000 but it is given as Rs. 920,000. What does this signify ? It implies that the balance figure of Rs. 55,000 represents the depreciation charged during the year. This depreciation will be added back to net profit to arrive at cash from operating activities. 9. Next item is Investment. Opening balance is Rs. 100,000 and Closing balance is Rs. 50,000. This shows that investment worth Rs. 50,000 were sold during the year. Additional information says that some investment was sold at a profit of Rs. 10,000. Thus the investment was sold at Rs. 60,000, this is cash inflow from investing activities. 10. Next four items are components of working capital. You can prepare a table showing the decrease or increase in working capital. The increase or decrease in working capital will be adjusted in cashflows from operating activities. 11. Opening balance for provision for taxation is Rs. 50,000. Provision made during the year is Rs. 55,000. Thus closing balance must be Rs. 105,000 but it is given as Rs. 60,000 which shows that there must be a payment of Rs. 45,000 towards income tax. 12. Payment of dividend is cash outflows due to financing activities. Working Notes : all figures in Rs. Machinery A/c Particulars Rs. Particulars Rs. To Opening balance 750,000 By Depreciation (bal.fig.) 55,000 To Bank 125,000 By Closing balance 920,000 To Share Capital 100,000 975,000 975,000 Share capital A/c To Closing balance 1,250,000 By Opening balance 1,000,000 By Machinery 100,000 By Bank 150,000 1,250,000 1,250,000 Investment A/c To Opening balance 100,000 By Bank (bal.fig.) 60,000 To Capital Reserve 10,000 By Closing balance 50,000 110,000 110,000 Proposed Dividend A/c To Bank 100,000 By Opening balance 100,000 To Closing balance 125,000 By Profit and loss a/c 125,000 225,000 (balancing figure) 225,000 X Ltd.

Cash Flow Statement for the period ended 31.03.06 (A) Cash flows from Operating Activities : Closing balance of profit and loss a/c 180,000 Adjustments for : Provision for taxation 55,000 Proposed Dividend 125,000 Transfer to General reserve 50,000 Opening balance of profit and loss a/c (150000) Net profit before taxation and extraordinary items 260,000 Adjustments for Depreciation : Depreciation on machinery 55,000 Depreciation on Building 20,000 Operating profit before working capital changes 335,000 Decrease in stock 20,000 Increase in debtors (20000) Decrease in creditors (100000) Cash generated from operations 235,000 Income tax paid (45000) (B) Cash flows from Investing Activities: Purchase of machinery (125000) Sale of investment 60,000 Cash used in investing activities (C) Cash flows from Financing Activities : Payment of dividend (100000) Payment of loan (100000) Proceeds from issue of share capital 150,000 Cash used in Financing activities Net increase in cash or cash equivalents (A+B+C) Cash or cash equivalents at the beginning of period Cash or cash equivalents at the end of period

190,000

(65000)

(50000) 75,000 500,000 575,000

2009 Dec 2(b) The Balance sheets of Magnavision Ltd. as on 31.03.2008 and 31.03.2009 are as given below: Assets 31.03.08 31.03.09 Liabilities 31.03.08 31.03.09 800,000 640,000 Share capital 900,000 900,000 Land and buildings 100,000 120,000 Reserves 600,000 620,000 Investments 480,000 420,000 Profit and loss a/c 112,000 136,000 Inventory 420,000 910,000 Term loan 540,000 A/c receivables 298,000 394,000 Current liabilities 486,000 288,000 Cash and Bank 2098000 2484000 2098000 2484000 The following details are provided : (i) Dividend paid during the year 2008 09 was Rs. 80,000. (ii) Net profit for the year 2008 09 was Rs. 124,000 after charging depreciation on fixed assets amounting to Rs. 140,000. (iii) Investments worth Rs. 16,000 were sold during the year 2008 09 for Rs. 17,000. No revaluation of the investments was carried out at the year end (iv) Fixed assets were sold at a profit of Rs. 4,000 during the year and the profit is included in the net profit for the year ended 31.03.09. You are required to prepare the statement of changes in working capital and the Funds Flow Statement for the year ended 31st March, 2009. Solution : Tutorial Notes: 1. Dividend paid is application of funds. You can straight away write this under application of funds in your funds flow statement. 2. Rs. 17,000 received on sale of investment is a source of funds.

3. Opening balance of fixed assets is Rs. 800,000 and depreciation charged is Rs. 140,000. Closing balance should be 800,000 140,000 = Rs. 660,000 but is given as Rs. 640,000. This means that the assets worth Rs. 20,000 were sold during the year. The question also mentions that fixed assets were sold at a profit of Rs. 4,000, hence Rs.24,000 (20,000 + 4,000) must have been received for sale of fixed asset. This is source of funds. Show fixed assets accounts as a working note. 4. Statement of changes in working capital should be prepared separately whether it is asked in the question or not. The increase in working capital is an application of funds and vice versa. 5. The opening balance of term loan is zero and closing balance is Rs. 540,000. This shows that Rs. 540,000 were received as loan during the year. This is source of funds. Statement of changes in working capital : (all figures are in Rs.) Assets 31.03.08 31.03.09 480,000 420,000 Inventory 420,000 910,000 A/c receivables 298,000 394,000 Cash and Bank Total current assets 1198000 1724000 486,000 288,000 Less : Current liabilities 712,000 Working capital 1436000 724,000 Increase in working capital Funds Flow Statement : Sources of Funds Rs.. Application of Funds Rs. 259,000 Purchase of investment 36,000 Funds from operation 540,000 Dividend paid 80,000 Term loan raised 24,000 Increase in working capital 724,000 Sale of Fixed assets 17,000 840,000 Sale of Investment 840,000 Total 840,000 2010 June 3b: The following particulars.. using Indirect method. Solution : Take the same solution.

Chapter 8 : Leverage and CVP Analysis


2001 Dec 2b: The present output details of manufacturing department of Simple Ltd are as follows: Average output per week 48,000 units Saleable value of output : Rs. 60,000 ; Contribution made from sale : Rs. 24,000 The management plans to introduce more mechanization in the department at a capital cost of Rs. 16,000. As an effect of this the number of employees will be reduced from the existing strength of 160 nos to 120 nos, but the output of the individual will increase by 60%. As an incentive to achieve the extra output, the management proposes to offer an one per cent increase in the existing piece work price of Re. 0.10 per article for every 2% increase in the individual output achieved. In order to sell the increased output, it will be necessary to reduce the sale price by 4%. You are required to calculate extra weekly contribution resulting from the proposed changes, as above, and give your recommendation. Solution : Tutorial notes: 1. At present 160 workers are producing 48,000 units per week. It means present rate of production is 300 units per worker per week. 2. With more mechanization, workers will be 120 producing 480 (60% above current level of 300 units) per worker per week. Production will go up to 120 x 480 = 57,600 units. 3. The incentive is given as 1% for every 2% increase in individual output. The individual output increases by 60% as given, hence incentive will be 30% of the present rate of Re. 0.10. The revised rate will be Re.0.13 per piece. 4. The variable cost will also change with production. Presently the variable cost is Rs. 36,000 which includes Rs. 4,800 (48,000 x 0.10) as labour. Variable cost without labour is Rs. 31,200 which is 0.65 per unit (31,200 / 48,000). 5. Present selling price per unit is Rs. 1.25 (60,000 / 48,000). The proposed selling price is 4% less than the current selling price i.e. 0.96 x 1.25 = Rs. 1.20 per unit. 6. Solve the question on the above lines and compare your solution with the one given below. Comparative statement of present and proposed position Present Proposed Production in units 48,000 57,600 Selling price Rs. per unit 1.25 1.20 Sales revenue 60,000 69,120 Variable cost per unit Labour 0.10 0.13 Other 0.65 0.65 Total variable cost per unit 0.75 0.78 Total variable cost 36,000 44,928 Contribution 24,000 24,192 Extra contribution in proposed scheme : Rs. 192 per week. Present capital outlay : Rs. 16,000. Break even will be 16,000 / 192 = 83.3 weeks. If the present scheme is implemented, the scheme will start giving profit of Rs. 192 per week after a period of 83.3 weeks. 2001 Dec 4b: The following figures are available for Success and Company: Net Sales : Rs. 15 crores; EBIT as % of sales : 12% Income tax applicable : 40% Capital employed : (a) Equity Rs. 5 crores (b) Debt at 15% Rs. 3 crores (c) Preference shares of Rs. 1 crore at 13% rate of dividend. You are required to calculate : (i) the Return on equity of the company and (ii) the operating leverage of the company given that combined leverage is 3 Solution: It is very easy. Solve it on your own. Net sales 1500 (figures in Rs. lacs)

EBIT @ 12% of net sales 180 Less : Interest on debt (15% of Rs. 300 lacs) 45 EBT 135 Tax @ 40% 54 Earning after tax 81 Less : Preference dividend at 13% on Rs. 100 lacs 13 Earning for equity shareholder Or Return on equity Or ROE 68 ROE as % (68 /500) = 13.6%. (ii) Second part is formula based : Financial leverage = EBIT / [ EBT { Pref.Div./ (1 T)}] Financial leverage = 180 / [135 ( 13 ) / (1 0.40)] = 1.6 We have Combined leverage = Financial Leverage x Operating leverage CL is given as 3. So operating leverage = 3 / 1.6 = 1.9 2005 June 3b: During the year 2000 01, Gulf Oil India made a sales of lubricants and greases worth Rs. 257 crores, which was down by 7.4% compared to previous year's sales. While the company could reduce its overheads, its variable input (base oil and additives) cost went up significantly. As a result, variable cost-to-sales ratio in 2000 - 01 stood at 55.8% as opposed to 48.5% in the previous year. The financial highlights of the company for the year 2000-01 are as under: Sales (Rs. crores) 257.0 (figures in Rs. crores) Overheads (excluding depreciation and interests) 103.5 Depreciation 2.7 Interest 6.5 Earnings before tax (excluding other income) 0.7 (i) Calculate the operating leverage. The company is expecting a decline in sales by 1% in the next year. If the cost structure remains the same, what will be the expected EBIT? Show necessary calculations. (ii) Calculate financial leverage. What would be the impact of financial leverage if company's sales decline by 1 per cent in the next year? Assume that financial structure remains the same. Solution : Tutorial notes: 1. Some additional data have been given which are superfluous (extra). (i) OL is Contribution / EBIT. Variable cost is given as 55.8% of sales, hence contribution will be 44.2% of sales of Rs. 257 crores. Contribution is 0.442 x 257 = Rs. 113.6 crores. EBIT is EBT plus interest. EBT is given as 0.7 crore and interest is Rs. 6.5 crore. EBIT is Rs. 7.2 crores. EBIT and contribution both are known, compute the OL. You must find it to be 15.78. OL of 15.78 means if sale increases or decreases by 1%, the EBIT will increase or decrease by 15.78%. As the present EBIT is Rs. 7.2 crores, it will decline by 15.78% (i.e. to Rs. 6.06 crores) if sale declines by 1%. (ii) FL is EBIT / [EBT (pref.div) /(1 T)]. As the pref. dividend is nil here the FL is simply the ratio of EBIT with EBT. Both are given with the question. Find FL. FL is 7.20 / 0.7 = 10.3. The EPS will decline by OL x FL i.e. by 10.3 x 15.78 = 162.31%. In other words the EPS will be negative.
2006 June 2b: The following key information pertains to Excel Ltd. for the year 2005-06.(Rs. In lacs) Sales 82.50 Variable cost 46.20 9% Debentures 50.00 Eq. shares Rs.100 each 60.00 Fixed cost 6.60 Corporate tax 35% You are required to work out : What is the company's ROI ? (i) Does it have favourable financial leverage ? (ii) (iii) If the company belongs to an industry whose asset Turnover is 3, does it have a high or low asset leverage

(iv) (v) (vi)

What are the operating, Financial and Combined Leverage of the Company ? What is the company's EPS ? What will be the expected EPS if the Sales of Excel Ltd.,increases by 10% in the next year and Cost structure as well as financial structure remains the same ?

Solution : (i) ROI = EBIT / Investment. EBIT is Sales Variable cost Fixed cost = Rs. 29.70 lacs. Investment = Share capital + Debentures = Rs. 110 lacs. ROI = 29.70 / 110 = 27% (ii) Yes, The company has favourable Financial leverage as its ROI is higher than the interest on debt. (iii) Asset Turnover = Sales / Total assets or total investment.= 82.50/110 = 0.75. (iv) Calculation of Leverages : Operating leverage = Contribution / EBIT = 36.30/29.70 1.22 Financial leverage = EBIT / EBT = 29.70 / (29.70 4.50) 1.18 Combined leverage = Contribution/EBT or OL x FL = 1.22 x 1.18 1.44 (v) Determination of EPS current with 10% increase Rs. Lacs Rs. Lacs Sales 82.50 90.75 Less Variable cost 46.20 50.82 Contribution 36.30 39.93 Fixed cost 6.60 6.60 EBIT 29.70 33.33 Less Interest (9% of Rs. 50 lacs) 4.50 4.50 EBT 25.20 28.83 Less :Tax @ 35% 8.82 10.09 EAT or Earnings after tax 16.38 18.74 No. of shares in lacs 0.6 Earnings per share Rs. 27.30 31.23 (vi) Expected EPS : % increase in EPS due to increase in sales by 10% = 10 x OL = 10 x 1.44 = 14.40% So Expected EPS = + 14.40% = 1.144 x 27.30 = Rs. 31.23

2008 June 2a The Financial highlights of Amtex Ltd. for the year 2007 08 are given as under: EBDIT Rs.830 crores Effective tax rate 30% EPS : Rs. 4.00 Depreciation Rs. 6 crores Book value : Rs. 30 per share Number of outstanding shares : 33 crores and D/E ratio : 1.5 : 1 Required : (i) Calculate the degree of financial leverage (ii) What is the financial Break even point of Amtek Ltd. (iii) What should be the impact of EPS if the EBIT is increased by 5%? Solution : 1. Degree of FL is given by EBIT / EBT. EBIT is EBDIT less depreciation. EBIT is known. 2. How will you calculate the Interest? EBIT less Interest will give EBT. Think how can you calculate interest with the data given. 3. Financial break even is that point EPS is zero. When EBIT is just equal to Interest, the EBT is zero. 4. The relationship between EPS and EBIT is given by FL. Once FL is known, the answer to part (ii) becomes easy. 5. You should try to solve it on your own before you see below. Computation of EBT : EPS is given as Rs. 4.0 per share and number of shares is 33 crores. Total earning after tax is 4.0 x 33 = Rs. 132 crores. The tax rate is 30%. EAT is EBT taxes. Hence EBT = EAT / 0.70 = 132 / 0.70 = Rs. 188.57 crores. If EBT is Rs. 188.57 crores and EBIT is Rs. 824 crores. Interest must be 824 188.57 = 635.43. As far as computation of FL is concerned, you can compute it without calculating the Interest. FL is ratio of EBIT / EBT = 824 / 188.57 = 4.37. However, it is advised that you should compute the interest and present your data in proper format as given below.

EBIDT (as given) 830.0 (figures in Rs. crores) Less : Depreciation (given) 6.0 EBIT 824.0 Note : Presentation carries marks in the Less : Interest (working note) 635.4 exam. You must present the data in EBT 188.6 proper format. Less Tax at 30% 56.6 EAT (Earning after tax) 132.0 EPS :EAT / Number of share (33 crores) 4.0 Financial Leverage : EBIT / EBT = 824.0 / 188.6 = 4.4. (ii) At Financial Break even is the point at which the company earns just enough to pay the interest on its debts i.e. it is the point at which EBIT is equal to Interest. At Financial Break even : EBIT = Interest = Rs. 635.4 crores. (iii) If EBIT changes by 5%, the EPS will correspondingly change by 5% x FL = 5% x 4.4 = 22%.

Chapter 7 : Financial Strategy


2004 June 7c: From the following information concerning Nebula Ltd., prepare a statement showing computation of EVA for the year ended 31st March 2004: Summarized Profit and loss a/c for the year ended 31st March 2004: Rs. Rs. Sales 2000000 Cost of goods sold 1200000 Gross profit 800,000 Expenses General 200,000 Office and Administration 250,000 Selling and Distribution 64,000 514,000 Profit before Interest and Tax (PBIT) 286,000 Less : Interest 36,000 Profit before tax (PBT) 250,000 Tax @ 40% 100,000 Profit after tax (PAT) 150,000 Summarized Balance sheet as on 31st March,2004 Liabilities Rs. Assets Rs. Equity shares 240,000 Fixed assets (net) 600,000 Reserves 160,000 Current assets Stock 120,000 Term loan 240,000 Debtors 60,000 Current liabilities 160,000 Bank 20,000 200,000 800,000 800,000 Other particulars: 1. General expenses include R and D expenses of Rs. 80,000. For EVA computation R & D expenses are to be taken as an investment. 2. Cost of goods sold include depreciation expense of Rs. 60,000. 3. The expectation return of shareholders is 12%. Solution : Tutorial notes: 1. EVA = NOPAT Cost of Capital employed. 2. NOPAT is net profit after tax plus interest (after tax). Interest is Rs. 36,000, hence interest after tax would be (1 T) x Rs. 36,000. With tax rate at 40%, the interest after tax would be Rs. 21,600. 3. R&D expenses have to be treated as investment, it means R&D expense is part of NOPAT. 4. To determine the cost of capital employed, you must know the after tax cost of each component of capital employed, Can you do it? 5. What is the cost of term loan ? Read the question and find out. Computation of after tax cost of capital employed : Component Amount After tax cost of capital Cost Rs. Share capital 240,000 12% 28,800 Reserves 160,000 12% 19,200 Investment 80,000 12% 9,600 Term loan 240,000 9% 21,600 720,000 79,200 Note : (i) Investment in R&D has been taken as part of capital employed and (ii) Rate of term loan (before tax) is 36,000 / 240,000 = 15%. After tax cost of loan would be 15% less 40% i.e. 9% Computation of NOPAT 150,000 21,600 (not Rs. 36,000) 171,600 80,000 251,600 Note : No adjustment is necessary if R&D expenses are revenue in nature. EVA = NOPAT Cost of capital employed = 251,600 79,200 = Rs. 172,400. Net profit after tax Add : Adjusted interest after tax Profit after tax but before interest R&D expenses adjusted

2005 Dec 7a : The accounts of Siteraze Ltd. (SL) engaged in manufacturing business are summarized below: Income Statement For The Year Ended March 31, 2005 (Rs. in million) Sales Revenue 95.0 Less: Cost of goods sold 59.1 General Expenses 6.8 Administrative expenses 7.8 Selling and Distribution expenses 2.9 Interest on loan 1.8 78.4 Earning before tax 16.6 Corporate tax (0.35) 5.8 Earning after tax 10.8 Balance sheet as at March 31,2005 ( in Rs. million) Liabilities Assets Equity Share capital Land and buildings net 20.0 10 lakh shares of Rs. 10 each. 10.0 Plant & machinery net 28.5 Reserves 31.5 Current assets : Stock 10.0 10% Loan 18.0 Debtors 15.0 Creditors and other liabilities 18.0 Bank 4.0 77.5 77.5 Additional Information : 1. The risk free rate of return in the economy is 8% and the premium expected from business in general is 5%. The beta of company is currently 1.27. 2. The equity shares of this company quoted in the market on 31.3.05 at Rs. 50 per share. 3. General expenses include R&D expenses of Rs. 0.50 million. 4. For EVA computation R&D expenses are to be considered as investment. Requirements: (i) Determine the EVA for the year ended 31.3.2005 and (ii) Determine the amount of MVA (market value added) for the year ended 31.3.2005. Solution : Tutorial Notes: 1. First additional information should make it clear to you that CAPM is to be used. 2. To arrive at EVA, you must know Nopat and cost of capital employed. How would you know these two figures? 3. What you will do with R&D expense of Rs. 0.50 million? You are likely to forget this point. As per CAPM : Cost of capital = Risk free rate + Beta x risk premium; Putting values, one gets Cost of capital = 8% + 1.27 x 5% = 14.35% Cost of loan is 10% and after tax cost will be ( 1 0.35) x 10 = 6.5%. Since net profit after tax is to be used in EVA, after tax cost of loan should be used in computation of capital employed. Determination of capital employed Component Amount Cost Total Share capital 10.0 14.35% 1.435 Reserves 31.5 14.35% 4.520 R&D 0.5 14.35% 0.072 Loan 18.0 6.50% 1.170 Cost of capital employed in Rs. million 7.20 Determination of NOPAT : You are advised to calculate Nopat before you see the following : Net profit after tax and interest 10.80 Add : After tax cost of interest : 1.80 x (1 0.35) 1.17 Profit after tax and before interest 11.97 Add : R & D expenses of capital nature 0.50 Net Operating profit after tax 12.47 Less : Cost of capital employed 7.20 Economic Value Added 5.27 (ii) Determination of Market Value Added for the year ended 31.3.2005:

Market value added = Market value Shareholders funds (net worth) = Rs. 50 per share x 10 lakh shares (Share capital + Reserves + R&D adjustment) = Rs. 50 million (10.0 + 31.50 + 0.50) = 50 42 = Rs. 8 million. 2006 June 2b : The Profit and loss a/c of Cimex Ltd. for the year ended 31.03.06 is as below: Rs.'000 Sales 6,500 Analysis of Cost of Goods sold Less : Cost of good sold 4,200 Materials 40% Gross profit 2,300 Wages 50% Less : Office etc. expenses (1,200) Depreciation 4% Net profit before interest and taxes 1,100 Electricity 6% Less : Interest expenses 180 Analysis of Office etc. expenses Net profit before tax 920 Salaries 60% Less : Income tax 380 Postages, printing, stationery 15% Net profit after tax 540 Rent, Electricity and Insurance 20% Add: Non trading expenses 90 Depreciation 5% Profit available for distribution 630 Dividend paid and payable 210 Prepare a Value added statement for the year ended 31.3.2006 Solution : Tutorial Notes 1. The value addition is utilised in the following manner : To Employees as Salaries etc. To Fixed assets as Depreciation To Government as Taxes To Lenders as Interest To Owners as Dividend and retained profit Value addition = total turnover bought in goods and services Some professionals treat the depreciation as part of BGS (bought in goods and services). 2. For computing the value addition, you must know how to compute value of bought-in goods and services. From the data supplied with the question, you should try to compute BGS before you see below: 3. The value addition is applied to five element as shown above. You can counter check it by computing the application of value addition as given below: Application of value addition : To employees as salaries etc (2,100 + 720) To Fixed assets as depreciation (168 + 60) To Government as taxes To Lenders as interests To Owners as dividend and retained profit 2,820 228 380 180 630 4,238

4. Determination of value of BGS (bought in goods and services) and Value addition : Determination of Value addition: Determination of Bought in goods and services: Turnover 6,500 Materials : 1,680 Less : BGS 2,352 Postage, printing etc. 180 Value addition 4,148 Rent, Electricity and Insurance 240 Add : Non trading income 90 Electricity (6% of 4,200) 252 Value addition for application 4,238 Bought in goods and services (BGS) 2,352 Additional Information : On the basis of the following Income Statement pertaining to Dark Ltd., you are required to prepare: (a) Gross Value Added statement and (b) Statement showing reconciliation of gross value added with profit before taxation. Profit and Loss Account of Dark Ltd. for the year ending March 31, 2007 Rs. in '000 Income Sales less returns 1527956 Dividends and interest 130 Miscellaneous income 474

1528560 Expenditure Production and Operational Expenses: Decrease in inventory of finished goods 26,054 Consumption of raw materials 740,821 Power and lighting 120,030 Wages, Salaries and Bonus 381,760 Staff Welfare 26,240 Excise Duty 14,540 Other manufacturing expenses 32,565 1342010 Administrative Expenses: Director's Remuneration 7,810 Other Administrative expenses 32,640 40,450 Interest on 9% mortgage Debentures 14,400 Long term loan from financial institutions 10,000 Bank overdraft 100 24,500 Depreciation on fixed assets 50,600 1457560 Profit before taxation 71,000 Provision for income tax @ 35.875% 25,470 Profit after taxation 45,530 Balance of Account as per the last balance sheet 6,300 51,830 Transferred to General Reserves 18,212 Proposed Dividend @ 22% 22,000 Tax on distributed profits @ 12.81% 2,818 43,030 Surplus carried to Balance sheet 8,800 Solution: Working Notes: Cost of bought in material and services Decrease in inventory of finished goods Consumption of raw materials Power and lighting Other manufacturing expenses Rs. in '000 26,054 740,821 120,030 32,565 919,470

Dark Ltd. Value added statement for the year ended 31st March 2007 Sales less returns 1527956 less: Cost of bought in material and services 919,470 Administrative expenses 32,640 Interest on bank overdraft 100 952,210 Value added by manufacturing and trading activities 575,746 Add: Dividends and interest 130 Miscellaneous income 474 Total value added 576,350 Application of Value added To Pay employees % Wages, Salaries and Bonus 381,760 Staff Welfare 26,240 408,000 70.79% To Pay directors Director's remuneration 7,810 1.36% To Pay government Excise duty 14,540 Income tax 25,470 Tax on distributed profits 2,818 42,828 7.43%

71,312 12.37% 576,350 100.00% Statement showing reconciliation of total value added with profit before taxation Profit before taxation 71,000 Add: Wages, Salaries and bonus 381,760 Staff welfare expenses 26,240 Excise duty 14,540 Director's remuneration 7,810 Interest on 9% mortgage debentures 14,400 Interest on long term loan 10,000 Depreciation on fixed asset 50,600 505,350 Total value added 576,350 2006 Dec 5b: Magnavision Corporation Ltd. (MCL) is considering a new project involving an investment of Rs. 50 million for which the following information is available: Project life 4 years Depreciation Straight line Salvage value Nil Debt-Equity ratio 4:5 Annual revenues Rs. 60 million Interest rate on debt (post tax) 9% Annual costs Rs. 40 million Tax rate 40% (excluding depre,interest and taxes) Beta of company 1.50 Risk free rate of return 6% Market risk premium 8% You are required to (i) Compute the EVA of the project over its life. (ii) Calculate the NPV of the project using EVA approach. (iii) Pv is as follows Year 0 1 2 3 4 14% 1 0.877 0.769 0.675 0.592 15% 1 0.870 0.756 0.658 0.572 Solution : Tutorial Notes: 1. Think how you are going to solve the question. What will be the working notes? 2. The EVA is Nopat less cost of capital employed. How will you compute Nopat? 3. Cost of equity will be given by CAPM as clear from data given in the question. Compute cost of equity from CAPM. This is your working note. 4. What are the other components of capital employed and what are their costs? 5. How will you calculate the WACC for the project? 6. What will be capital employed for all the four years? Will it be same as Rs. 50 million or something else? 7. Cost of equity as per CAPM is 6% + 1.5 x 8% = 18%. The Debt equity ratio is given as 4:5 and debt cost (after tax) has already been given as 9%. From this data can you compute the WACC? 8. EVA is Nopat less cost of capital employed. Capital employed is the capital less depreciation every year. WACC will be its cost. 9. WACC = (4/9) x 9% + (5/9) x 18% = 14% where 4:5 is debt-equity ratio. 10. Capital employed is at the beginning and depreciation is at the end of the year. The solution goes as follows : Computation of Capital Employed Year 1 2 3 Capital employed net after depreciation 50.0 37.5 25.0 Cost of capital employed at 14% 7.00 5.25 3.50 Revenues 60.0 60.0 60.0

To Pay providers of capital Interest on 9% debentures 14,400 Interest on long term loans 10,000 Dividend to shareholders 22,000 To Provide for maintenance & expansion of Co. Depreciation on fixed assets 50,600 Transfer to general reserve 18,212 Retained profit (8,800 -- 6,300) 2,500

46,400

8.05%

4 12.5 1.75 60

Costs 40.0 40.0 40.0 40.0 PBDIT 20.0 20.0 20.0 20.0 Depreciation 12.5 12.5 12.5 12.5 PBIT 7.5 7.5 7.5 7.5 Tax @ 40% 3.0 3.0 3.0 3.0 NOPAT 4.5 4.5 4.5 4.5 EVA = NOPAT Cost of capital employed (2.5) (0.75) 1.0 2.75 Discount factor at 14% 0.877 0.769 0.675 0.592 PV of EVA (2.2) (0.6) 0.68 1.63 (0.466) 2007 June 7b: X Ltd. earned Rs. 1.6 million on net asset of Rs. 20 million. The cost of capital is 11.5%. Calculate the net percentage return on investment and EVA. Solution : It is very easy. Solve it on your own. Return on investment = Return / Investment = 1.6 / 20.0 = 8%. EVA = Return less Cost of capital employed = 1.60 11.5% of Rs. 20 million = Rs. 0.7 million. EVA is negative. 2008 June 6b : The financial statements of Deportive Ltd. are given below: (Amount in Rs. million) Balance sheet at 31.3.2007 31.3.2008 Fixed assets 370 480 Net Current assets 400 500 Financed by : 770 980 Shareholders' Funds 595 720 Medium and long term bank loans 175 260 770 980 Profit and loss a/c for years ended 31.3.2007 31.3.2007 Turnover 995 1,180 Pretax accounting profit 210 265 Taxation 63 80 Profit after tax 147 185 Dividend 50 60 Retained earning 97 125 Pretax accounting profit is taken after deducting the economic depreciation of the company's fixed assets (also depreciation used for tax purposes). Additional Information : 1. Economic depreciation was Rs. 95 million in 2003 07 and Rs. 105 million in 2007 08. 2. Interest expenses were Rs. 13 million in 0607 and Rs. 18 million in 0708. 3. Other non-cash expenses were Rs. 32 million in 0607 and Rs. 36 million in 0708. 4. Corporate tax rate in 0607 and 0708 was 30%. 5. Deportivo Ltd. had non-capitalised leases valued at Rs. 35 million in each year. 6. The company's cost of equity was estimated to be 14% in 0607 and 16% in 0708. 7.The company's pre-tax cost of debt was estimated as 7% in 0607 and 8% 0708. 8. The target capital structure is 75% equity and 25% debt. 9. Balance sheet Capital employed at the end of 0506 was Rs. 695 million. Required : EVA for the company for 200607 and 200708. Solution: Take the same solution. 2009 June 7b:
Multiforum Ltd. is expected to grow at a higher rate for four years; thereafter the growth rate may fall and stabilize at a lower level. The following information is available : Base Year (Year = 0) information Rs. Lacs 1,500 250

Revenues EBIT

Capital Expenditure Depreciation Paid up equity capital (Rs. 10 par) Market value of debt Corporate tax rate (for all time) Working Capital as a percentage of revenue

175 125 200 600 30% 25% Inputs for the High Growth Period Growth rate in revenues, depreciation, EBIT and Capital expenditure 20% Length of high growth phase 4 years Working Capital as a per cent of revenues 25% Cost of debt (post debt) 9.1% Debt-equity ratio 1:1 Cost of equity 18.90% Inputs for the Stable period Expected growth rate in revenues and EBIT 10% Working Capital as a per cent of revenues 25% Cost of debt (post-tax) 8.50% Debt-equity ratio 2:3 Cost of equity. 16% Requirements : (i) What is the value of Multiforum Ltd. in terms of forecasted free cash flows? (ii) Calculate the value of shareholders. Note : Extracted from the table of present value of Re. 1: Year 0 1 2 3 4 PVIF at 13% 1.000 0.885 0.783 0.693 0.613 PVIF at 14% 1.000 0.877 0.769 0.675 0.592 Solution : 1. The valuation of a firm can be done with the figures of balance sheet or on ROI basis. None is provided here. 2. The valuation is to be based on forecasted free cash flows. Cash flows of future years will have to be discounted to get the PV of cash flows. Can you guess what should be the discounting rate and how it is going to be determined? 3. Future cash flows for future years are indirectly given. Do you see where? 4. How can you calculate the value of shareholders from value of Multiforum Ltd.? 5. Cost of equity capital is given by CAPM : Cost of capital = Risk free return + beta (market return - risk free rate) 6. Cost of capital is also determined with debt-equity ratio. Determination of Future Cash Flows : fig. in Rs. Lacs Particulars Growth Year 0 Year 1 Year 2 Year 3 Year 4 Terminal Revenues 20% 1,500 1,800 2,160 2,592 3,110 3,421 EBIT 20% 250 300 360 432 518 570 Less; Tax @ 30% 75 90 108 130 156 171 EAT 175 210 252 302 363 399 Capital Expenditure 20% 175 210 252 302 363 0 Depreciation 20% 125 150 180 216 259 0 Working capital as 25% of revenues 375 450 540 648 778 855 Additional working capital required 75 90 108 130 78 Free cash flows: 75 90 108 130 321 Terminal cash flow is 321 (399 - 78). For terminal value the growth is 10%, hence revenue = 1.10 x Rs. 3,110 = Rs. 3,421. Similarly other figures. Determination of Cost of Capital for both the periods : Growth Period : Cost of capital = 0.50 x 0.189 + 0.50 x 0.091 = 14% Stable Period : Cost of capital = 0.40 x 0.085 + 0.60 x 0.16 = 13%. Discounting rate is known. The cash flows are also known. To arrive at the cumulative cash flows beyond the year 4, the cash flows will be adjusted with growth rate and cost of capital. The cost of capital is 13% and growth is 10%. The cumulative cash flows would be cash flow for 5th period / (0.13 - 0.10). This cash flows will be discounted with cost of capital of growth period i.e. 14%. Particulars Free cash flows: Fig. in Rs. Lacs Pv factors Pv of future cash flows Year 1 75 0.877 65.8 Year 2 90 0.769 69.2 Year 3 108 0.675 72.9 Year 4 130 0.592 76.7 Terminal 321 0.592 6342.5 Total

6627.1

Pv of terminal flow (321 x 0.592 ) / (0.13 -.10) The value of Multiforum Ltd. is Rs. 6627.1 lacs and value of shareholders : Rs. 6627.1 lacs less share capital Rs. 600 = Rs. 6027.1 lacs.

2009 Dec 6b A financial analyst has been is it a good offer? Take the same solution. 2010 June 3 AMTEK Ltd. which is . Take the same solution.

Chapter 10 : Investment Decisions


2004 June 7: On March 9, 2004.. Trading value as at 23rd July, 2004 Solution : Take the same solution. 2005 June 6a: Ritz Ltd.. outstanding debt. Solution : Take the same solution. 2008 Dec Portfolio A consists.. 0.3820 Solution ; Take the same solution. 2009 Dec 5b: The equity shares. Gainfully exercised. Solution: Take the same solution. 2009 Dec 7a Vatsan Ltd to take up the project Solution : Take the same solution. 2010 June 5b: The manager .. Of the year ? Solution: Take the same solution.

Chapter 11 : Project Management


2001 Dec 2b: The following table. 0.50. Solution : Take the same solution. 2001 Dec 5b: Apex enterprises.. Management for consideration. Solution : Tutorial notes: 1. The new machine would be favourable if its Npv is positive. You must be able to assess various cash flows associated with new machines. What are the cash flows (in and out both) associated with new machine? 2. The cost of new machine is Rs. 4.0 lacs. The old machine can now be sold at Rs. 90,000. The net effective cost of new machine would Rs. 310,000. On what amount, the depreciation should be charged whether on Rs. 400,000 (actual cost) or on Rs. 310,000 (effective cost) and why ? 3. Discounting rate has nowhere been mentioned. You must state that discounting technique could not be used because of insufficient data.

Paste the solution here.


2001 Dec 7c Five projects M,N,O,P and Q are available to a company for consideration. The investment required for each project and the cash flows it yields are tabulated below. Projects N and Q are mutually exclusive. Taking the cost of capital @ 10%, which combination of projects should be taken up for a total capital outlay not exceeding Rs. 3 lakhs on the basis of NPV and benefit cost ratio ? Project Investment Rs. Cash flows Rs. No.of years PV at 10% M 50,000 18,000 10 6.145 N 100,000 50,000 4 3.170 O 120,000 30,000 8 5.335 P 150,000 40,000 16 7.824 Q 200,000 30,000 25 9.077 Solution : Project Investment Cash flows Pv factor PV NPV BC ratio Rs. Rs. Rs. Rs. M 50,000 18,000 6.145 110,610 60,610 2.21 N 100,000 50,000 3.170 158,500 58,500 1.59 O 120,000 30,000 5.335 160,050 40,050 1.33 P 150,000 40,000 7.824 312,960 162,960 2.09 Q 200,000 30,000 9.077 272,310 72,310 1.36 BC ratio is Pv of cash flows / Investment Feasible combinations of various projects for an outlay of Rs. 300,000: Combinations Investment Pv NPV BCR Rank as per Npv. Rs. Rs. Rs. M,N and O 300,000 582,070 282,070 1.94 1 M,N and P 270,000 429,160 159,160 1.59 4 O&P 270,000 473,010 203,010 1.75 3 M and Q 250,000 382,920 132,920 1.53 5 N and P 250,000 471,460 221,460 1.89 2 Comments: The combination of projects M,N and P is optimum for a capital outlay of Rs. 300,000. It will yield net present value of Rs. 282,070 and a BCR of 1.94. This combination is recommended to be taken up. Additional problem from June 2009: Vedika Ltd. with a limited investment funds of Rs. 6,00,000 is evaluating the desirablility of five investment proposals. Their profiles are summarised below: Project M N O P Investment Rs. 100,000 200,000 240,000 300,000 Annual cash flow Rs. 36,000 100,000 60,000 80,000 Life in years 10 4 8 16

Q 400,000 60,000 25 Project N and Question are mutually exclusive. The cost of funds is 10%. Required: Find out the feasible combination of projects and rank them on the basis of Net present value. Note : Extracted from the table of PV of Re. 1 Year 10 4 8 16 25 PVIFA at 10% 6.145 3.170 5.335 7.824 9.077 Solution: See solution to Problem 18. Project M N O P Q Life in years 10 4 8 16 25 Annual cash flow Rs. Lacs 0.36 1.00 0.60 0.80 0.60 Pv factor at 10% 6.145 3.170 5.335 7.824 9.077 Pv of cash inflows 2.212 3.170 3.201 6.259 5.446 Cash outflow 1.000 2.000 2.400 3.000 4.000 Net present value 1.212 1.170 0.801 3.259 1.446 Statements showing feasible combination of projects and their NPV: Combination Total investment Total NPV Rank M, N and O 5.4 3.18 4 M, N and P 6.0 5.64 1 M.Q. 5.0 2.65 5 N,P 2.0 4.43 3 N,Q 6.0 2.61 6 O,P 5.4 4.06 2 The feasible combination of projects M,N and P having a NPV of Rs. 5.64 lacs is recommended.

2002 june 2: Read the following case. european markets. Solution : Take the same solution. 2002 June 5a: Sterling video taken by the management. Solution : Take the same solution. 2002 June 7b : What is the . For this investment. Solution: Take the same solution. 2002 Dec 6a:
S Ltd. has Rs. 10,00,000 allocated for capital budgeting purposes. The following proposals and associated indexes have been determined: Project Amount Rs. Profitability Index 1 300,000 1.22 2 150,000 0.95 3 350,000 1.20 4 450,000 1.18 5 200,000 1.20 6 400,000 1.05 Which of the above investments should be undertaken? Assume that projects are indivisible and there is no alternative use of money allocated for capital budgeting. Solution : Statement showing ranking of projects on the basis of Profitability index Project Amount Rs. Profitability Index 1 300,000 1.22 2 150,000 0.95 3 350,000 1.20 4 450,000 1.18 5 200,000 1.20

Rank 1 5 2 3 2

6 400,000 1.05 4 If PI is made the basis of selection of projects for capital budgeting purposes, the S ltd. is advised to under take the project 1 (Rs. 300,000), 3 (Rs. 350,000) and project 5 (Rs. 200,000). The selection should add most to the shareholder's wealth, the Net present value method is appropriate with this objective. Statement showing NPV of the projects Cash inflows Npv of project Rs. Project Amount Rs. Profitability Index 1 300,000 1.22 366,000 66,000 2 150,000 0.95 142,500 (7500) 3 350,000 1.20 420,000 70,000 4 450,000 1.18 531,000 81,000 5 200,000 1.20 240,000 40,000 6 400,000 1.05 420,000 20,000 The allocation of funds to the projects 1,3 and 5 on the basis of PI, will give Npv of Rs. 176,000 and Rs. 150,000 will remain unspent. However, the Npv of the projects 3,4 and 5 is Rs. 191,000 which is more than the combined Npv of projects 1,3 and 5 (Rs. 176,000). Further, by undertaking the projects 3,4 and 5, no money remains unspent. Therefore, the S ltd. is advised to undertake the projects 3,4 and 5 for investments.

2003 June 6b: A project requires an initial. Of the project. Solution ;


What is risk coefficient? To reduce or eliminate the risk of errors in estimation of cash flows, risk coefficients are used to reduce the estimated cash flows. Risk coefficient of 0.75 means, the estimated cash flows will be multiplied by 0.75 for computations. It may also be called as certainty equivalence factor.

Paste the solution 2003 Dec 9 : A company is consideringindex of each project. Solution : Take the same solution. 2004 Dec 7a: (cwa final june 01,dec.04 similar june 07) A manufacturer is considering four locations for establishing a new project. The location specific costs are as follows: Item I II III IV Rs. Rs. Rs. Rs. Plant and machinery 60,00,000 50,00,000 55,00,000 50,00,000 Taxes per year 30,000 28,000 65,000 45,000 Electricity per year 35,000 30,000 28,000 25,000 Water per year 8,000 7,000 7,000 6,000 Labour per unit 1.00 1.50 1.10 1.75 Material & Equip./unit 0.50 0.60 0.40 0.55 Transportation per unit 0.05 0.10 0.10 0.05 If the cost of capital is 10%, determine the most suitable location for volumes of 50,000 units, 1 lakh units and 1.5 lakh units. Solution: The fixed costs are as follows : Item I II III IV Capital cost @ 10% 600,000 500,000 550,000 500,000 Taxes per year 30,000 28,000 65,000 45,000 Electricity per year 35,000 30,000 28,000 25,000 Water per year 8,000 7,000 7,000 6,000 Total fixed cost in Rs. Lakhs 6.73 5.65 6.5 5.76 The Variable costs are as follows : Rs. Rs. Rs. Rs.

Labour per unit 1.00 1.50 1.10 1.75 Material & Equipment/unit 0.50 0.60 0.40 0.55 Transportation per unit 0.05 0.10 0.10 0.05 Total Variable cost in Rs. Per unit 1.55 2.20 1.60 2.35 Statement showing total cost under different capacities under different locations: Rs. Lacs Rs. Lacs Rs. Lacs Rs. Lacs For 50,000 units : Variable cost 0.775 1.100 0.80 1.175 Fixed costs 6.73 5.65 6.5 5.76 Total 7.505 6.750 7.30 6.935 For 100,000 units Variable cost 1.55 2.20 1.60 2.35 Fixed costs 6.73 5.65 6.50 5.76 Total 8.28 7.85 8.10 8.11 For 150,000 units Variable cost 2.325 3.300 2.40 3.525 Fixed costs 6.73 5.65 6.5 5.76 Total 9.055 8.950 8.90 9.285 Recommendation : For 50,000 units : Location 2 is most economical. For 100,000 units Location 2 is most economical. For 150,000 units Location 3 is most economical. 2004 Dec 8:
KMC company is considering setting up of a computer coaching center at a total outlay of Rs. 18 lakhs. The operational and maintenance costs excluding depreciation are expected to be Rs. 3 lakhs per annum. The useful life of the equipment is 10 years with negilible salvage value. The rate of dep is 30% wdv. The center will generate an income of Rs. 6 lakhs per annum by way of tuition fee and Rs. 1 lakh on job work. The tax rate for the firm is 40%. Calculate the net present value, internal rate of return and benefit cost ratio taking the cost of capital as 12%.

Solution : Take the same solution. 2004 Dec 9: The balance sheet of 31st March 2005,. Solution : Take the same solution. 2005 June10b Lucky computers . All operations costs are equal. Solution Take the same solution. 2005 June 11b: The balance sheet of Sagar.. As on 31st March, 2005. Solution : Take the same solution. 2005 Dec 8b: The projected cash flows and the expected net abandonment values for a project are given below: Year 0 1 2 3 4 Cash inflows Rs. (100000) 35,000 30,000 25,000 20,000 Abandonment value Rs. 65,000 45,000 20,000 0 Should the project be abandoned and if so when ? (take cost of capital as 10%) Solution : Tutorial notes: This question was repeated in Dec 2007.

What is abandonment: The capital budgeting analysis assumes that the project will complete its expected life, however, for some reason or the other, the project may have to be terminated or ceased during its lifetime. This is termed as abandonment of project. The economic rationale for abandonment is the same as that for capital budgeting. Funds should be removed from a project, or disinvested, whenever the project does not economically justify their use. In simpler words, the project is to be abandoned when the inflows from the project are expected to be less than outflows required to run the project. The project should be given up; else we would make less than optimum use of capital if we continued. In general the investment project should be abandoned when : (a) its abandonment value exceeds the present values of the subsequent future cash flows, and (b) it is better to abandon then than in the future. For example, a company may decide to suspend extraction of gold at its mine if the price of gold falls below the extraction cost. Conversely, a company with the right to mine in a particular area may decide to begin operations if the price rises above the cost of extraction. These strategic options, which are known as real options, are typically ignored in standard discounted cash flow analysis (DCF analysis) where a solitary expected present value is computed. These real options, however, can significantly enhance the value of a project by eliminating unfavourable outcomes. Year 0 1 2 3 4 Cash inflows Rs. (100000) 35,000 30,000 25,000 20,000 PV factor 1.000 0.909 0.826 0.751 0.683 Pv of cash flows (100,000) 31,815 24,780 18,775 13,660 (10,970) Net present value (10,970) The project has negative net present value ; It should be abandoned. To determine the appropriate time to abandone the project, we should find out the net present value at the end for all the four years. The year having minimum (negative) net present value should be selected. Present value of abandonment values are as follows: Abandonment value Rs. 65,000 45,000 20,000 0 PV factor 0.909 0.826 0.751 0.683 Present value of abandonment values 59,085 37,170 15,020 0 Statement showing the determination of year of abandonment: Year 1st yr. 2nd yr. 3rd. Yr 4th yr. Rs. Rs. Rs. Rs. 0 Cash outflow (100000) (100000) (100000) (100000) 1 Cash inflow 31,815 31,815 31,815 31,815 Abandonment value 59,085 2 Cash inflow 24,780 24,780 24,780 Abandonment value 37,170 3 Cash inflow 18,775 18,775 Abandonment value 15,020 4 Cash inflow 13,660 Abandonment value 0 Net present value of cash outflow 9,100 6,235 9,610 10,970 There project incurs loss in all the four years as is evident from the above table. The loss is minimum at end of 2nd year (Rs. 6,235), hence the project should be abandoned at the end of 2nd year. 2005 Dec 11 : An enterpreneur wishes..20. Solution ; Take the same solution. 2006 June 9b

A carpet stores has kept the following record of sales of carpets vis--vis the housing permits that the municipality of the town gave in each of the past nine years Year 1997 1998 1999 2000 2001 2002 No. of permits 18 15 12 10 20 28 Sales (Sq.yds) 13,000 12,000 11,000 10,000 14,000 16,000 Year 2003 2004 2005 No. of permits 35 30 20 Sales (Sq.yds) 19,000 17,000 13,000 The Stores Manager feels that the sales of carpets have a linear relationship with the number of housing permits issued by the municipality. Forecast the carpet sales in the year 2006, if the no. of housing permits is estimated to be 25 in 2006. Solution : The relationship is stated to be linear. Let Y be the Sales and X be the number of permits issued. The linear relationship can be expressed as Y = a + b X where a and b are constants to be determined. We can take data of any two years to determine the values of a and b. Let us take latest years 2004 and 2005 for this purpose. Putting the values of sales and permits for these years in the linear relationship equation, one gets For the year 2004: 17,000 = a + 30 b and for the year 2005, 13,000 = a + 20 b. On solution one gets a = 5000 and b = 400. You may get some other values of a and b for other combination of years as the relationship is not perfectly linear between two variables. In such cases it is better to use regression analysis. More appropriately this can also be solved by computing average carpet sales per permit, as follows: Year 1997 1998 1999 2000 2001 Nol of permits 18 15 12 10 20 Sales (Sq.yds) 13,000 12,000 11,000 10,000 14,000 Sales per permit 722 800 917 1000 700 Year 2002 2003 2004 2005 Nol of permits 28 35 30 20 Sales (Sq.yds) 16,000 19,000 17,000 13,000 Sales per permit 571 543 567 650 Total sales per permit (722+800+917+1000+700+571+543+567+650) 6470 Average Sales per permit = 6470/9 = 720 carpets. Forecasted sales of carpet in 2006 with no. of permits as 25 = 25 x 720 = 18,000 sq.yards.

Paste the solution here.


2006 June 10: Superior engineering . Coverage ratio. Solution : Take the same solution.
2006 June 11b: A new project is being set up by ABC Ltd. The company plans to achieve 80% capacity in the third year of operation, when the expected sales will be 64,000 units and selling price will be Rs. 15 per unit. The corresponding variable cost and fixed cost would be Rs. 576,000 and Rs.300,000 respectively. Calculate the following from the given data (i) Pv ratio (ii) Break even point in terms of sales volume, sales revenue and % capacity and (iii) MOS, Solution : 1. Variable cost : Rs. 576000 / 64,000 = Rs. 9 per unit. 2. Contribution per unit = 15 9 = Rs. 6 per unit. 3. Break even point : Fixed cost Rs. 300,000 / Contribution per unit Rs. 6 = 50,000 units at sales of Rs. 750,000. 4. Pv ratio : Contribution / Sales = 6/15 = 40%. 5. MOS = Sales BE sales = 960,000 (64,000 units at Rs. 15) 750,000 = Rs. 210,000. 6. Capacity is 80% at 64,000 units. 100% capacity will be 80,000.

7. Capacity at BE sales : 50,000 / 80,000 = 62.5%

2006 Dec 8c : Khushboo. For this product ? Solution : Take the same solution. 2006 Dec 11b: The CST of two quotations received for a particular project item is as below : Particulars Quotation 1 Quotation 2 Technical suitability Accepted Accepted Basic price Rs. 1,000 1,160 Freight and Packing Rs. 50 75 Taxes and Duties Rs. Actual and subject to Included in basic price future increase firm and fixed. The present rate of statutory taxes and duties on the item is 16% of basic price. As a Project Manager which firm you will select ? Justify. Solution : Particulars Quotation 1 Quotation 2 Technical suitability Accepted Accepted Basic price Rs. 1,000 1,160 Freight and Packing Rs. 50 75 Taxes and Duties Rs. 160 ('@ 16%) 1,210 1,235 Quotation 1 is the lower of the two and is recommended for selection. The future is not certain. The tax rate may go either way viz. upward or downward. It is not prudent to make decision on uncertain factors. 2007 June 8a A manufacture. and 1.5 lakh units. Solution : Please see 2004 Dec 7a 2007 June 10b: The balance sheet of Optima . March 2008. Solution : Take the same solution. 2007 Dec 9b: A ferry service. Effects. Solution : Take the same solution. 2007 Dec 10 An entrepreneur..as follows: Solution : Take the same solution. 2007 Dec 11b: The projected cash flows.. If so when. Solution : See 2005 Dec 8b 2008 June 9b: What category should .. Rs. 270 lakhs. 2010 June 7b: Ankrit Ltd is given Solution : Take the same solution.

ent proposals. Their

ated indexes have been

is no alternative use of

der take the project 1

e with this objective.

. 150,000 will remain

jects 1,3 and 5 (Rs.

cients are used to plied by 0.75 for

2002 Dec 12 a An Indian importer has to settle a bill for $135,000. The exporter has given the Indian company two options : (i) Pay immediately without any interest charges; (ii) Pay after 3 months, with interest 6% per annum. The importer's bank charges 16% per annum on overdrafts. If the exchange rates are as follows: What should the company do? Spot (Rs./ USD) : 48.35 / 48.36 and 3 - month (Rs./USD) : 48.81 / 48.83 Give reasons for your advice. Solution : Tutorial Notes: 1. Both the options are to be evaluated. In first option the company pays immediately. It means the company has to draw money from overdraft and pay the interest for 3 months at 16% per annum. How much the company would require to draw in Rs., for the payment of 135,000 USD. There are two rates given i.e. 48.35 and 48.36, which one would you use and why/ The theory is 'bank always wins'. The company wants USD and will pay rupees to the bank. The bank will charge Rs. 48.36 (not Rs. 48.35) for every dollar given by it. Option 1: Pay immediately without any interest charges to exporter: 1. Bill value for 135,000 usd @ Rs. 48.36 6,528,600 2. Interest charges for 3 months @ 16% on Rs. 65,28,600 261,144 Total amount needed today 6,789,744 Option 2: Pay after 3 months and pay interest to exporter at 6% per annum: In this case the company will pay 135,000 dollars and interest on it at 6% p.a. for 3 months. Principal amount in dollars 135,000 Add : Interest @ 6% for 3 months (135,000 x 6% x 3/12) 2,025 Total amount in dollars to paid after 3 months. 137,025 The company will buy today 137,025 dollars in the forward market payable after three months. Two rates viz. 48.81 / 48.83 are available. Which one would you use and why? The bank will sell dollars to the company. It will sell every dollar at Rs. 48.83 per dollar (not Rs. 48.81). In all it will require 137,025 x 48.81 = Rs. 66,90,931. Amount in Rs. (137,025 x 48.83) 6,690,931 Recommendation : Amount required as per option 1 (pay today) in Rs. 6,789,744 Amount required as per option 2 (go forward) 6,690,931 Saving in option 2 (go forward) 98,813 Option 2 i.e. pay after 3 months and buy forward is recommended in the light of above table. Similar problem 1: An Import house in India has bought goods from Switzerland for SF 10,00,000. The exporter has given the Indian company two options: (i) Pay immediately the bill for SF 10,00,000 (ii) Pay after 3 months, with interest @ 5% p.a. The importer's bank charges 14% on overdrafts. If the exchange rates are as follows, what should the company do? Spot (Rs. / SF) 30.00 / 30.50 3 month (Rs. / SF) : 31.10 / 31.60 Answer : Option 1 : Rs. 315,67,500 Option 2: Rs. 319,95,000 Similar problem 2: Electronic Ltd., your customer has imported 5,000 cartiges at landed cost in Mumbai of US $ 20 each. The company has the choice for paying for the goods immediately or in 3 months time. It has a clean overdraft limit with you where 14% p.a. rate of interest is charged. Calculate which of the following methods would be cheaper to your customer: (i) Pay in 3 months time with interest @ 10% and cover risk forward for 3 months. (ii) Settle now at a current spot rate and pay interest of the overdraft for 3 months. The rates are as follows : Mumbai Rs. / $ spot : 43.25 43.55 and 3 months swap 35/25. Answer : Option 1 : Rs. 44,38,250 and Option 2: Rs. 45,07,425

Note : When swap points are ascending they should be added to spot and when they are descending order they should be deducted from spot. In this case the swap points are descending order (35/25), they should be deducted from spot.

2002 Dec 12 b An Americal Multinational Corporation has subsidies whose cash positions for the month of September, 2002 are given below: Swiss subsidiary Cash surplus of SF 1,50,00,000 Canadian subsidiary Cash deficit of CAD 2,50,00,000 UK Subsidiary Cash deficit of UK pound 30,00,000. What are the cash requirements, if : (i) Decentralised cash management is adopted? (ii) Centralised cash management is adopted? Exchange rate : SF 1.48 / $; CAD 1.58 /$ , $1.57 / Pound. Solution : Tutorial notes : 1. Under decentralised system the surplus cash available is not available to cover the cash deficit of other subsidiaries. 2. The cash requirement will be reported in US dollars as the holding company is American. 3. SF 1.48/$ means 1.48 SF per dollar or simply one dollar = 1.48 SF. Cash requirement under decentralised system : Swiss subsidiary No cash requirement as it is already surplus. Canadian subsidiary Deficit is 250 lac CAD. It is to be reported in USD. As per exchange rate CAD 1.58 /$. It implies that 1.58 CAD per dollar or simply 1 USD = 1.58 CAD. You are to convert 250 lac CAD into USD with this conversion rate. The USD required for canadian subsidiary would be 250 lac / 1.58 = 158,22,785 USD UK Subsidiary Deficit is 30,00,000 UKPound. It is to be reported in USD. Exchange rate is given as 1.57 dollars per pound. You are to convert 30 lac pounds to USD at the rate of 1.57 doll Cash deficit in UK subsidiary : 30,00,000 x 1.57 = 47,10,000 dollars. Total cash requirement : Swill subsidiary 0 Canadian subsidiary 15822785 UK subsidiary 4710000 20532785 dollars Cash requirement under centralised system : Total cash requirement : USD in lacs Swill subsidiary Surplus (150,00,000/1.48) 101.35 Canadian subsidiary Deficit (158.23) UK subsidiary Deficit (47.10) Total cash requirement : (205.33) lac USD. Problem 8: (ca final nov 06) Your bank wants to calculate Rupee. TT selling rate of exchange for DM since a deposit of DM 100,000 in a FNCR a/c has matured, when : Euro 1 = DM 1.95583 locked in rate Euro 1 = USD 1.02348 / 43 USD 1 = Rs. 48.51 / 53 What is the Rupee TT selling rate for DM currency ? Solution : 1. Convert DM 100,000 into Euro. The equation is 1 Euro = 1.95583 DM 100,000 DM = 100,000 / 1.95583 = 51,129.2 Euro. 2. Convert Euro into Dollars. Bank will pay dollars hence it will pay 1.02343 dollars for every Euro. 51,129.2 Euros = 51,129.2 x 1.02343 = 52,327.1 dollars. 3. Convert dollars into rupees. Bank will pay rupees and receive dollars. For every dollar, it will pay Rs. 48.51. For 52,327.1 dollars it will pay 52,327.1 x Rs. 48.51 = Rs. 25,38,390. The Conversion rate is 100,000 DM = 25,38,390 Or 1DM = 25.3839 Rs.. 2003 June 12a:

During the year the price of British Gilts (face value 100 pounds) rose from 105 pounds to 110 pounds while paying a coupon of 8 pounds. At the same time the exchange rate moved from dollar/pound of 1.80 to 1.70. What is the total return to an investor in USA who invested in this security? Solution : Tutorial notes : 1. The investor is in USA. She will have dollars which she will exchange in pounds, these pounds will be invested in British Gilts.(Gilts mean government based security). 2. Let us assume that the investor has 1000 USD to invest. First of all she will convert the dollars into pounds. Two rates are given viz. dollar / pound of 1.80 or 1.70. Which one will you use and why? 3. The pounds so obtained will be invested in Gilts of face value 100 pounds per security. How many securities the investor will purchase ? What rate would you use for purchase of security i.e. face value of 100 or market value of 105 or 110? 4. What do you understand by 'a coupon of 8 pounds'? Solution goes as given below: A coupon of 8 pounds' , it is rate of interest on that security. On one security (or bond) the interest of 8 pounds will be paid in a year. Suppose the investor has 1,000 USD to invest. You can take any other figure as well. The current rate is one pound = 1.80 dollars. The investor will receive 1000 / 1.80 = 555.55 pounds on convertion. The security is available at 105 pounds per security. On 555.55 pounds, she will receive 555.55 / 105 = 5.29 security. Rate of interest is a coupon of 8 pounds. The investment is for one year as given in question. The interest at the rate of 8 pounds per security per year would be calculated as 5.29 x 8 = 42.32 pounds. Rate of security is increasing i.e. there is also a capital gain per security of pound 5 (110 105). On 5.29 security, the capital gain would be 5.29 x 5 = 26.45 pounds. You should present the solution in a tabular form as given below: Let the amount to be invested is 1000 USD. Pounds Investment in pounds @ one pound = 1.8 dollars : 1000 / 1.8 555.55 Nol of securities @ 105 pounds = one security 555.55 / 105 = 5.29 Coupon on security for one year @ 8 pounds per security 5.29 x 8 42.32 Capital gain @ 5 pounds per security for 5.29 security 5.29 x 5 26.45 Total pounds had by investor at the end of the year 624.32 Conversion into dollars @ 1.7 dollars per pound 624.32 x 1.7 1061.344 Less : Initial investment in dollars 1000.000 Net gain 61.344 Net gain in % 61.44 / 1000 6.13% 2003 Dec 2b(i) Interest rates for 3 months in US and Canada are as follows : Currency Borrow Invest CAD / USD Spot : 1.235 1.240 USD 4.00% 2.50% CAD / USD 3 month forward : 1.255 1.260 CAD 4.50% 3.50% Advise the currency in which borrowing and leding for 3 months needs to be done for a US company. Take 3 months as 90 days or 0.25 year. Solution ; Take the same solution. 2003 Dec 5a: An Import house in India has bought goods from Switzerland for SF 10,00,000. The exporter has given the Indian company two options: (i) Pay immediately the bill for SF 10,00,000 (ii) Pay after 3 months, with interest @ 5% p.a. The importer's bank charges 14% on overdrafts. If the exchange rates are as follows, what should the company do? Spot (Rs. / SF) 30.00 / 30.50 3 month (Rs. / SF) : 31.10 / 31.60

Solution : Take the same solution. 2003 Dec 8a Bansali . . Give reasons. Solution : Already solved in Chapter 3. 2004 June 5b: Take the same solution. 2004 June 6: For imports from UK, Philadelphia Ltd. of USA owes 650,000 pounds to London Ltd. payable on May 2004. It is now 12 February, 2004. The following futures contracts (contract size 62,500 pounds) are available on the Philadelphia exchange: Expiry Current futures rate March 1.4900 dollars/pound June 1.4960 dollars/pound (a) Illustrate how Philadelphia Ltd. can use future contracts to reduce the transaction risk if, on 20 May the spot rate is 1.5030 dollar/pound and June futures are trading at 1.5120 dollar/pound. The spot rate on 12 February is 1.4850 dollar/pound. (b) Calculate the hedge efficiency and comment on it. Solution : Take the same solution. 2004 Dec 4(a) An investor purchased.. Investor was logical? Solution: Take the same solution. 2004 Dec 7b: The annual interest rate is 5% in the United States and 8% in the UK. The spot exchange rate is pound / dollar = 1.50 and forward exchange rate, with one year maturity is pound / dollar = 1.48. In view of the fact that the arbitrage can borrow 10,00,000 dollars at current spot rate, what would be the arbitrage profit or loss? Solution : Take the same solution. 2005 June 2a: What is covered Solution : Take the same solution. 2005 June 7b : A proposed foreign investment involves a plant whose entire output of 1 million units per annum is to be exported. With a selling price of 10 USD per unit, the yearly revenue from this investment equal 10 million USD. At present rate of exchange, dollar costs of local production equal to 6 USD per unit. 10% devaluation is expected to lower unit costs by 0.30 USD, while 15% devaluation will reduce these costs by an additional 0.15 USD. Suppose a devaluation of either 10% or 15% is likely, with respective probabilities of 0.4 and 0.2 (the probability of no exchange rate is 0.4). Depreciation at the current exchange rate equals Rs. 1 million annually, while local tax rate is 40%. (a) What will annual dollar cash flows (after tax) be under each exchange scenario? (b) What is the expected value of annual after-tax dollar cash flows assuming no repatriation of profits to the United States? (c) Considering that the project involves a total investment of 25 million USD on plant and working capital, would you recommend the investment ? Answer this question assuming that expected annual dollars cash flows as worked out in (b) above, would continue in perpetuatity at a growth rate of 2%, assuming that the minimum required return on investment is 12%. Solution : It is an easy question. You should try to solve it before you see below.

The current cost is 6 USD per unit. At 10% devaluation the cost would be reduced by 0.30 USD per unit or 5.70 USD per unit. 15% devaluation would reduce the cost further by 0.15 USD or the unit cost would be 5.55 USD per unit. Statement of Cash flows : In million USD Revenue 10.00 10.00 10.00 Less : Cost 6.00 5.70 5.55 Depreciation 1.00 1.00 1.00 Profit before tax 3.00 3.30 3.45 Tax @ 40% 1.20 1.32 1.38 PAT 1.80 1.98 2.07 Cash flows (PAT + Depreciation) 2.80 2.98 3.07 Probabilities 0.40 0.40 0.20 1.12 1.19 0.61 2.93 USD million (c) If the growth rate is 2% per annum perpetually, the required rate of return would be 12% 2% = 10%. The total cash inflows of 2.93 USD million would create a wealth of 2.93/0.10 = 29.3 USD million. The net investment required is 25 million USD. Thus the npv of the project works out to Rs. 4.30 million USD. The project is recommednde to be taken up. 2005 June 8a An Indian exporter. decision ? Solution : Take the same solution. 2005 Dec 6a Consider the following . Still exists ? Solution : Take the same solution. 2005 Dec 6b: Company A has .. Citi Bank receive? Solution : Take the same solution. 2005 Dec 8b Zenith LTd Zenith Ltd. Solution : Take the same solution. 2006 June 3a: The shares of .. Expiration ? Solution : Take the same solution. 2006 June 6a Electronic Ltd., your customer has imported 5,000 cartiges at landed cost in Mumbai of US $ 20 each. The company has the choice for paying for the goods immediately or in 3 months time. It has a clean overdraft limit with you where 14% p.a. rate of interest is charged. Calculate which of the following methods would be cheaper to your customer: (i) Pay in 3 months time with interest @ 10% and cover risk forward for 3 months. (ii) Settle now at a current spot rate and pay interest of the overdraft for 3 months. The rates are as follows : Mumbai Rs. / $ spot : 43.25 43.55 and 3 months swap 35/25.
Solution : Tutorial Notes : 1. The solution is very easy provided you know how to calculate the bid/ask rate from swap points. Can you determine the bid/ask rate Rs./USD? 2. Whenever swap points are given, you should determine the bid/ask rate as follows: if the swap points are in Ascending Order, ADD the swap points while if they are in the Descending Order as is the case now, DEDUCT the swap points. This is the simple and unambiguous method. 3. In the present case, the swap points are in Descending Order hence Deduct the swap points. Thus the bid rate would be 43.25 less 35 points = Rs. 42.90 and Ask rate would be 43.55 less 25 points = Rs. 43.30. Thus after three months, the rates of Rs. / USD would be 42.90 43.30. You can evaluate both the options on your own and compare your evaluation with the one given below: Amount to be paid : 5,000 x 20 USD = 100,000 USD. Option 1: Pay 3 months later and pay interest @ 10% per annum

Interest @ 10% per annum for 3 months on 100,000 USD USD 2,500 Principal USD 100,000 Total to be paid after three months USD 102,500 You are paying rupees to bank to purchase or buy USD. Bank will charge higher rate for each USD i.e Rs. 43.30 for one USD. Total cost in Rs. for Option 1 43.30 x 102,500 Rs. Option 2: Pay now and pay interest @ 14% for 3 months If you pay now, the USD is available at Rs. 43.55 (Not Rs. 43.25) per USD. Principal 100,000 x Rs. 43.55 Add: Overdraft interest @ 14% per annum for 3 months 14% x 3/12 x 43,55,000 Total cost in Rs. for Option 2 Rs. Additional cost of Option 2 Recommendation : Option 1 will be more economical to the customer by Rs. 69,175

2006 June 6b:


The finance director of Molson Ltd. has been studying exchange rates and interest rates relevant in India and USA. Molson Ltd. has purchased goods from the US company at a cost of $ 40.50 lakhs payable in $ in 3 months time. In order to maintain profit margins the finance director wishes to adopt, if possible, a risk-free strategy that will ensure that the cost of goods to Molson Ltd. is no more than Rs. 18 crores. Rs. / $ Spot : 41 / 43 ; Rs. / $ (1 month forward) : 42 / 44 and Rs. / $ (3 months forward) : 43 / 46 Interest rates are : India Rates in % USA Rates in % Deposit Borrowing Deposit Borrowing 1 Month 9.0 12.0 4.0 7.0 3 Months 9.0 13.0 5.0 8.0 Calculate whether it is possible for Molson Ltd. to achieve a cost directly associated with transaction not more than Rs. 18 crores, by means of a forward market hedge or money market hedge. Ignore transaction costs. Solution : Tutorial Notes : 1. The first strategy is obviously to go to forward market, buy three months forward USD today and sleep well. Can you tell what will be the rate at which Molson can buy 3-months forward? 2. Three months forward rates are given as Rs. / USD : 43 / 46. Which one of the two, 43 or 46, will you use to determine the amount payable for purchase of 40.50 lakhs USD? 3. The bank always wins in selling and buying both. Molson Ltd. wants to purchase USD from bank, bank will sell USD at higher of the two rates i.e. Rs. 46 per USD. It will ask Molson Ltd. to pay Rs. 46 x 40.50 = Rs. 1863 lakhs to buy 40.50 lakhs USD. 4. Thus Molson Ltd. will pay Rs. 1863 lakhs to receive 40.50 lakhs USD. Does that mean than if Molson Ltd. returns these 40.50 lakhs USD to bank, it will receive back Rs. 1863 lacs? NO, because now the bank will say that it will pay Rs. 43 (lower of the two) for each USD. Thus Molson Ltd. will receive Rs. 1741.5 lakhs (40.50 x 43) and Not Rs. 1863. This is not related to the problem, but it is supplied as additional knowledge. 5. So you have determined the cost of 3-months forward. The next strategy is money market strategy. Do you remember the various steps required to be taken to complete the money market hedge? Try yourself before you see any further. 6. Steps relating to Money market hedge: (a) Molson needs 40.50 lakh USD after 3 months. The 3 month deposit rate in USA is 5.0% per annum. Molson should deposit such deposit in USA which will mature to 40.5 lakh USD in 3 months @ 5% per annum. Obviously Molson should deposit 40.50 / 1.0 + 5% x 3/12) = USD 40.0 lakhs now. (b) To get 40 lakh USD today, Molson will go to bank where the spot rate (today's rate) is 41 / 43. Which one will be suitable for receiving USD from bank? The bank will give one USD at the payment of Rs. 43 (not Rs. 41). The Molson will need today 43 x 40 = Rs. 1720 lakhs to receive 40 lakhs USD from bank. (c) Now Molson will borrow Rs. 1720 lakhs in India at 13% for 3 months. Thus total cost of this strategy would be Rs. 1720 lakhs + Interest at 13% per annum for 3 months. Total cost comes to Rs. 1776 lakhs. Cost of 3 months forwards Rs. 1,863 lakhs Cost of Money market hedge Rs. 1,776 lakhs Difference 87 lakhs The money market hedge would be more economical to the tune of Rs. 87 lakhs. Recommended.

2006 June 6b:

On August 2.. End of the day. Solution : Take the same solution. 2006 Dec 2a An investor purchased.. Investor was logical? Solution : Take the same solution. 2006 Dec 5a
Given the following information : Spot rate Rs. 46.88 / USD 3 - month forward rate Rs. 47.28 / USD 3 - month interest rate in USA 7% per annum 3 - month interest rate in India 9% per annum Assuming no transaction cost or taxes exist, what operation would be carried out to take the possible gain ? Assume Rs. 10.0 million USD to explain your answer. Solution : If the difference between interest rates is lower than the forward premium, the possibility of arbitrage gain exists. First compute the forward premium and then compare it with the interest rate. Interest rate difference : 9% 7% = 2%. Rate of forward premium per annum : [(47.28 46.88) / 46.88] x 12/3 = 3.41%. Forward premium is more, the possibility of inflow to USA exists. Alternatively the possibility of arbitrage gain can also be assessed by using IRPT equation: ( 1 + 0.0225) / ( 1 + 0.0175) = Forward / 46.88 If you are confused about the figure of 0.0225 and 0.0175 in the above formula, revise your knowledge about IRPT. (these represent interest for 3 months) As the maturity of forward is 3 months, the interest rates have been taken for 3 months in the IRPT equation. Forward as per the equation = Rs. 47.11. This is less than the actual forward rate of 47.28. This means the dollar is overvalued in the forward market, Hence it is profitable to investing in USA by borrowing rupees. Following steps need be taken to take arbitrage gain : (i) Borrow Rs. 10.0 million at 9% for 3 months. Amount to be paid after 3 months will be Rs. 10.0 million x ( 1 + 0.09 x 3/12) = Rs. 10.225 million. (ii) Convert Rs. 10.0 million into USD at Rs. 46.88 per dollar. USD received would be 0.21331 million. (iii) Place 0.21331 million USD in the money market for 3 months to obtain 0.21331 x ( 1 + 0.07 x 3/12) = 0.21704 million USD. (iv) Sell USD 0.21704 million in the forward market at Rs. 47.28 per dollar to receive 10.2616 million USD. (v) Refund the debt taken with interest i.e. Rs. 10.225 million. (vi) Pocket the net arbitrage gain ( 10.2616 10.225) = Rs. 0.0366 million.

2006 Dec 5b NBA Bank Ltd of the forward contract. Solution : This is exactly similar to the following problem. Leojileojileo Bank Ltd. transacted on March 19, the following : (a) Sold USD 100,000 two months forward to Kareena Company at Rs. 44.50 (b) Purchased Euro 100,000 two months forward from Saif company at Rs. 47.20 On May 19, 2006, both the customers approached the bank. Kareena Ltd wants the forward contract to be cancelled while Saif Ltd wants the contract to be extended by one month. The following exchange rates prevailed on that day : Rs. / USD Rs. / Euro Spot 44.60 / 65 47.75 / 85 Forward 44.75 / 44.85 48 / 48.20 Based on the information given above (ignore interest etc.), you are required to : (a) Calculate the amount to be paid to or recovered from Kareena Company Ltd. due to cancellation of forward contract. (b) Calculate the amount to be paid to or recovered from Saif Ltd. due to extension of forward contract.
The solution goes as follows:

First part is very easy. The Kareena company is cancelling the contract. In other words, the Kareena company is selling 100,000 USD to Bank at the spot rate. The bank will buy USD at Rs. 44.60. The agreemented rate was for Rs. 44.50 per USD. The difference of Rs. 0.10 per dollar is Payable to Kareena Ltd. The above cancellation can also be represented as follows: Kareena bought 3 months forward dollars from the Bank at Rs. 44.50 value May 19, 2006. Now the company requests cancellation. Thus the bank has to square of the purchase of USD made for Kareena ltd. Thus the bank will sell USD at Rs. 44.60 per dollar. Here the bank is customer for the dealer from whom it had purchased dollars for Kareena. Rate of the original contract 44.50 Rate for cancellation 44.60 Difference 0.10 Amount to be paid to the Kareena Ltd. 0.10 x 100,000 = Rs. 10,000
Second Part :

Here the Saif Ltd. wishes to extend the contract by one month. Earlier the Saif Ltd had sold 100,000 Euros at Rs. 47.20 per Euro. To cancel the contract, the Saif ltd will purchase the contracted Euros from the bank at the spot rate prevailing at the time of cancellation of the contract. Rate at the time of contract 44.20 Rate at the time of cancellation of contract 44.85 Difference 0.65 Amount to be paid by the bank to Saif Ltd. 100,000 x 0.65 65,000 Now the bank would sell forward value 19th June @ Rs. 48.0 per Euro value new date. 2007 June 2b: The settlement price of the contracts. Solution : Take the same solution. 2007 June 6a Carlhams corporation Ltd corporation Ltd. Solution : Take the same solution. 2007 June 6b Yorkshire Industries. Period of the swap? Solution : Take the same solution. 2007 Dec 6a A forex trader wants.. cost or taxes exist ? Solution : Take the same solution. 2007 6b Fresco corporation costs or taxes. Solution : Take the same solution. 2008 June 3b: In March,2008.. due in September,2008? Solution : Take the same solution. 2008 June 5a Sunshine Ltd. Point of view? Solution : Take the same solution. 2008 June 5b Mumbai Ltd. is an. Of 2:1. Solution : Take the same solution. 2008 Dec 7a
Unilever's subsidiary in India, Hindustan Lever, procures most of its soaps from a Japanese company, Because of the shortage of working capital in India, payment terms for the Indian importers are typically 180 days or more. Hindustan Lever wishes to hedge a 8.5 million Japanese Yen payable.

Although the options are not available on the Indian Rupee (Rs.), forward rates are available against the Yen. Additionally, a common practice in India is, for companies like Hindustan Lever, to work with currency agent who will, in this case, look in the current spot exchange for a 4.85% fee. Using the following data, recommend a hedging strategy. Spot rate USD/JPY Yen 120.60/USD Spot rate USD/INR Rs. 47.75/USD 180-day forward rate JPY/INR Re.0.4166/Yen Expected spot exchange rate in 180 days Re.0.3846/Yen 180-day Yen Investment rate 1.5% 180-day rupee investment rate 8.0% Cost of Capital 12.0% Solution : Tutorial Notes : 1. Can you think how many strategies are available for the present purpose? 2. Can you compute the implied or indirect spot rate for Yen per Rupee from the data? 3. One strategy is to buy 180-day forward Yen for 8.5 million Yen, and sleep well. Can you compute how much amount in Rs.you will require for this strategy? 4. Another strategy is to go to Money Market. Purchase Yen and invest it for 180 days so as to receive 8.5 million Yen at maturity. How much Rs.will you need for this purpose? 5.What is use and significance of Cost of capital in these strategies? 6. Other strategy is to go to agent and pay her fees. How much Rs. would you require for this purpose? What will you do with cost of capital under this strategy? 7. One strategy may be not to worry today at all. Remain uncovered and wait for six months. When the time of payment comes, buy Yen from spot market and pay. What calculations will be required against this strategy? 8. You must make ample working notes in your answer. 9. You must try to solve this problem and compare your solution with the one given below. The solution goes as follows: Note that money transaction cost is altogether ignored. Strategy 1 : Remain uncovered remain uncertain You wait for 180 days and purchase 8.5 million Yen from the spot market at the prevailing rate whatever that may be and pay. You are totally uncertain about your liability today and that is a great risk. Today Yen is indirectly available via USD. You purchase USD at Rs. 47.75 per USD. Yen is available at 120.60 Yen per USD. Your equation of conversion is simply Rs. 47.75 = 120.60 yen. Thus indirectly you can receive Yen at 120.60 / 47.75 = 2.5257 Yen per Rupee. You can approximately compute your liability as 8.5 million / 2.5257 = Rs. 33,65,464 Following points should be considered in this strategy: 1. If the spot rate remains same in 180-day period. You liability does not undergo any change. It is risky to the extent of uncertainty. 2. If the spot rate remains same as forwar rate The 180-day forward rate is Re.0.4166 per yen. Your liability becomes 85 lacs Yen x 0.4166 = Rs. 35,41,100. It is again risky as it is not certain. Note here that JPY/INR is given as 0.4166. If you change it to INR/JPY = 1/0.4166 = 2.4 Yen per Rupee. You need 85,00,000 / 2.4= Rs. 3541666. The difference is due to approximation. Both ways are correct. If you take the correct figure of 2.40038, the figure will be Rs. 35,41,100. 3. The expected spot rate is Re. 0.3846/Yen. You will require 8500,000 x 0.3846 = Rs. 32,69,100. It is risky due to its uncertainty. You should present the above calculations in a tabular form. Strategy 2: Buy Japanese Yen forward 180 days and sleep well. You can go to 180-day forward market, pay amount today and become certain that you will receive the required 8.5 million Yen at the required time and sleep well. 180-day forward rate is Re.0.4166 per Yen. You pay 85,00,000 x 0.4166 = Rs. 35,41,100 today and become certain. Strategy 3: Go to Money Market The Yen investment rate is 1.5%. The rate is per annum. You can invest certain amount carrying interest of 1.5% per annum so as to receive 8.5 million Yen on maturity after 180 days. You will require 8.5 million / 1.0075 = 84,36,760 Yen to invest. If you invest 84,36,760 yen today for 180 days (say half year) at 1.5% per annum interest rate, you will receive 8.5 million Yen at the end of 180 days.

To arrange 84,36,760 Yen today you have to go to Money market pay Rs. at the rate of 120.60/47.75 = 2.5257 Yen per Rupee to receive Yen. You are required to pay 84,36,760 / 2.5257 = Rs. 33,40,365. Here comes the use and significance of cost of capital. The money is available at 12%. Thus for half year the cost is 6%. You are paying 33,40,365 x 1.06 = Rs. 35,40,787. The cost of this strategy is thus Rs. 35,40,787. Strategy 4: Go to Indian Currency Agent In this case you purchase the required Yen from the spot market (note that Yen is available in spot at 2.5257 Yen per rupee as computed earlier). You also pay fee to Indian agent at 4.85% of your present investment. Agent is paid to bear the fluctuations risk. Thus interest at 12% per annum (cost of capital) or 6% per half year will have to included in agent's fees. Total cost can be computed as given below: Principal payable (8500,000/2.5257) Rs. 3,365,404 Add: Agent's fees at 4.85% Rs. 163,222 Cost of agent's fees (6% for 180 days) Rs. 9,793 Rs. 3,538,419 You should present the results in a tabular form: (This carries marks in the examination) Strategy Particulars Cost in Rs. Comment 1 Remain uncovered Uncertain Uncertain and risky 2 Buy Forward 3,541,100 Certain 3 Go to Money Market 3,540,787 Certain 4 Go to Agent 3,538,419 Certain Least cost Recommendation : The strategy 4 : Go to Agent is causing the least cost of Rs. 35,38,419 is recommended.

2009 June 3b: X Company Ltd..City Bank receive? Solution : Take the same solution. 2009 June 6b: The Ferguson systems was. would you adopt? Solution : Take the same solution. 2009 Dec 7b Wilson Ltd of hedging. Solution : Take the same solution. 2010 June 6b Vedika Traders. merchant quotes. Solution : Take the same solution.

mpany two options :

means the company num. How much the rates given i.e. 48.35

e bank. The bank will

months. Two rates viz.

not Rs. 48.81). In all it

0,00,000. The

hat should the

ed cost in Mumbai of months time. It has a

e descending order 5/25), they should be

h of September, 2002

cash deficit of other

= 1.58 CAD. You are

SD at the rate of 1.57 dollars = 1 pound..

of DM 100,000 in a

it will pay Rs. 48.51.

to 110 pounds while ound of 1.80 to 1.70.

pounds will be

e dollars into pounds.

. How many .e. face value of 100

e interest of 8

55 pounds on

555.55 / 105 = 5.29

stion. The interest at ds. 105). On 5.29

US company. Take 3

orter has given the

hat should the

yable on May 2004. It

elphia exchange:

k if, on 20 May the e spot rate on 12

e rate is pound /

hat would be the

per annum is to be ment equal 10 million it.

ill reduce these costs espective probabilities exchange rate equals

iation of profits to the

and working capital, d annual dollars cash %, assuming that the

0 USD per unit or nit cost would be 5.55

USD million 2% 2% = 10%. The ion. The net million USD. The

of US $ 20 each. The has a clean overdraft

Can you determine the

points are in Ascending T the swap points. This

hus the bid rate would after three months, the

D i.e Rs. 43.30 for one 4,438,250

4,355,000 152,425 4,507,425 69,175

n India and USA. 3 months time. In order will ensure that the cost

n not more than Rs. 18

sleep well. Can you

ll you use to determine

k, bank will sell USD at 3 lakhs to buy 40.50

olson Ltd. returns these at it will pay Rs. 43 Not Rs. 1863. This is not

egy. Do you remember ou see any further.

num. Molson should bviously Molson should

Which one will be

x 40 = Rs. 1720 lakhs

ategy would be Rs.

9% per annum ble gain ?

ge gain exists. First

edge about IRPT.

T equation.

A by borrowing rupees.

million x ( 1 + 0.09 x

on. 3/12) = 0.21704 million

on USD.

orward contract to be wing exchange rates

cancellation of

ward contract.

words, the Kareena D at Rs. 44.60. The Payable to Kareena

06. Now the company Kareena ltd. Thus the er from whom it had

sold 100,000 Euros at uros from the bank at

any, Because of the or more. Hindustan

st the Yen. Additionally, o will, in this case, look tegy.

e how much amount in

eive 8.5 million Yen at

ose? What will you do

n the time of payment ?

atever that may be and

ble at 120.60 Yen per e Yen at 120.60 / 47.75 . 33,65,464

35,41,100. It is again

Rupee. You need u take the correct

It is risky due to its

the required 8.5 million

d become certain.

nterest of 1.5% per 075 = 84,36,760 Yen to ate, you will receive 8.5

75 = 2.5257 Yen per

alf year the cost is 6%.

at 2.5257 Yen per Agent is paid to bear to included in agent's

ommended.

S-ar putea să vă placă și